Π—Π°ΠΊΠΎΠ½ ΠΎΠΌΠ° всС Ρ„ΠΎΡ€ΠΌΡƒΠ»Ρ‹ ΠΈ опрСдСлСния: Π—Π°ΠΊΠΎΠ½ Ома | ВсС Ρ„ΠΎΡ€ΠΌΡƒΠ»Ρ‹

Π‘ΠΎΠ΄Π΅Ρ€ΠΆΠ°Π½ΠΈΠ΅

Π—Π°ΠΊΠΎΠ½ Ома для ΠΏΠΎΠ»Π½ΠΎΠΉ Ρ†Π΅ΠΏΠΈ: ΠΎΠΏΡ€Π΅Π΄Π΅Π»Π΅Π½ΠΈΠ΅ для Π·Π°ΠΌΠΊΠ½ΡƒΡ‚ΠΎΠ³ΠΎ участка

Одним ΠΈΠ· ΠΏΡ€ΠΈΠ½Ρ†ΠΈΠΏΠΎΠ² элСктротСхники являСтся Π·Π°ΠΊΠΎΠ½ Ома для ΠΏΠΎΠ»Π½ΠΎΠΉ Ρ†Π΅ΠΏΠΈ. Π˜ΡΠΏΠΎΠ»ΡŒΠ·ΡƒΡ ΡƒΡΡ‚Π°Π½ΠΎΠ²Π»Π΅Π½Π½ΡƒΡŽ ΡƒΡ‡Ρ‘Π½Ρ‹ΠΌ Π·Π°ΠΊΠΎΠ½ΠΎΠΌΠ΅Ρ€Π½ΠΎΡΡ‚ΡŒ, ΠΌΠΎΠΆΠ½ΠΎ Π²Ρ‹Ρ‡ΠΈΡΠ»ΠΈΡ‚ΡŒ сопротивлСниС элСктричСской Ρ†Π΅ΠΏΠΈ ΠΈΠ»ΠΈ источника Ρ‚ΠΎΠΊΠ°, Ρ€Π°ΡΡΡ‡ΠΈΡ‚Π°Ρ‚ΡŒ ΡΠ»Π΅ΠΊΡ‚Ρ€ΠΎΠ΄Π²ΠΈΠΆΡƒΡ‰ΡƒΡŽ силу (Π­Π”Π‘). ΠŸΡ€Π°ΠΊΡ‚ΠΈΡ‡Π΅ΡΠΊΠΎΠ΅ ΠΆΠ΅ ΠΏΡ€ΠΈΠΌΠ΅Π½Π΅Π½ΠΈΠ΅ ΠΏΠΎΠ»ΡƒΡ‡Π΅Π½Π½Ρ‹ΠΌ ΠΏΡ€ΠΈ расчётС Π΄Π°Π½Π½Ρ‹ΠΌ Π²Π΅Π»ΠΈΠΊΠΎ. Π­Ρ‚ΠΎ ΠΏΠΎΠ΄Π±ΠΎΡ€ ΡˆΡƒΠ½Ρ‚ΠΈΡ€ΡƒΡŽΡ‰ΠΈΡ… ΠΈ ΠΏΡ€Π΅Π΄ΠΎΡ…Ρ€Π°Π½ΠΈΡ‚Π΅Π»ΡŒΠ½Ρ‹Ρ… элСмСнтов, вычислСниС Π½Π΅ΠΎΠ±Ρ…ΠΎΠ΄ΠΈΠΌΠΎΠΉ мощности ΠΈΡΠΏΠΎΠ»ΡŒΠ·ΡƒΠ΅ΠΌΡ‹Ρ… Π΄Π΅Ρ‚Π°Π»Π΅ΠΉ, согласованиС элСктронных ΡƒΠ·Π»ΠΎΠ².

Π˜ΡΡ‚ΠΎΡ€ΠΈΡ открытия

Π—Π°Π²ΠΈΡΠΈΠΌΠΎΡΡ‚ΡŒ ΠΌΠ΅ΠΆΠ΄Ρƒ Ρ‚ΠΎΠΊΠΎΠΌ, напряТСниСм ΠΈ сопротивлСниСм Π² элСктричСской Ρ†Π΅ΠΏΠΈ Π±Ρ‹Π»Π° установлСна ΠΎΠΏΡ‹Ρ‚Π½Ρ‹ΠΌ ΠΏΡƒΡ‚Ρ‘ΠΌ Π² 1827 Π³ΠΎΠ΄Ρƒ. Π—Π°Π½ΠΈΠΌΠ°ΡΡΡŒ исслСдованиями элСктричСства, Π“Π΅ΠΎΡ€Π³ Π‘ΠΈΠΌΠΎΠ½ Ом ΠΏΡ€ΠΎΠ²ΠΎΠ΄ΠΈΠ» ряд экспСримСнтов Π½Π°Π΄ ΠΏΡ€ΠΎΠ²ΠΎΠ΄Π½ΠΈΠΊΠ°ΠΌΠΈ, изучая ΠΈΡ… ΠΏΡ€ΠΎΠ²ΠΎΠ΄ΠΈΠΌΠΎΡΡ‚ΡŒ, ΠΈ Π² частности, ΠΏΠΎΠ΄ΠΊΠ»ΡŽΡ‡Π°Ρ ΠΏΠΎΡΠ»Π΅Π΄ΠΎΠ²Π°Ρ‚Π΅Π»ΡŒΠ½ΠΎ ΠΊ источнику энСргии Ρ‚ΠΎΠ½ΠΊΠΈΠ΅ ΠΏΡ€ΠΎΠ²ΠΎΠ΄Π½ΠΈΠΊΠΈ, Π²Ρ‹ΠΏΠΎΠ»Π½Π΅Π½Π½Ρ‹Π΅ ΠΈΠ· Ρ€Π°Π·Π»ΠΈΡ‡Π½Ρ‹Ρ… ΠΌΠ°Ρ‚Π΅Ρ€ΠΈΠ°Π»ΠΎΠ². ИзмСняя ΠΈΡ… Π΄Π»ΠΈΠ½Ρƒ, ΠΎΠ½ ΠΏΠΎΠ»ΡƒΡ‡Π°Π» ΠΎΠΏΡ€Π΅Π΄Π΅Π»Ρ‘Π½Π½ΡƒΡŽ силу Ρ‚ΠΎΠΊΠ°. Им Π±Ρ‹Π»ΠΎ установлСно, Ρ‡Ρ‚ΠΎ Π½Π° Ρ€Π΅Π·ΡƒΠ»ΡŒΡ‚Π°Ρ‚Ρ‹ экспСримСнтов влияСт источник элСктричСской энСргии, сопротивлСниС ΠΊΠΎΡ‚ΠΎΡ€ΠΎΠ³ΠΎ ΠΎΠΊΠ°Π·Ρ‹Π²Π°Π»ΠΎΡΡŒ Π²Ρ‹ΡˆΠ΅, Ρ‡Π΅ΠΌ Ρƒ ΠΈΡΠΏΠΎΠ»ΡŒΠ·ΡƒΠ΅ΠΌΡ‹Ρ… Π² ΠΎΠΏΡ‹Ρ‚Π°Ρ… ΠΏΡ€ΠΎΠ²ΠΎΠ΄Π½ΠΈΠΊΠΎΠ².

По совСту своСго наставника ΠŸΠΎΠ³Π³Π΅Π½Π΄ΠΎΡ€Ρ„Π° Ом собрал Ρ‚Π΅Ρ€ΠΌΠΎΡΠ»Π΅ΠΊΡ‚Ρ€ΠΈΡ‡Π΅ΡΠΊΡƒΡŽ Π±Π°Ρ‚Π°Ρ€Π΅ΡŽ, ΠΎΡ‚ΠΊΠ°Π·Π°Π²ΡˆΠΈΡΡŒ ΠΎΡ‚ использования химичСских элСмСнтов, ΠΏΡ€ΠΈΠΌΠ΅Π½ΠΈΠ² вмСсто Π½ΠΈΡ… ΠΎΡ‚ΠΊΡ€Ρ‹Ρ‚ΡƒΡŽ Π—Π΅Π΅Π±Π΅ΠΊΠΎΠΌ Ρ‚Π΅Ρ€ΠΌΠΎΠΏΠ°Ρ€Ρƒ мСдь-висмут. Для измСрСния ΠΏΠ°Ρ€Π°ΠΌΠ΅Ρ‚Ρ€ΠΎΠ² Ρ†Π΅ΠΏΠΈ ΠΈΠΌ использовались ΠΊΡ€ΡƒΡ‚ΠΈΠ»ΡŒΠ½Ρ‹Π΅ вСсы, с ΠΌΠ°Π³Π½ΠΈΡ‚Π½ΠΎΠΉ стрСлкой сконструированныС ΠšΡƒΠ»ΠΎΠ½ΠΎΠΌ.

На основании своих исслСдований Ρ„ΠΈΠ·ΠΈΠΊ-экспСримСнтатор ΠΏΡ€ΠΈΡˆΡ‘Π» ΠΊ Π²Ρ‹Π²ΠΎΠ΄Ρƒ, Ρ‡Ρ‚ΠΎ ΠΎΡ‚ΠΊΠ»ΠΎΠ½Π΅Π½ΠΈΠ΅ стрСлки зависит ΠΎΡ‚ ΠΎΠΏΡ€Π΅Π΄Π΅Π»Ρ‘Π½Π½ΠΎΠΉ силы, Π½Π°Π·Π²Π°Π½Π½ΠΎΠΉ силой Ρ‚ΠΎΠΊΠ°. Когда стрСлка ΠΎΡ‚ΠΊΠ»ΠΎΠ½ΡΠ»Π°ΡΡŒ, Ом Π·Π°ΠΊΡ€ΡƒΡ‡ΠΈΠ²Π°Π» вСсы Ρ‚Π°ΠΊΠΈΠΌ ΠΎΠ±Ρ€Π°Π·ΠΎΠΌ, Ρ‡Ρ‚ΠΎΠ±Ρ‹ ΠΎΠ½Π° Π²ΠΎΠ·Π²Ρ€Π°Ρ‰Π°Π»Π°ΡΡŒ Π² своё Π½Π°Ρ‡Π°Π»ΡŒΠ½ΠΎΠ΅ ΠΏΠΎΠ»ΠΎΠΆΠ΅Π½ΠΈΠ΅. Π£Π³ΠΎΠ», Π½Π° ΠΊΠΎΡ‚ΠΎΡ€Ρ‹ΠΉ Π·Π°ΠΊΡ€ΡƒΡ‡ΠΈΠ²Π°Π»Π°ΡΡŒ Π½ΠΈΡ‚ΡŒ, ΠΎΠ½ считал ΠΏΡ€ΠΎΠΏΠΎΡ€Ρ†ΠΈΠΎΠ½Π°Π»ΡŒΠ½ΠΎΠΉ силС Ρ‚ΠΎΠΊΠ°. ИзмСняя условия, Ом Π²Ρ‹Π²Π΅Π» ΠΌΠ°Ρ‚Π΅ΠΌΠ°Ρ‚ΠΈΡ‡Π΅ΡΠΊΡƒΡŽ Π·Π°Π²ΠΈΡΠΈΠΌΠΎΡΡ‚ΡŒ, составив ΡƒΡ€Π°Π²Π½Π΅Π½ΠΈΠ΅. ВыглядСло ΠΎΠ½ΠΎ ΡΠ»Π΅Π΄ΡƒΡŽΡ‰ΠΈΠΌ ΠΎΠ±Ρ€Π°Π·ΠΎΠΌ: Π₯ = Π°/b + x, Π³Π΄Π΅ Π·Π° Π₯ ΠΏΡ€ΠΈΠ½ΠΈΠΌΠ°Π»Π°ΡΡŒ сила, ΠΎΡ‚ΠΊΠ»ΠΎΠ½ΡΡŽΡ‰Π°Ρ ΠΌΠ°Π³Π½ΠΈΡ‚Π½ΡƒΡŽ стрСлку, Π·Π° Π° β€” Π΄Π»ΠΈΠ½Π° исслСдуСмого ΠΎΠ±Ρ€Π°Π·Ρ†Π°, Π° b+x ΠΎΠ±ΠΎΠ·Π½Π°Ρ‡Π°Π»ΠΈ ΠΈΠ½Ρ‚Π΅Π½ΡΠΈΠ²Π½ΠΎΡΡ‚ΡŒ ΠΈ ΡΡ‡ΠΈΡ‚Π°Π»ΠΈΡΡŒ постоянной Π²Π΅Π»ΠΈΡ‡ΠΈΠ½ΠΎΠΉ.

Π’ 1862 Π³ΠΎΠ΄Ρƒ Π² ΠΆΡƒΡ€Π½Π°Π»Π΅ Β«Π€ΠΈΠ·ΠΈΠΊΠ° ΠΈ химия» публикуСтся ΡΡ‚Π°Ρ‚ΡŒΡ Ома ΠΏΠΎΠ΄ Π½Π°Π·Π²Π°Π½ΠΈΠ΅ΠΌ Β«ΠžΠΏΡ€Π΅Π΄Π΅Π»Π΅Π½ΠΈΠ΅ Π·Π°ΠΊΠΎΠ½Π°, ΠΏΠΎ ΠΊΠΎΡ‚ΠΎΡ€ΠΎΠΌΡƒ ΠΌΠ΅Ρ‚Π°Π»Π»Ρ‹ проводят ΠΊΠΎΠ½Ρ‚Π°ΠΊΡ‚Π½ΠΎΠ΅ элСктричСство». Π Π΅Π·ΡƒΠ»ΡŒΡ‚Π°Ρ‚Ρ‹ Π΅Π³ΠΎ исслСдований Π½Π΅ производят впСчатлСния Π½Π° Π΄Ρ€ΡƒΠ³ΠΈΡ… ΡƒΡ‡Π΅Π½Ρ‹Ρ…, ΠΈ Π΅Π³ΠΎ Π²Ρ‹Π²ΠΎΠ΄Ρ‹ ΠΎΡΡ‚Π°ΡŽΡ‚ΡΡ Π½Π΅Π·Π°ΠΌΠ΅Ρ‡Π΅Π½Π½Ρ‹ΠΌΠΈ. Ом ΠΏΡ€ΠΎΠ΄ΠΎΠ»ΠΆΠ°Π΅Ρ‚ экспСримСнты, выясняя, Ρ‡Ρ‚ΠΎ элСктричСство ΠΌΠΎΠΆΠ½ΠΎ Ρ€Π°ΡΡΠΌΠΎΡ‚Ρ€Π΅Ρ‚ΡŒ Π½Π°ΠΏΠΎΠ΄ΠΎΠ±ΠΈΠ΅ Ρ‚Π΅ΠΏΠ»ΠΎΠ²ΠΎΠ³ΠΎ ΠΏΠΎΡ‚ΠΎΠΊΠ°. Подобно Ρ€Π°Π·Π½ΠΈΡ†Π΅ Ρ‚Π΅ΠΌΠΏΠ΅Ρ€Π°Ρ‚ΡƒΡ€, благодаря ΠΊΠΎΡ‚ΠΎΡ€ΠΎΠΉ ΡΠΎΠ²Π΅Ρ€ΡˆΠ°Π΅Ρ‚ΡΡ Ρ‚Π΅ΠΏΠ»ΠΎΠ²ΠΎΠ΅ Π΄Π²ΠΈΠΆΠ΅Π½ΠΈΠ΅, Π½Π΅ΠΊΠΎΠΉ Π²Π΅Π»ΠΈΡ‡ΠΈΠ½ΠΎΠΉ ΠΌΠΎΠΆΠ½ΠΎ ΠΎΠΏΠΈΡΠ°Ρ‚ΡŒ Π΄Π²ΠΈΠΆΠ΅Π½ΠΈΠ΅ элСктричСского заряда. Π’Π°ΠΊ ΠΎΠ½ Π²Π²ΠΎΠ΄ΠΈΡ‚ понятиС Π­Π”Π‘.

ΠžΡ‚ΠΊΡ€Ρ‹Ρ‚ΠΈΠ΅ Ома Π±Ρ‹Π»ΠΎ принято ΡƒΡ‡Ρ‘Π½Ρ‹ΠΌ ΠΌΠΈΡ€ΠΎΠΌ ΡƒΠΆΠ΅ послС Π΅Π³ΠΎ смСрти. БущСствСнный Π²ΠΊΠ»Π°Π΄ Π² это внСсли русскиС ΡƒΡ‡Ρ‘Π½Ρ‹Π΅ Π›Π΅Π½Ρ† ΠΈ Π―ΠΊΠΎΠ±ΠΈ. Π’ 1842 Π³ΠΎΠ΄Ρƒ ЛондонскоС ΠšΠΎΡ€ΠΎΠ»Π΅Π²ΡΠΊΠΎΠ΅ общСство Π½Π°Π³Ρ€Π°Π΄ΠΈΠ»ΠΎ Ρ„ΠΈΠ·ΠΈΠΊΠ° Π·ΠΎΠ»ΠΎΡ‚ΠΎΠΉ мСдалью, Π° Π·Π°ΠΊΠΎΠ½, ΠΎΡ‚ΠΊΡ€Ρ‹Ρ‚Ρ‹ΠΉ ΠΈΠΌ, Π±Ρ‹Π» Π½Π°Π·Π²Π°Π½ Π΅Π³ΠΎ ΠΈΠΌΠ΅Π½Π΅ΠΌ.

ΠŸΠΎΠ½ΡΡ‚ΠΈΠ΅ Ρ‚ΠΎΠΊΠ° ΠΈ напряТСния

Π—Π°ΠΊΠΎΠ½ΠΎΠΌΠ΅Ρ€Π½ΠΎΡΡ‚ΡŒ ΡƒΡ‡Ρ‘Π½ΠΎΠ³ΠΎ устанавливаСт Π·Π°Π²ΠΈΡΠΈΠΌΠΎΡΡ‚ΡŒ ΠΌΠ΅ΠΆΠ΄Ρƒ собой Ρ‚Ρ€Ρ‘Ρ… элСктричСских Π²Π΅Π»ΠΈΡ‡ΠΈΠ½: Ρ‚ΠΎΠΊΠ°, напряТСния ΠΈ сопротивлСния. ΠŸΠΎΡΡ‚ΠΎΠΌΡƒ для Ρ‚ΠΎΠ³ΠΎ Ρ‡Ρ‚ΠΎΠ±Ρ‹ Ρ€Π°Π·ΠΎΠ±Ρ€Π°Ρ‚ΡŒΡΡ Π² сути Π·Π°ΠΊΠΎΠ½Π° Ома для ΠΏΠΎΠ»Π½ΠΎΠΉ элСктричСской Ρ†Π΅ΠΏΠΈ, Π½Π΅ΠΎΠ±Ρ…ΠΎΠ΄ΠΈΠΌΠΎ ΠΏΠΎΠ½ΠΈΠΌΠ°Ρ‚ΡŒ, Ρ‡Ρ‚ΠΎ ΠΆΠ΅ ΠΈΠ· сСбя ΠΎΠ½ΠΈ ΠΏΡ€Π΅Π΄ΡΡ‚Π°Π²Π»ΡΡŽΡ‚.

Π’ любом Ρ‚Π΅Π»Π΅ ΡΡƒΡ‰Π΅ΡΡ‚Π²ΡƒΡŽΡ‚ свободныС элСмСнтарныС частички, ΠΎΠ±Π»Π°Π΄Π°ΡŽΡ‰ΠΈΠ΅ ΠΎΠΏΡ€Π΅Π΄Π΅Π»Ρ‘Π½Π½Ρ‹ΠΌ количСством энСргии β€” зарядом. Если Ρ‚Π΅Π»ΠΎ находится Π² спокойном состоянии, Ρ‚ΠΎ Π΅ΡΡ‚ΡŒ Π½Π° Π½Π΅Π³ΠΎ Π½Π΅ оказываСтся Π½ΠΈΠΊΠ°ΠΊΠΎΠ³ΠΎ воздСйствия, Ρ‚ΠΎ происходит ΠΈΡ… Ρ…Π°ΠΎΡ‚ΠΈΡ‡Π½ΠΎΠ΅ ΠΏΠ΅Ρ€Π΅ΠΌΠ΅Ρ‰Π΅Π½ΠΈΠ΅. Если ΠΆΠ΅ ΠΊ Ρ‚Π΅Π»Ρƒ ΠΏΡ€ΠΈΠ»ΠΎΠΆΠ΅Π½ΠΎ элСктричСскоС ΠΏΠΎΠ»Π΅, Ρ‚ΠΎ ΠΈΡ… ΠΏΠ΅Ρ€Π΅ΠΌΠ΅Ρ‰Π΅Π½ΠΈΠ΅ становится упорядочСнным, ΠΈ ΠΎΠ½ΠΈ Π½Π°Ρ‡ΠΈΠ½Π°ΡŽΡ‚ ΠΏΠ΅Ρ€Π΅Π΄Π²ΠΈΠ³Π°Ρ‚ΡŒΡΡ Π² ΠΎΠ΄Π½Ρƒ сторону.

Π’Π°ΠΊΠΎΠ΅ Π½Π°ΠΏΡ€Π°Π²Π»Π΅Π½Π½ΠΎΠ΅ Π΄Π²ΠΈΠΆΠ΅Π½ΠΈΠ΅ Π½Π°Π·Ρ‹Π²Π°ΡŽΡ‚ элСктричСским Ρ‚ΠΎΠΊΠΎΠΌ. ΠœΠ΅Ρ€ΠΎΠΉ Π΅Π³ΠΎ слуТит сила Ρ‚ΠΎΠΊΠ°, скалярная Π²Π΅Π»ΠΈΡ‡ΠΈΠ½Π°, опрСдСляСмая ΠΎΡ‚Π½ΠΎΡˆΠ΅Π½ΠΈΠ΅ΠΌ количСства зарядов ΠΏΡ€ΠΎΡˆΠ΅Π΄ΡˆΠΈΡ… Ρ‡Π΅Ρ€Π΅Π· ΠΏΠΎΠΏΠ΅Ρ€Π΅Ρ‡Π½ΠΎΠ΅ сСчСниС ΠΏΡ€ΠΎΠ²ΠΎΠ΄Π½ΠΈΠΊΠ° Π·Π° Π΅Π΄ΠΈΠ½ΠΈΡ†Ρƒ Π²Ρ€Π΅ΠΌΠ΅Π½ΠΈ: I = dq/dt. Π—Π° Π΅Π΄ΠΈΠ½ΠΈΡ†Ρƒ измСрСния силы Ρ‚ΠΎΠΊΠ° принят Π°ΠΌΠΏΠ΅Ρ€.

Если Π½Π°ΠΏΡ€Π°Π²Π»Π΅Π½ΠΈΠ΅ пСрСмСщСния зарядов остаётся Π½Π΅ΠΈΠ·ΠΌΠ΅Π½Π½Ρ‹ΠΌ, Ρ‚ΠΎ Π΄Π²ΠΈΠΆΠ΅Π½ΠΈΠ΅ Ρ‚ΠΎΠΊΠ° считаСтся постоянным, Π° Ссли измСняСтся β€” ΠΏΠ΅Ρ€Π΅ΠΌΠ΅Π½Π½Ρ‹ΠΌ. Π’ΠΎΠ·Π½ΠΈΠΊΠ½ΠΎΠ²Π΅Π½ΠΈΠ΅ Ρ‚ΠΎΠΊΠ° Π²ΠΎΠ·ΠΌΠΎΠΆΠ½ΠΎ Ρ‚ΠΎΠ»ΡŒΠΊΠΎ Π² Π·Π°ΠΌΠΊΠ½ΡƒΡ‚ΠΎΠΉ Ρ†Π΅ΠΏΠΈ. Для Ρ‚ΠΎΠ³ΠΎ Ρ‡Ρ‚ΠΎΠ±Ρ‹ заряд пСрСмСстился, ΠΏΡ€ΠΈΠ»ΠΎΠΆΠ΅Π½Π½ΠΎΠ΅ ΠΏΠΎΠ»Π΅ Π΄ΠΎΠ»ΠΆΠ½ΠΎ Π²Ρ‹ΠΏΠΎΠ»Π½ΠΈΡ‚ΡŒ Ρ€Π°Π±ΠΎΡ‚Ρƒ. Π’ΠΎ Π΅ΡΡ‚ΡŒ Π·Π°Ρ‚Ρ€Π°Ρ‚ΠΈΡ‚ΡŒ ΠΊΠ°ΠΊΡƒΡŽ-Ρ‚ΠΎ ΡΠ½Π΅Ρ€Π³ΠΈΡŽ для пСрСмСщСния заряда с ΠΎΠ΄Π½ΠΎΠΉ Ρ‚ΠΎΡ‡ΠΊΠΈ Π² Π΄Ρ€ΡƒΠ³ΡƒΡŽ. Если ΠΏΡ€ΠΈΠ½ΡΡ‚ΡŒ, Ρ‡Ρ‚ΠΎ Π² Π½Π°Ρ‡Π°Π»ΡŒΠ½ΠΎΠΌ ΠΏΠΎΠ»ΠΎΠΆΠ΅Π½ΠΈΠΈ частичка ΠΎΠ±Π»Π°Π΄Π°Π΅Ρ‚ Π½ΡƒΠ»Π΅Π²Ρ‹ΠΌ зарядом, Ρ‚ΠΎ Ρ‚ΠΎΠ³Π΄Π° ΠΏΠ΅Ρ€Π΅ΠΌΠ΅ΡΡ‚ΠΈΠ²ΡˆΠΈΡΡŒ, ΠΎΠ½Π° ΡƒΠΆΠ΅ Π±ΡƒΠ΄Π΅Ρ‚ ΠΈΠΌΠ΅Ρ‚ΡŒ Π΄Ρ€ΡƒΠ³ΠΎΠ΅ Π΅Π³ΠΎ Π·Π½Π°Ρ‡Π΅Π½ΠΈΠ΅. Π Π°Π·Π½ΠΎΡΡ‚ΡŒ ΠΌΠ΅ΠΆΠ΄Ρƒ этими Π²Π΅Π»ΠΈΡ‡ΠΈΠ½Π°ΠΌΠΈ называСтся Ρ€Π°Π·Π½ΠΎΡΡ‚ΡŒΡŽ ΠΏΠΎΡ‚Π΅Π½Ρ†ΠΈΠ°Π»ΠΎΠ² ΠΈΠ»ΠΈ напряТСниСм.

Для поддСрТания силы Ρ‚ΠΎΠΊΠ° Π² ΠΏΠΎΠ»Π½ΠΎΠΉ Ρ†Π΅ΠΏΠΈ Π½Π΅ΠΎΠ±Ρ…ΠΎΠ΄ΠΈΠΌ источник, постоянно Π²ΠΎΠ·Π΄Π΅ΠΉΡΡ‚Π²ΡƒΡŽΡ‰ΠΈΠΉ Π½Π° свободныС заряды ΠΈ ΠΏΠΎΠ΄Π΄Π΅Ρ€ΠΆΠΈΠ²Π°ΡŽΡ‰ΠΈΠΉ разности ΠΏΠΎΡ‚Π΅Π½Ρ†ΠΈΠ°Π»ΠΎΠ² Π½Π° Ρ€Π°Π·Π»ΠΈΡ‡Π½Ρ‹Ρ… участках Ρ†Π΅ΠΏΠΈ. Π’Π΅Π»ΠΈΡ‡ΠΈΠ½Π° силы, которая дСйствуСт Π½Π° Ρ†Π΅ΠΏΡŒ, называСтся Π­Π”Π‘. ЀизичСски ΠΎΠ½Π° прСдставляСт собой ΠΎΡ‚Π½ΠΎΡˆΠ΅Π½ΠΈΠ΅ Ρ€Π°Π±ΠΎΡ‚Ρ‹, Π·Π°Ρ‚Ρ€Π°Ρ‡ΠΈΠ²Π°Π΅ΠΌΠΎΠΉ Π½Π° ΠΏΠ΅Ρ€Π΅Π΄Π²ΠΈΠΆΠ΅Π½ΠΈΠ΅ заряда ΠΎΡ‚ ΠΎΠ΄Π½ΠΎΠ³ΠΎ своСго полюса ΠΊ Π΄Ρ€ΡƒΠ³ΠΎΠΌΡƒ, ΠΊ Π·Π½Π°Ρ‡Π΅Π½ΠΈΡŽ заряда: E = A/q. Π˜Π·ΠΌΠ΅Ρ€ΡΠ΅Ρ‚ΡΡ Π­Π”Π‘, Ρ‚Π°ΠΊ ΠΆΠ΅ ΠΊΠ°ΠΊ ΠΈ напряТСниС, Π² Π²ΠΎΠ»ΡŒΡ‚Π°Ρ….

ΠŸΡ€ΠΈ ΠΏΠ΅Ρ€Π΅ΠΌΠ΅Ρ‰Π΅Π½ΠΈΠΈ заряд ΠΈΠ·-Π·Π° особСнностСй строСния кристалличСской Ρ€Π΅ΡˆΡ‘Ρ‚ΠΊΠΈ вСщСства, ΠΎΠ½ сталкиваСтся с Ρ€Π°Π·Π»ΠΈΡ‡Π½Ρ‹ΠΌΠΈ Π΄Π΅Ρ„Π΅ΠΊΡ‚Π°ΠΌΠΈ ΠΈ примСсями. Π’ Ρ€Π΅Π·ΡƒΠ»ΡŒΡ‚Π°Ρ‚Π΅ этого происходит частичноС рассСиваниС Π΅Π³ΠΎ ΠΏΠΎΡ‚Π΅Π½Ρ†ΠΈΠ°Π»Π°, Π° ΡΠΊΠΎΡ€ΠΎΡΡ‚ΡŒ двиТСния замСдляСтся. ΠŸΠΎΡ‚Π΅Ρ€Ρ энСргии характСризуСтся элСктричСской Π²Π΅Π»ΠΈΡ‡ΠΈΠ½ΠΎΠΉ-сопротивлСниСм. Π”Ρ€ΡƒΠ³ΠΈΠΌΠΈ словами, сопротивлСниС β€” это Π²Π΅Π»ΠΈΡ‡ΠΈΠ½Π°, ΠΏΡ€Π΅ΠΏΡΡ‚ΡΡ‚Π²ΡƒΡŽΡ‰Π°Ρ ΠΏΡ€ΠΎΡ…ΠΎΠΆΠ΄Π΅Π½ΠΈΡŽ Ρ‚ΠΎΠΊΠ°.

ИмпСданс Ρ†Π΅ΠΏΠΈ

НСмСцкий Ρ„ΠΈΠ·ΠΈΠΊ, проводя экспСримСнты, смог ΠΎΠ±Π½Π°Ρ€ΡƒΠΆΠΈΡ‚ΡŒ Π·Π°Π²ΠΈΡΠΈΠΌΠΎΡΡ‚ΡŒ ΠΌΠ΅ΠΆΠ΄Ρƒ Ρ‚ΠΎΠΊΠΎΠΌ ΠΈ напряТСниСм. Π˜Ρ… связь ΠΎΠΏΡ€Π΅Π΄Π΅Π»ΡΠ»Π°ΡΡŒ Ρ‡Π΅Ρ€Π΅Π· ΠΏΠΎΡΡ‚ΠΎΡΠ½Π½ΡƒΡŽ Π²Π΅Π»ΠΈΡ‡ΠΈΠ½Ρƒ, которая послС Π±Ρ‹Π»Π° Π½Π°Π·Π²Π°Π½Π° сопротивлСниСм. Π’Π°ΠΊ, Ρ„ΠΎΡ€ΠΌΡƒΠ»Π° Π·Π°ΠΊΠΎΠ½Π° Ома для ΠΏΠΎΠ»Π½ΠΎΠΉ Ρ†Π΅ΠΏΠΈ

ΠΌΠΎΠΆΠ΅Ρ‚ Π±Ρ‹Ρ‚ΡŒ записана Π² Π²ΠΈΠ΄Π΅ выраТСния:

I = E/Z, Π³Π΄Π΅:

  • I β€” сила Ρ‚ΠΎΠΊΠ° Ρ†Π΅ΠΏΠΈ;
  • E β€” элСктродвиТущая сила, прилоТСнная ΠΊ Ρ†Π΅ΠΏΠΈ;
  • Z β€” постоянная Π²Π΅Π»ΠΈΡ‡ΠΈΠ½Π° (ΠΏΠΎΠ»Π½ΠΎΠ΅ сопротивлСниС).

ПолноС сопротивлСниС (импСданс) элСктричСской Ρ†Π΅ΠΏΠΈ Π²Π°ΠΆΠ½Ρ‹ΠΉ ΠΏΠ°Ρ€Π°ΠΌΠ΅Ρ‚Ρ€, ΠΎΠΏΡ€Π΅Π΄Π΅Π»ΡΡŽΡ‰ΠΈΠΉ силу Ρ‚ΠΎΠΊΠ° ΠΈ ΠΏΠΎΠ»Π΅Π·Π½ΡƒΡŽ ΠΌΠΎΡ‰Π½ΠΎΡΡ‚ΡŒ. Бостоит ΠΎΠ½Π° ΠΈΠ· Π½Π΅ΡΠΊΠΎΠ»ΡŒΠΊΠΈΡ… ΡΠΎΡΡ‚Π°Π²Π»ΡΡŽΡ‰ΠΈΡ…: Π²Π½ΡƒΡ‚Ρ€Π΅Π½Π½Π΅Π³ΠΎ сопротивлСния источника Ρ‚ΠΎΠΊΠ° ΠΈ сопротивлСния элСмСнтов, ΠΈΠ· ΠΊΠΎΡ‚ΠΎΡ€Ρ‹Ρ… состоит схСма.

ΠŸΠΎΡΡ‚ΠΎΠΌΡƒ Π² ΠΎΡ‚Π»ΠΈΡ‡ΠΈΠ΅ ΠΎΡ‚ участка Ρ†Π΅ΠΏΠΈ, Π³Π΄Π΅ бСрётся Π²ΠΎ Π²Π½ΠΈΠΌΠ°Π½ΠΈΠ΅ Ρ‚ΠΎΠ»ΡŒΠΊΠΎ сопротивлСниС ΠΏΡ€ΠΎΠ²ΠΎΠ΄Π½ΠΈΠΊΠΎΠ², Π·Π°ΠΊΠΎΠ½ для всСй Ρ†Π΅ΠΏΠΈ ΡƒΡ‡ΠΈΡ‚Ρ‹Π²Π°Π΅Ρ‚ ΠΈ элСктричСскоС сопротивлСниС источника Ρ‚ΠΎΠΊΠ°. Π’ Ρ‚ΠΎ ΠΆΠ΅ врСмя Ρ…Π°Ρ€Π°ΠΊΡ‚Π΅Ρ€ происхоТдСния сопротивлСния ΠΌΠΎΠΆΠ΅Ρ‚ Π½ΠΎΡΠΈΡ‚ΡŒ ΠΊΠ°ΠΊ Π°ΠΊΡ‚ΠΈΠ²Π½ΡƒΡŽ ΡΠΎΡΡ‚Π°Π²Π»ΡΡŽΡ‰ΡƒΡŽ, Ρ‚Π°ΠΊ ΠΈ Ρ€Π΅Π°ΠΊΡ‚ΠΈΠ²Π½ΡƒΡŽ, ΡƒΡ‡ΠΈΡ‚Ρ‹Π²Π°ΡŽΡ‰ΡƒΡŽΡΡ для ΠΏΠ΅Ρ€Π΅ΠΌΠ΅Π½Π½ΠΎΠ³ΠΎ Ρ‚ΠΎΠΊΠ°.

Активная ΡΠΎΡΡ‚Π°Π²Π»ΡΡŽΡ‰Π°Ρ

Π’Π°ΠΊΠΎΠ΅ сопротивлСниС называСтся Π°ΠΊΡ‚ΠΈΠ²Π½Ρ‹ΠΌ, Ρ‚Π°ΠΊ ΠΊΠ°ΠΊ ΠΎΠ½ΠΎ Π·Π°Π±ΠΈΡ€Π°Π΅Ρ‚ Π½Π° сСбя Ρ‡Π°ΡΡ‚ΡŒ мощности, ΠΏΠΎΡΡ‚ΡƒΠΏΠ°ΡŽΡ‰Π΅ΠΉ ΠΎΡ‚ источника питания. Π­Ρ‚Π° забираСмая энСргия, проходя Ρ‡Π΅Ρ€Π΅Π· ΠΏΡ€ΠΎΠ²ΠΎΠ΄Π½ΠΈΠΊ, прСвращаСтся Π² Ρ‚Π΅ΠΏΠ»ΠΎ. ΠŸΡ€ΠΈ этом ΠΌΠΎΠΆΠ½ΠΎ ΠΎΠ±Π½Π°Ρ€ΡƒΠΆΠΈΡ‚ΡŒ, Ρ‡Ρ‚ΠΎ Ссли ΠΏΡ€ΠΎΠ²ΠΎΠ΄Π½ΠΈΠΊ ΠΏΠΎΠ΄ΠΊΠ»ΡŽΡ‡ΠΈΡ‚ΡŒ ΠΊ ΠΏΠ΅Ρ€Π΅ΠΌΠ΅Π½Π½ΠΎΠΌΡƒ источнику сигнала, Ρ‚ΠΎ Π΅Π³ΠΎ сопротивлСниС Π±ΡƒΠ΄Π΅Ρ‚ Π½Π΅ΠΌΠ½ΠΎΠ³ΠΎ большС. Бвязано это с Ρ‚Π΅ΠΌ, Ρ‡Ρ‚ΠΎ индуцируСмая Π­Π”Π‘ Π² ΠΌΠ°Ρ‚Π΅Ρ€ΠΈΠ°Π»Π΅ Π² любой Π΅Π³ΠΎ Ρ‚ΠΎΡ‡ΠΊΠ΅ нСодинаковая. Π‘Π»ΠΈΠΆΠ΅ ΠΊ Ρ†Π΅Π½Ρ‚Ρ€Ρƒ ΠΎΠ½Π° Π±ΡƒΠ΄Π΅Ρ‚ большС, Ρ‡Π΅ΠΌ Ρƒ повСрхности. Π’ΠΎ Π΅ΡΡ‚ΡŒ ΠΏΡ€ΠΈ ΠΏΠ΅Ρ€Π΅ΠΌΠ΅Π½Π½ΠΎΠΌ сигналС ΠΊΠ°ΠΊ Π±Ρ‹ происходит ΡƒΠΌΠ΅Π½ΡŒΡˆΠ΅Π½ΠΈΠ΅ ΠΏΠΎΠ»Π΅Π·Π½ΠΎΠ³ΠΎ сСчСния ΠΏΡ€ΠΎΠ²ΠΎΠ΄Π½ΠΈΠΊΠ°.

Π‘ΠΎΠΏΡ€ΠΎΡ‚ΠΈΠ²Π»Π΅Π½ΠΈΠ΅ зависит ΠΎΡ‚ физичСских ΠΏΠ°Ρ€Π°ΠΌΠ΅Ρ‚Ρ€ΠΎΠ² ΠΌΠ°Ρ‚Π΅Ρ€ΠΈΠ°Π»Π°. ΠœΠ°Ρ‚Π΅ΠΌΠ°Ρ‚ΠΈΡ‡Π΅ΡΠΊΠΈ это ΠΌΠΎΠΆΠ΅Ρ‚ Π±Ρ‹Ρ‚ΡŒ описано Π²Ρ‹Ρ€Π°ΠΆΠ΅Π½ΠΈΠ΅ΠΌ: R = p*L/S, Π³Π΄Π΅ L β€” Π΄Π»ΠΈΠ½Π° ΠΏΡ€ΠΎΠ²ΠΎΠ΄Π½ΠΈΠΊΠ°, S β€” ΠΏΠΎΠΏΠ΅Ρ€Π΅Ρ‡Π½ΠΎΠ΅ сСчСниС, p β€” ΡƒΠ΄Π΅Π»ΡŒΠ½ΠΎΠ΅ сопротивлСниС (Ρ‚Π°Π±Π»ΠΈΡ‡Π½ΠΎΠ΅ Π·Π½Π°Ρ‡Π΅Π½ΠΈΠ΅).

АктивноС сопротивлСниС слабо зависит ΠΎΡ‚ частоты сигнала, Π½ΠΎ ΠΏΡ€ΠΈ Π΅Π³ΠΎ ΡƒΠ²Π΅Π»ΠΈΡ‡Π΅Π½ΠΈΠΈ возрастаСт.

ΠžΡ‚Π»ΠΈΡ‡ΠΈΡ‚Π΅Π»ΡŒΠ½ΠΎΠΉ Ρ‡Π΅Ρ€Ρ‚ΠΎΠΉ элСмСнта, ΠΎΠ±Π»Π°Π΄Π°ΡŽΡ‰Π΅Π³ΠΎ Π°ΠΊΡ‚ΠΈΠ²Π½Ρ‹ΠΌ сопротивлСниСм, Π±ΡƒΠ΄Π΅Ρ‚ совпадСниС ΠΏΠΎ Ρ„Π°Π·Π΅, ΠΏΡ€ΠΎΡ‚Π΅ΠΊΠ°ΡŽΡ‰Π΅Π³ΠΎ Ρ‡Π΅Ρ€Π΅Π· Π½Π΅Π³ΠΎ Ρ‚ΠΎΠΊΠ° ΠΈ напряТСния. ΠŸΠΎΡΡ‚ΠΎΠΌΡƒ Π²Ρ‹Ρ‡ΠΈΡΠ»ΡΡ‚ΡŒΡΡ ΠΎΠ½ΠΎ ΠΏΠΎ Ρ„ΠΎΡ€ΠΌΡƒΠ»Π΅: R = U/I.

Π Π΅Π°ΠΊΡ‚ΠΈΠ²Π½ΠΎΠ΅ сопротивлСниС

Π˜Π½Π΄ΡƒΠΊΡ‚ΠΈΠ²Π½ΠΎΠ΅ сопротивлСниС связано с Π­Π”Π‘ самоиндукции. ΠŸΡ€ΠΈ ΠΏΡ€ΠΎΡ‚Π΅ΠΊΠ°Π½ΠΈΠΈ Ρ‡Π΅Ρ€Π΅Π· элСмСнт, ΠΎΠ±Π»Π°Π΄Π°ΡŽΡ‰ΠΈΠΉ ΠΈΠ½Π΄ΡƒΠΊΡ‚ΠΈΠ²Π½ΠΎΡΡ‚ΡŒΡŽ, ΠΏΠ΅Ρ€Π΅ΠΌΠ΅Π½Π½ΠΎΠ³ΠΎ Ρ‚ΠΎΠΊΠ°, Π²ΠΎΠ·Π½ΠΈΠΊΠ°Π΅Ρ‚ ΠΌΠ°Π³Π½ΠΈΡ‚Π½ΠΎΠ΅ ΠΏΠΎΠ»Π΅, ΡΠΎΠ·Π΄Π°ΡŽΡ‰Π΅Π΅ Π­Π”Π‘. Π­Ρ‚Π° сила противодСйствуСт Π²Π½Π΅ΡˆΠ½Π΅ΠΌΡƒ полю ΠΈ прСпятствуСт Π΅Π³ΠΎ Ρ€Π°ΡΠΏΡ€ΠΎΡΡ‚Ρ€Π°Π½Π΅Π½ΠΈΡŽ. ЗатрачиваСмая энСргия ΡƒΠ²Π΅Π»ΠΈΡ‡ΠΈΠ²Π°Π΅Ρ‚ ΠΌΠΎΡ‰Π½ΠΎΡΡ‚ΡŒ ΠΌΠ°Π³Π½ΠΈΡ‚Π½ΠΎΠ³ΠΎ поля. Как Ρ‚ΠΎΠ»ΡŒΠΊΠΎ Ρ‚ΠΎΠΊ ΡƒΠΌΠ΅Π½ΡŒΡˆΠ°Π΅Ρ‚ΡΡ, Π·Π½Π°Ρ‡Π΅Π½ΠΈΠ΅ ΠΌΠ°Π³Π½ΠΈΡ‚Π½ΠΎΠ³ΠΎ поля Π½Π°Ρ‡ΠΈΠ½Π°Π΅Ρ‚ Ρ‚ΠΎΠΆΠ΅ ΡΠ½ΠΈΠΆΠ°Ρ‚ΡŒΡΡ, индуцируя Ρ‚ΠΎΠΊ самоиндукции. Π•Π³ΠΎ Π½Π°ΠΏΡ€Π°Π²Π»Π΅Π½ΠΈΠ΅ совпадаСт с ΡƒΠ±Ρ‹Π²Π°ΡŽΡ‰ΠΈΠΌ Ρ‚ΠΎΠΊΠΎΠΌ. Π’ Ρ€Π΅Π·ΡƒΠ»ΡŒΡ‚Π°Ρ‚Π΅ энСргия, отобранная ΠΌΠ°Π³Π½ΠΈΡ‚Π½Ρ‹ΠΌ ΠΏΠΎΠ»Π΅ΠΌ, Π½Π°Ρ‡ΠΈΠ½Π°Π΅Ρ‚ ΠΎΡ‚Π΄Π°Π²Π°Ρ‚ΡŒΡΡ ΠΎΠ±Ρ€Π°Ρ‚Π½ΠΎ Π² Ρ†Π΅ΠΏΡŒ. Π’ΠΎ Π΅ΡΡ‚ΡŒ фактичСски, Π² ΠΎΡ‚Π»ΠΈΡ‡ΠΈΠ΅ ΠΎΡ‚ Π°ΠΊΡ‚ΠΈΠ²Π½ΠΎΠ³ΠΎ сопротивлСния, ΠΏΠΎΡ‚Π΅Ρ€ΡŒ энСргии Π½Π΅ Π²ΠΎΠ·Π½ΠΈΠΊΠ°Π΅Ρ‚.

Π’Π΅Π»ΠΈΡ‡ΠΈΠ½Π° ΠΈΠ½Π΄ΡƒΠΊΡ‚ΠΈΠ²Π½ΠΎΠ³ΠΎ сопротивлСния находится ΠΏΠΎ Ρ„ΠΎΡ€ΠΌΡƒΠ»Π΅ X L = 2 p * f * L, Π³Π΄Π΅: f β€” частота сигнала, L β€” Π·Π½Π°Ρ‡Π΅Π½ΠΈΠ΅ индуктивности. НапряТСниС, ΠΏΡ€ΠΈΠ»ΠΎΠΆΠ΅Π½Π½ΠΎΠ΅ ΠΊ индуктивности ΠΈ Ρ‚ΠΎΠΊ, ΠΏΠΎΡΡ‚ΡƒΠΏΠ°ΡŽΡ‰ΠΈΠΉ ΠΎΡ‚ источника энСргии, сдвинуты ΠΎΡ‚Π½ΠΎΡΠΈΡ‚Π΅Π»ΡŒΠ½ΠΎ Π΄Ρ€ΡƒΠ³ Π΄Ρ€ΡƒΠ³Π° ΠΏΠΎ Ρ„Π°Π·Π΅ Π½Π° 90, ΠΏΡ€ΠΈ этом Ρ‚ΠΎΠΊ отстаёт ΠΎΡ‚ напряТСния.

ЁмкостноС ΠΆΠ΅ сопротивлСниС обусловлСно Π²ΠΎΠ·Π½ΠΈΠΊΠ½ΠΎΠ²Π΅Π½ΠΈΠ΅ΠΌ элСктродвиТущСй силы. ΠŸΡ€ΠΈ ΠΏΡ€ΠΎΡ…ΠΎΠΆΠ΄Π΅Π½ΠΈΠΈ Ρ‡Π΅Ρ€Π΅Π· Ρ‘ΠΌΠΊΠΎΡΡ‚ΡŒ энСргия, ΠΏΠΎΡΡ‚ΡƒΠΏΠ°ΡŽΡ‰Π°Ρ ΠΎΡ‚ источника питания Π΄ΠΎΠ»ΠΆΠ½Π° ΠΏΡ€Π΅ΠΎΠ΄ΠΎΠ»Π΅Ρ‚ΡŒ ёмкостноС сопротивлСниС, затрачивая Ρ‡Π°ΡΡ‚ΡŒ мощности для Π΅Ρ‘ заряда. Но ΠΊΠ°ΠΊ Ρ‚ΠΎΠ»ΡŒΠΊΠΎ ΠΏΠΎΠ΄Π°Π²Π°Π΅ΠΌΡ‹ΠΉ сигнал ΠΈΠ·ΠΌΠ΅Π½ΠΈΡ‚ Π·Π½Π°ΠΊ, вСсь Π½Π°ΠΊΠΎΠΏΠ»Π΅Π½Π½Ρ‹ΠΉ заряд Ρ‘ΠΌΠΊΠΎΡΡ‚ΡŒΡŽ Π½Π°Ρ‡Π½Ρ‘Ρ‚ Π²ΠΎΠ·Π²Ρ€Π°Ρ‰Π°Ρ‚ΡŒΡΡ Π² Ρ†Π΅ΠΏΡŒ, увСличивая ΡΠ½Π΅Ρ€Π³ΠΈΡŽ элСктричСского поля.

Π”Ρ€ΡƒΠ³ΠΈΠΌΠΈ словами, Ρ‘ΠΌΠΊΠΎΡΡ‚ΡŒ становится источником Π­Π”Π‘. ЁмкостноС сопротивлСниС описываСтся Π²Ρ‹Ρ€Π°ΠΆΠ΅Π½ΠΈΠ΅ΠΌ: X c = 1/ (2 p * f * C), Π³Π΄Π΅: C β€” Π²Π΅Π»ΠΈΡ‡ΠΈΠ½Π° ёмкости. ΠŸΡ€ΠΈ Ρ‚Π°ΠΊΠΎΠΌ Ρ€ΠΎΠ΄Π΅ сопротивлСния Ρ‚ΠΎΠΊ Π±ΡƒΠ΄Π΅Ρ‚ ΠΎΠΏΠ΅Ρ€Π΅ΠΆΠ°Ρ‚ΡŒ напряТСниС ΠΏΠΎ Ρ„Π°Π·Π΅ Π½Π° 90

.

Π’Π°ΠΊΠΈΠΌ ΠΎΠ±Ρ€Π°Π·ΠΎΠΌ, Ρ€Π΅Π°ΠΊΡ‚ΠΈΠ²Π½ΠΎΠ΅ сопротивлСниС зависит ΠΎΡ‚ частоты сигнала. ΠžΠ±Ρ‰ΠΈΠΉ ΠΆΠ΅ импСданс опрСдСляСтся Π½Π΅ ΠΊΠ°ΠΊ сумма всСх сопротивлСний, Π° ΠΏΠΎ Ρ„ΠΎΡ€ΠΌΡƒΠ»Π΅ Z = (R2+ X l2+ X c2)Β½.

Π‘ΡƒΡ‚ΡŒ Π·Π°ΠΊΠΎΠ½Π°

ΠžΠ±Ρ‰Π΅ΠΏΡ€ΠΈΠ½ΡΡ‚Π°Ρ Ρ„ΠΎΡ€ΠΌΡƒΠ»ΠΈΡ€ΠΎΠ²ΠΊΠ° Π·Π°ΠΊΠΎΠ½Π° Ома гласит, Ρ‡Ρ‚ΠΎ сила Ρ‚ΠΎΠΊΠ° Π² ΠΏΠΎΠ»Π½ΠΎΠΉ Ρ†Π΅ΠΏΠΈ прямо ΠΏΡ€ΠΎΠΏΠΎΡ€Ρ†ΠΈΠΎΠ½Π°Π»ΡŒΠ½Π° элСктродвиТущСй силС источника, Π΄Π΅Π»Ρ‘Π½Π½ΠΎΠΉ Π½Π° ΠΎΠ±Ρ‰Π΅Π΅ сопротивлСниС всСх элСмСнтов Π·Π°ΠΌΠΊΠ½ΡƒΡ‚ΠΎΠΉ Ρ†Π΅ΠΏΠΈ. ΠšΠ»Π°ΡΡΠΈΡ‡Π΅ΡΠΊΠ°Ρ Ρ„ΠΎΡ€ΠΌΡƒΠ»Π° Π·Π°ΠΊΠΎΠ½Π° Ома для Ρ†Π΅ΠΏΠΈ постоянного Ρ‚ΠΎΠΊΠ° выглядит ΡΠ»Π΅Π΄ΡƒΡŽΡ‰ΠΈΠΌ ΠΎΠ±Ρ€Π°Π·ΠΎΠΌ:

I = E /(r+R), Π³Π΄Π΅:

  • R β€” сопротивлСниС внСшнСй части Ρ†Π΅ΠΏΠΈ, Ом;
  • r β€” Π²Π½ΡƒΡ‚Ρ€Π΅Π½Π½ΠΈΠΉ импСданс источника энСргии.

Π’ Π·Π°ΠΌΠΊΠ½ΡƒΡ‚ΠΎΠΉ схСмС Ρ‚ΠΎΠΊ Ρ‚Π΅Ρ‡Ρ‘Ρ‚ ΠΎΡ‚ источника энСргии, ΠΏΡ€ΠΎΡ‚Π΅ΠΊΠ°Π΅Ρ‚ Ρ‡Π΅Ρ€Π΅Π· Ρ€Π°Π·Π»ΠΈΡ‡Π½Ρ‹Π΅ элСмСнты, ΠΏΠΎΡΠ»Π΅Π΄ΠΎΠ²Π°Ρ‚Π΅Π»ΡŒΠ½ΠΎ ΠΈΠ»ΠΈ ΠΏΠ°Ρ€Π°Π»Π»Π΅Π»ΡŒΠ½ΠΎ ΠΏΠΎΠ΄ΠΊΠ»ΡŽΡ‡Ρ‘Π½Π½Ρ‹Π΅ ΠΊ Π½Π΅ΠΌΡƒ, ΠΈ возвращаСтся ΠΎΠ±Ρ€Π°Ρ‚Π½ΠΎ. Π˜Π·ΡƒΡ‡Π°Ρ ΠΎΡ‚ΠΊΡ€Ρ‹Ρ‚ΠΈΠ΅ Ома ΠΌΠΎΠΆΠ½ΠΎ ΡΡ„ΠΎΡ€ΠΌΡƒΠ»ΠΈΡ€ΠΎΠ²Π°Ρ‚ΡŒ основной физичСский ΠΏΡ€ΠΈΠ½Ρ†ΠΈΠΏ, Π½Π° ΠΊΠΎΡ‚ΠΎΡ€ΠΎΠΌ строится элСктротСхника. Π—Π°ΠΊΠ»ΡŽΡ‡Π°Π΅Ρ‚ΡΡ ΠΎΠ½ Π² Ρ‚ΠΎΠΌ, Ρ‡Ρ‚ΠΎ Ρ‡Π΅ΠΌ большС Π­Π”Π‘, Ρ‚Π΅ΠΌ большСй энСргиСй Π±ΡƒΠ΄ΡƒΡ‚ ΠΎΠ±Π»Π°Π΄Π°Ρ‚ΡŒ носитСли заряда, Π° Π·Π½Π°Ρ‡ΠΈΡ‚ ΠΈ ΠΈΡ… ΡΠΊΠΎΡ€ΠΎΡΡ‚ΡŒ пСрСмСщСния Π±ΡƒΠ΄Π΅Ρ‚ большС.

ΠŸΡ€ΠΈ ΡƒΠ²Π΅Π»ΠΈΡ‡Π΅Π½ΠΈΠΈ сопротивлСния Π² Ρ†Π΅ΠΏΠΈ ΡΠΊΠΎΡ€ΠΎΡΡ‚ΡŒ двиТСния, Π° Π·Π½Π°Ρ‡ΠΈΡ‚, ΠΈ энСргия носитСлСй заряда ΡƒΠΌΠ΅Π½ΡŒΡˆΠ°Π΅Ρ‚ΡΡ, соотвСтствСнно сниТаСтся ΠΈ Ρ‚ΠΎΠΊ.

Π’Π΅Π»ΠΈΡ‡ΠΈΠ½Π° Π­Π”Π‘ зависит ΠΎΡ‚ характСристик источника энСргии, Π° сопротивлСниС ΠΎΡ‚ физичСских ΠΏΠ°Ρ€Π°ΠΌΠ΅Ρ‚Ρ€ΠΎΠ² ΠΌΠ°Ρ‚Π΅Ρ€ΠΈΠ°Π»Π° ΠΈ Ρ‚Π΅ΠΌΠΏΠ΅Ρ€Π°Ρ‚ΡƒΡ€Ρ‹. Π—Π½Π°Ρ‡Π΅Π½ΠΈΠ΅ Π°ΠΊΡ‚ΠΈΠ²Π½ΠΎΠ³ΠΎ сопротивлСния Π½Π΅ ΠΌΠΎΠΆΠ΅Ρ‚ Π±Ρ‹Ρ‚ΡŒ ΠΈΠ·ΠΌΠ΅Π½Π΅Π½ΠΎ ΡƒΠ²Π΅Π»ΠΈΡ‡Π΅Π½ΠΈΠ΅ΠΌ Π½Π°ΠΏΡ€Π°Π²Π»Π΅Π½Π½ΠΎΠ³ΠΎ двиТСния частиц ΠΈΠ»ΠΈ напряТСния, Π½ΠΎ ΠΏΡ€ΠΈ этом рСактивная ΡΠΎΡΡ‚Π°Π²Π»ΡΡŽΡ‰Π°Ρ зависит ΠΎΡ‚ частоты сигнала.

ΠŸΠΎΡΡ‚ΠΎΠΌΡƒ Π·Π°ΠΊΠΎΠ½ Ома для ΠΏΠΎΠ»Π½ΠΎΠ³ΠΎ участка Ρ†Π΅ΠΏΠΈ ΠΏΠ΅Ρ€Π΅ΠΌΠ΅Π½Π½ΠΎΠ³ΠΎ Ρ‚ΠΎΠΊΠ° ΠΈ ΡƒΡ‡ΠΈΡ‚Ρ‹Π²Π°Π΅Ρ‚ ΠΈΠ½Π΄ΡƒΠΊΡ‚ΠΈΠ²Π½ΡƒΡŽ ΠΈ Ρ‘ΠΌΠΊΠΎΡΡ‚Π½ΡƒΡŽ ΡΠΎΡΡ‚Π°Π²Π»ΡΡŽΡ‰ΡƒΡŽ, ΠΏΡ€ΠΈΡ‡Ρ‘ΠΌ ΠΊΠ°ΠΊ источника питания, Ρ‚Π°ΠΊ ΠΈ самой Ρ†Π΅ΠΏΠΈ. ΠžΠΏΠΈΡΡ‹Π²Π°Π΅Ρ‚ΡΡ матСматичСски ΠΎΠ½ Ρ„ΠΎΡ€ΠΌΡƒΠ»ΠΎΠΉ: I = Um /Z, Π³Π΄Π΅:

  • Um β€” Π­Π”Π‘ источника питания;
  • Z β€” импСданс всСй Π·Π°ΠΌΠΊΠ½ΡƒΡ‚ΠΎΠΉ Ρ†Π΅ΠΏΠΈ: Z = (R2+(wL β€” 1/wC)2)Β½.

Π’ΠΎ Π΅ΡΡ‚ΡŒ для ΠΏΠ΅Ρ€Π΅ΠΌΠ΅Π½Π½ΠΎΠ³ΠΎ Ρ‚ΠΎΠΊΠ° Π·Π°ΠΊΠΎΠ½ Π±ΡƒΠ΄Π΅Ρ‚ ΠΎΠΏΠΈΡΡ‹Π²Π°Ρ‚ΡŒΡΡ Π²Ρ‹Ρ€Π°ΠΆΠ΅Π½ΠΈΠ΅ΠΌ Π²ΠΈΠ΄Π°:

I = Um/ (R2+(wL -1/wC)2)Β½.

Однако слСдуСт ΠΏΠΎΠ½ΠΈΠΌΠ°Ρ‚ΡŒ, Ρ‡Ρ‚ΠΎ Π² Ρ„ΠΎΡ€ΠΌΡƒΠ»Π΅ ΠΈΡΠΏΠΎΠ»ΡŒΠ·ΡƒΡŽΡ‚ΡΡ Π°ΠΌΠΏΠ»ΠΈΡ‚ΡƒΠ΄Π½Ρ‹Π΅ значСния Π²Π΅Π»ΠΈΡ‡ΠΈΠ½, Π° Π½Π΅ ΠΌΠ³Π½ΠΎΠ²Π΅Π½Π½Ρ‹Π΅.

Π”ΠΈΡ„Ρ„Π΅Ρ€Π΅Π½Ρ†ΠΈΠ°Π»ΡŒΠ½ΠΎΠ΅ ΡƒΡ€Π°Π²Π½Π΅Π½ΠΈΠ΅

Π’Π°ΠΊ ΠΊΠ°ΠΊ сопротивлСниС зависит Π½Π΅ Ρ‚ΠΎΠ»ΡŒΠΊΠΎ ΠΎΡ‚ физичСских свойств ΠΌΠ°Ρ‚Π΅Ρ€ΠΈΠ°Π»Π°, Π½ΠΎ ΠΈ ΠΎΡ‚ Π΅Π³ΠΎ гСомСтричСских ΠΏΠ°Ρ€Π°ΠΌΠ΅Ρ‚Ρ€ΠΎΠ², часто послСднСС ΠΏΡ€ΠΈ использовании Π·Π°ΠΊΠΎΠ½Π° Ома ΠΈΡΠΊΠ»ΡŽΡ‡Π°Π΅Ρ‚ΡΡ ΠΈΠ· Ρ„ΠΎΡ€ΠΌΡƒΠ»Ρ‹. ΠžΡ‚ΠΊΡ€Ρ‹Ρ‚ΠΈΠ΅ ΡƒΡ‡Ρ‘Π½ΠΎΠ³ΠΎ, ΡƒΡ‡ΠΈΡ‚Ρ‹Π²Π°ΡŽΡ‰Π΅Π΅ Ρ‚ΠΎΠ»ΡŒΠΊΠΎ элСктропроводящиС свойства, Π·Π°ΠΏΠΈΡΡ‹Π²Π°ΡŽΡ‚ Π² Ρ‚Π°ΠΊ Π½Π°Π·Ρ‹Π²Π°Π΅ΠΌΠΎΠΉ Π΄ΠΈΡ„Ρ„Π΅Ρ€Π΅Π½Ρ†ΠΈΠ°Π»ΡŒΠ½ΠΎΠΉ Ρ„ΠΎΡ€ΠΌΠ΅.

Вакая Ρ„ΠΎΡ€ΠΌΡƒΠ»Π° ΠΈΠΌΠ΅Π΅Ρ‚ Π²ΠΈΠ΄: J = Οƒ*E, Π³Π΄Π΅:

  • J β€” ΠΏΠ»ΠΎΡ‚Π½ΠΎΡΡ‚ΡŒ, Ρ…Π°Ρ€Π°ΠΊΡ‚Π΅Ρ€ΠΈΠ·ΡƒΡŽΡ‰Π°Ρ силу элСктричСства ΠΏΡ€ΠΎΡ‚Π΅ΠΊΠ°ΡŽΡ‰Π΅Π³ΠΎ Ρ‡Π΅Ρ€Π΅Π· Π΅Π΄ΠΈΠ½ΠΈΡ†Ρƒ ΠΏΠ»ΠΎΡ‰Π°Π΄ΠΈ;
  • Οƒ β€” ΡƒΠ΄Π΅Π»ΡŒΠ½Π°Ρ ΠΏΡ€ΠΎΠ²ΠΎΠ΄ΠΈΠΌΠΎΡΡ‚ΡŒ, Π²Π΅Π»ΠΈΡ‡ΠΈΠ½Π° обратная ΡƒΠ΄Π΅Π»ΡŒΠ½ΠΎΠΌΡƒ ΡΠΎΠΏΡ€ΠΎΡ‚ΠΈΠ²Π»Π΅Π½ΠΈΡŽ;
  • E β€” Π½Π°ΠΏΡ€ΡΠΆΡ‘Π½Π½ΠΎΡΡ‚ΡŒ поля, опрСдСляСтся Π² ΠΎΠΏΡ€Π΅Π΄Π΅Π»Ρ‘Π½Π½ΠΎΠΉ Ρ‚ΠΎΡ‡ΠΊΠ΅ ΠΊΠ°ΠΊ ΠΎΡ‚Π½ΠΎΡˆΠ΅Π½ΠΈΠ΅ силы Π΄Π΅ΠΉΡΡ‚Π²ΡƒΡŽΡ‰Π΅ΠΉ Π½Π° Π½Π΅ΠΏΠΎΠ΄Π²ΠΈΠΆΠ½Ρ‹ΠΉ заряд ΠΊ Π΅Π³ΠΎ Π²Π΅Π»ΠΈΡ‡ΠΈΠ½Π΅.

Π‘ΠΎΡΡ‚Π°Π²Π»ΡΡŽΡ‰ΠΈΠ΅ уравнСния ΠΏΡ€Π΅Π΄ΡΡ‚Π°Π²Π»ΡΡŽΡ‚ΡΡ Π² Π²ΠΈΠ΄Π΅ Ρ„ΡƒΠ½ΠΊΡ†ΠΈΠΈ ΠΊΠΎΠΎΡ€Π΄ΠΈΠ½Π°Ρ‚ ΠΈ Π²Ρ€Π΅ΠΌΠ΅Π½ΠΈ. УдСльная ΠΏΡ€ΠΎΠ²ΠΎΠ΄ΠΈΠΌΠΎΡΡ‚ΡŒ выраТаСтся Π² Π²ΠΈΠ΄Π΅ Π΅Π΄ΠΈΠ½ΠΈΡ‡Π½ΠΎΠΉ ΠΌΠ°Ρ‚Ρ€ΠΈΡ†Ρ‹. ΠŸΠΎΡΡ‚ΠΎΠΌΡƒ Π·Π°ΠΊΠΎΠ½ ΠΌΠΎΠΆΠ½ΠΎ ΠΏΡ€Π΅Π΄ΡΡ‚Π°Π²ΠΈΡ‚ΡŒ Ρ„ΠΎΡ€ΠΌΡƒΠ»ΠΎΠΉ:

Π’Π°ΠΊΠΈΠΌ ΠΎΠ±Ρ€Π°Π·ΠΎΠΌ, Π·Π°ΠΊΠΎΠ½ Ома для Π·Π°ΠΌΠΊΠ½ΡƒΡ‚ΠΎΠΉ Ρ†Π΅ΠΏΠΈ практичСски Π½ΠΈΡ‡Π΅ΠΌ Π½Π΅ отличаСтся ΠΎΡ‚ Π΅Π³ΠΎ Ρ„ΠΎΡ€ΠΌΡƒΠ»ΠΈΡ€ΠΎΠ²ΠΊΠΈ для Π½Π΅ΠΏΠΎΠ»Π½ΠΎΠΉ схСмы, лишь Ρ‚ΠΎΠ»ΡŒΠΊΠΎ Π΄ΠΎΠΏΠΎΠ»Π½ΠΈΡ‚Π΅Π»ΡŒΠ½ΠΎ ΡƒΡ‡ΠΈΡ‚Ρ‹Π²Π°Π΅Ρ‚ Π²Π½ΡƒΡ‚Ρ€Π΅Π½Π½Π΅Π΅ сопротивлСниС источника Π­Π”Π‘. ΠŸΡ€ΠΈ этом Π΅Π³ΠΎ Ρ„ΠΎΡ€ΠΌΡƒΠ»ΠΈΡ€ΠΎΠ²ΠΊΠ° Π½Π΅ измСняСтся.

Π—Π°ΠΊΠΎΠ½ Ома ΠΊΡ€Π°Ρ‚ΠΊΠΎ ΠΈ понятно для Ρ‡Π°ΠΉΠ½ΠΈΠΊΠΎΠ²

Π—Π°ΠΊΠΎΠ½ Ома являСтся ΠΎΠ΄Π½ΠΈΠΌ ΠΈΠ· Ρ„ΡƒΠ½Π΄Π°ΠΌΠ΅Π½Ρ‚Π°Π»ΡŒΠ½Ρ‹Ρ… Π·Π°ΠΊΠΎΠ½ΠΎΠ² элСктродинамики, ΠΊΠΎΡ‚ΠΎΡ€Ρ‹ΠΉ опрСдСляСт взаимосвязь ΠΌΠ΅ΠΆΠ΄Ρƒ напряТСниСм, сопротивлСниСм ΠΈ силой Ρ‚ΠΎΠΊΠ°. Π•Π³ΠΎ Π²Π°ΠΆΠ½ΠΎ Π·Π½Π°Ρ‚ΡŒ ΠΈ ΠΏΠΎΠ½ΠΈΠΌΠ°Ρ‚ΡŒ. ΠŸΠΎΠ½ΡΡ‚Π½ΠΎΠ΅ объяснСниС Π²Ρ‹ Π½Π°ΠΉΠ΄Ρ‘Ρ‚Π΅ Π² ΡΡ‚Π°Ρ‚ΡŒΠ΅.

Π—Π°ΠΊΠΎΠ½ Ома ΠΎΡ„ΠΈΡ†ΠΈΠ°Π»ΡŒΠ½ΠΎ ΠΈ Π°Π±ΡΠΎΠ»ΡŽΡ‚Π½ΠΎ ΠΎΠΏΡ€Π°Π²Π΄Π°Π½ΠΎ ΠΌΠΎΠΆΠ½ΠΎ отнСсти ΠΊ ряду ΠΎΡΠ½ΠΎΠ²ΠΎΠΏΠΎΠ»Π°Π³Π°ΡŽΡ‰ΠΈΡ… Π² Ρ„ΠΈΠ·ΠΈΠΊΠ΅ ΠΏΠΎ нСскольким ΠΏΡ€ΠΈΠ·Π½Π°ΠΊΠ°ΠΌ. Π”Π°Π½Π½Ρ‹ΠΉ Π·Π°ΠΊΠΎΠ½ ΠΎΠ±ΡŠΡΡΠ½ΡΡŽΡ‚ Π² школС Π½Π° Π±Π°Π·ΠΎΠ²ΠΎΠΌ ΡƒΡ€ΠΎΠ²Π½Π΅, Π° послС, Π±ΠΎΠ»Π΅Π΅ ΡƒΠ³Π»ΡƒΠ±Π»Π΅Π½Π½ΠΎ, Π² учрСТдСниях, ΡΠΏΠ΅Ρ†ΠΈΠ°Π»ΠΈΠ·ΠΈΡ€ΡƒΡŽΡ‰ΠΈΡ…ΡΡ Π½Π° ΠΈΠ·ΡƒΡ‡Π΅Π½ΠΈΠΈ тСхничСских аспСктов Ρ‚Π΅Ρ…Π½ΠΎΠ»ΠΎΠ³ΠΈΠΉ.

Π—Π°ΠΊΠΎΠ½ Ома – ΠΎΠΏΡ€Π΅Π΄Π΅Π»Π΅Π½ΠΈΠ΅

Π’ΠΏΠ΅Ρ€Π²Ρ‹Π΅ Π΄Π°Π½Π½Ρ‹ΠΉ Π·Π°ΠΊΠΎΠ½ Π±Ρ‹Π» ΠΎΡ„ΠΈΡ†ΠΈΠ°Π»ΡŒΠ½ΠΎ зафиксирован ΠΈ сформулирован Π² восСмнадцатом Π²Π΅ΠΊΠ΅, благодаря сдСланному сСйчас ΡƒΠΆΠ΅ ΡˆΠΈΡ€ΠΎΠΊΠΎ извСстным всСм Π“Π΅ΠΎΡ€Π³ΠΎΠΌ Π‘ΠΈΠΌΠΎΠ½ΠΎΠΌ Омом ΠΎΡ‚ΠΊΡ€Ρ‹Ρ‚ΠΈΡŽ. Благодаря Π΄Π°Π½Π½ΠΎΠΌΡƒ Π·Π°ΠΊΠΎΠ½Ρƒ ΠΏΠΎΠ»ΡƒΡ‡ΠΈΠ»ΠΎ Π³Ρ€Π°ΠΌΠΎΡ‚Π½ΠΎΠ΅ ΠΈ ΠΈΡΡ‡Π΅Ρ€ΠΏΡ‹Π²Π°ΡŽΡ‰Π΅Π΅ объяснСниС Π½Π°Π»ΠΈΡ‡ΠΈΠ΅ количСствСнной связи ΠΌΠ΅ΠΆΠ΄Ρƒ трСмя Ρ„ΠΈΠ³ΡƒΡ€ΠΈΡ€ΡƒΡŽΡ‰ΠΈΠΌΠΈ Π² ΠΎΠΏΡ€Π΅Π΄Π΅Π»Π΅Π½ΠΈΠΈ ΠΏΠ°Ρ€Π°ΠΌΠ΅Ρ‚Ρ€Π°ΠΌΠΈ. Π—Π°Π²ΠΈΡΠΈΠΌΠΎΡΡ‚ΡŒ рассматриваСтся ΠΊΠ°ΠΊ ΠΏΡ€ΠΎΠΏΠΎΡ€Ρ†ΠΈΠΎΠ½Π°Π»ΡŒΠ½Π°Ρ. Когда Π΄Π°Π½Π½ΠΎΠ΅ явлСниС Ρ‚ΠΎΠ»ΡŒΠΊΠΎ Π±Ρ‹Π»ΠΎ выявлСно, Π·Π°ΠΊΠΎΠ½ нСсколько Ρ€Π°Π· Ρ„ΠΎΡ€ΠΌΡƒΠ»ΠΈΡ€ΠΎΠ²Π°Π»ΠΈ. Π’ ΠΈΡ‚ΠΎΠ³Π΅ сСйчас всСм извСстно Π΄Π°Π½Π½ΠΎΠ΅ ΠΎΠΏΡ€Π΅Π΄Π΅Π»Π΅Π½ΠΈΠ΅: Β«Π²Π΅Π»ΠΈΡ‡ΠΈΠ½Π° Ρ‚ΠΎΠΊΠ° Π½Π° участкС Ρ†Π΅ΠΏΠΈ прямо ΠΏΡ€ΠΎΠΏΠΎΡ€Ρ†ΠΈΠΎΠ½Π°Π»ΡŒΠ½Π° Π½Π°ΠΏΡ€ΡΠΆΠ΅Π½ΠΈΡŽ, ΠΏΡ€ΠΈΠ»ΠΎΠΆΠ΅Π½Π½ΠΎΠΌΡƒ ΠΊ этому участку, ΠΈ ΠΎΠ±Ρ€Π°Ρ‚Π½ΠΎ ΠΏΡ€ΠΎΠΏΠΎΡ€Ρ†ΠΈΠΎΠ½Π°Π»ΡŒΠ½Π° Π΅Π³ΠΎ ΡΠΎΠΏΡ€ΠΎΡ‚ΠΈΠ²Π»Π΅Π½ΠΈΡŽΒ».

Для Π»ΡƒΡ‡ΡˆΠ΅Π³ΠΎ понимания Ρ€Π°Π·Π΄Π΅Π»ΠΈΠΌ ΠΎΠΏΡ€Π΅Π΄Π΅Π»Π΅Π½ΠΈΠ΅ Π½Π° Π΄Π²Π΅ части ΠΈ Ρ€Π°Π·Π±Π΅Ρ€Ρ‘ΠΌ ΠΎΡ‚Π΄Π΅Π»ΡŒΠ½ΠΎ Π±ΠΎΠ»Π΅Π΅ понятным языком смысл ΠΊΠ°ΠΆΠ΄ΠΎΠΉ.

  1. ΠŸΠ΅Ρ€Π²Π°Ρ Ρ‡Π°ΡΡ‚ΡŒ опрСдСлСния ΡƒΠΊΠ°Π·Ρ‹Π²Π°Π΅Ρ‚ Π½Π° Ρ‚ΠΎ, Ρ‡Ρ‚ΠΎ Ссли Π½Π° ΠΎΠΏΡ€Π΅Π΄Π΅Π»Π΅Π½Π½ΠΎΠΉ ΠΎΡ‚Ρ€Π΅Π·ΠΊΠ΅ Ρ†Π΅ΠΏΠΈ происходит количСствСнный скачок напряТСния, Ρ‚ΠΎ Π²Π΅Π»ΠΈΡ‡ΠΈΠ½Π° Ρ‚ΠΎΠΊΠ° Ρ‚Π°ΠΊΠΆΠ΅ увСличиваСтся Π½Π° Π΄Π°Π½Π½ΠΎΠΌ участкС. Π’Π°ΠΆΠ½ΠΎ ΡƒΠΏΠΎΠΌΡΠ½ΡƒΡ‚ΡŒ, Ρ‡Ρ‚ΠΎ становится большС ΠΈ Π²Π΅Π»ΠΈΡ‡ΠΈΠ½Π° Ρ‚ΠΎΠΊΠ° Π½Π° Π·Π°Π΄Π°Π½Π½ΠΎΠΌ участкС Ρ†Π΅ΠΏΠΈ.
  2. ΠšΠΎΠ½Ρ†ΠΎΠ²ΠΊΠ° опрСдСлСния Ρ€Π°ΡΡˆΠΈΡ„Ρ€ΠΎΠ²Ρ‹Π²Π°Π΅Ρ‚ΡΡ Ρ‚Π°ΠΊΠΆΠ΅ просто. Π’Ρ‹ΡˆΠ΅ напряТСниС – мСньшС сила Ρ‚ΠΎΠΊΠ°.

Π—Π°ΠΊΠΎΠ½ Ома – Ρ„ΠΎΡ€ΠΌΡƒΠ»Π°

Π˜Π»Π»ΡŽΡΡ‚Ρ€Π°Ρ†ΠΈΡ связи сопротивлСния

Рисунок наглядно дСмонстрируСт связь Ρ„ΠΈΠ³ΡƒΡ€ΠΈΡ€ΡƒΡŽΡ‰ΠΈΡ… Π² понятии «участников». Π’Π°ΠΊΠΈΠΌ ΠΎΠ±Ρ€Π°Π·ΠΎΠΌ, Π²Ρ‹Ρ‚Π΅ΠΊΠ°ΡŽΡ‚ простыС Π²Ρ‹Π²ΠΎΠ΄Ρ‹:

1. ΠŸΡ€ΠΈ Π΄Π°Π½Π½Ρ‹Ρ… условиях: Π½Π° ΠΊΠΎΠ½ΠΊΡ€Π΅Ρ‚Π½ΠΎΠΌ ΠΎΡ‚Ρ€Π΅Π·ΠΊΠ΅ увСличиваСтся напряТСниС, Π½ΠΎ ΠΏΡ€ΠΈ Ρ‚ΠΎΠΌ сопротивлСниС остаётся ΠΏΡ€Π΅ΠΆΠ½ΠΈΠΌ, Ρ‚ΠΎΠΊ Ρ€Π΅Π·ΠΊΠΎ возрастаСт;

2. Иная ситуация: Π½Π°ΠΎΠ±ΠΎΡ€ΠΎΡ‚, измСняСтся сопротивлСниС, Π° Ρ‚ΠΎΡ‡Π½Π΅Π΅ возрастаСт, ΠΏΡ€ΠΈ Ρ‚ΠΎΠΌ Ρ‡Ρ‚ΠΎ ΡƒΡ€ΠΎΠ²Π΅Π½ΡŒ напряТСния Π½Π΅ мСняСтся вовсС, Ρ‚ΠΎΠΊΠ° становится мСньшС.

Π’ ΠΈΡ‚ΠΎΠ³Π΅ Π² Π·Π°ΠΊΠΎΠ½Π΅ Ома ΡƒΡ‡Π°ΡΡ‚Π²ΡƒΡŽΡ‚ всСго Ρ‚Ρ€ΠΈ Π²Π΅Π»ΠΈΡ‡ΠΈΠ½Ρ‹.

Готовая Ρ„ΠΎΡ€ΠΌΡƒΠ»Π° выглядит Ρ‚Π°ΠΊ:

I = U/R

Π€ΠΈΠ³ΡƒΡ€ΠΈΡ€ΡƒΡŽΡ‚ ΠΈ Π΄Ρ€ΡƒΠ³ΠΈΠ΅ Π΄Π²Π΅ ΠΏΠ΅Ρ€Π΅ΠΌΠ΅Π½Π½Ρ‹Π΅, ΠΈΡ… Ρ‚Π°ΠΊΠΆΠ΅ ΠΌΠΎΠΆΠ½ΠΎ Π²Ρ‹Ρ‡ΠΈΡΠ»ΠΈΡ‚ΡŒ, ΠΏΡ€ΠΈ условии, Ρ‡Ρ‚ΠΎ Π΄Ρ€ΡƒΠ³ΠΈΠ΅ Π΄Π²Π° значСния извСстны. Π’ΠΈΠ΄ΠΎΠΈΠ·ΠΌΠ΅Π½ΠΈΠ² Ρ„ΠΎΡ€ΠΌΡƒΠ»Ρƒ, ΠΏΠΎΠ»ΡƒΡ‡ΠΈΠΌ:

Π€ΠΎΡ€ΠΌΡƒΠ»Π° сопротивлСнияR = U/I
Π€ΠΎΡ€ΠΌΡƒΠ»Π° напряТСнияU = I Γ— R
Π€ΠΎΡ€ΠΌΡƒΠ»Π° силы Ρ‚ΠΎΠΊΠ°I = U/R

Π’Π°ΠΆΠ½ΠΎ!

Π¨ΠΏΠ°Ρ€Π³Π°Π»ΠΊΠ° для Π·Π°ΠΊΠΎΠ½Π° Ома

На Π½Π°Ρ‡Π°Π»ΡŒΠ½ΠΎΠΌ этапС, ΠΊΠΎΠ³Π΄Π° ΡΠΎΡΡ‚Π°Π²Π»ΡΡ‚ΡŒ Ρ„ΠΎΡ€ΠΌΡƒΠ»Ρ‹ Π΅Ρ‰Ρ‘ слоТно, ΠΌΠΎΠΆΠ½ΠΎ Π²ΠΎΡΠΏΠΎΠ»ΡŒΠ·ΠΎΠ²Π°Ρ‚ΡŒΡΡ нСбольшой ΡˆΠΏΠ°Ρ€Π³Π°Π»ΠΊΠΎΠΉ.

На Ρ‚Ρ€Π΅ΡƒΠ³ΠΎΠ»ΡŒΠ½ΠΈΠΊΠ΅ просто Π½ΡƒΠΆΠ½ΠΎ Π·Π°ΠΊΡ€Ρ‹Ρ‚ΡŒ Ρ‚ΠΎ Π·Π½Π°Ρ‡Π΅Π½ΠΈΠ΅, ΠΊΠΎΡ‚ΠΎΡ€ΠΎΠ΅ Π½Π΅ΠΎΠ±Ρ…ΠΎΠ΄ΠΈΠΌΠΎ Π½Π°ΠΉΡ‚ΠΈ.

Π—Π°ΠΊΠΎΠ½ Ома для участка Ρ†Π΅ΠΏΠΈ

Π˜Ρ‚ΠΎΠ³ΠΎΠ²Π°Ρ Ρ„ΠΎΡ€ΠΌΡƒΠ»Π° Π½Π΅ видоизмСняСтся вовсС. ΠžΠ±Ρ‹Ρ‡Π½ΠΎ сопротивлСниС Π² Π΄Π°Π½Π½ΠΎΠΌ Π·Π°ΠΊΠΎΠ½Π΅ являСтся явной характСристикой ΠΏΡ€ΠΎΠ²ΠΎΠ΄Π½ΠΈΠΊΠ°, ΠΏΠΎΡ‚ΠΎΠΌΡƒ Ρ‡Ρ‚ΠΎ это Π·Π½Π°Ρ‡Π΅Π½ΠΈΠ΅ Π½Π΅ постоянная Π²Π΅Π»ΠΈΡ‡ΠΈΠ½Π°: Π² зависимости ΠΎΡ‚ ΠΌΠ°Ρ‚Π΅Ρ€ΠΈΠ°Π»Π° ΠΈ Π΄Ρ€ΡƒΠ³ΠΈΡ… ΠΏΠ°Ρ€Π°ΠΌΠ΅Ρ‚Ρ€ΠΎΠ² число ΠΌΠΎΠΆΠ΅Ρ‚ ΡƒΠ²Π΅Π»ΠΈΡ‡ΠΈΠ²Π°Ρ‚ΡŒΡΡ ΠΈΠ»ΠΈ ΡƒΠΌΠ΅Π½ΡŒΡˆΠ°Ρ‚ΡŒΡΡ. Π—Π°ΠΊΠΎΠ½ ΠΏΡ€ΠΈΠΌΠ΅Π½ΠΈΠΌ ΠΊΠ°ΠΊ ΠΏΡ€ΠΈ расчётС с использованиСм ΠΌΠ΅Ρ‚Π°Π»Π»ΠΎΠ², Ρ‚Π°ΠΊ ΠΈ растворов элСктролитов, ΠΎΠ΄Π½Π°ΠΊΠΎ сущСствуСт Π²Π°ΠΆΠ½Ρ‹ΠΉ нюанс: Π² Ρ†Π΅ΠΏΠΈ Π½Π΅ Π΄ΠΎΠ»ΠΆΠ½ΠΎ Π±Ρ‹Ρ‚ΡŒ Ρ€Π΅Π°Π»ΡŒΠ½ΠΎΠ³ΠΎ источника Ρ‚ΠΎΠΊΠ°, ΠΈΠ»ΠΈ ΠΆΠ΅ источник Π΄ΠΎΠ»ΠΆΠ΅Π½ Π±Ρ‹Ρ‚ΡŒ ΠΈΠ΄Π΅Π°Π»ΡŒΠ½Ρ‹ΠΌ, Ρ‚ΠΎ Π΅ΡΡ‚ΡŒ ΠΎΠ½ Π½Π΅ Π΄ΠΎΠ»ΠΆΠ΅Π½ ΡΠΎΠ·Π΄Π°Π²Π°Ρ‚ΡŒ Π΄ΠΎΠΏΠΎΠ»Π½ΠΈΡ‚Π΅Π»ΡŒΠ½ΠΎΠ΅ сопротивлСниС.

Π¨ΠΏΠ°Ρ€Π³Π°Π»ΠΊΠ° для использования Π·Π°ΠΊΠΎΠ½Π° Ома

Π‘ Π­Π”Π‘

ΠžΠ±ΠΎΠ±Ρ‰Ρ‘Π½Π½Ρ‹ΠΉ Π·Π°ΠΊΠΎΠ½ Ома формулируСтся Ρ‚Π°ΠΊ:

I = (Uab+E)/R

Π’Π°ΠΊΠΆΠ΅ Ρ„ΠΎΡ€ΠΌΡƒΠ»Ρƒ ΠΌΠΎΠΆΠ½ΠΎ Π²Ρ‹Ρ€Π°Π·ΠΈΡ‚ΡŒ Ρ‡Π΅Ρ€Π΅Π· ΠΏΡ€ΠΎΠ²ΠΎΠ΄ΠΈΠΌΠΎΡΡ‚ΡŒ:

I = (Uab + E) Γ— G, ΠΊΠ°ΠΊ понятно, G – ΠΏΡ€ΠΎΠ²ΠΎΠ΄ΠΈΠΌΠΎΡΡ‚ΡŒ участка элСктричСской Ρ†Π΅ΠΏΠΈ. Π­Ρ‚ΠΈ Ρ„ΠΎΡ€ΠΌΡƒΠ»Ρ‹ ΠΌΠΎΠΆΠ½ΠΎ ΠΈΡΠΏΠΎΠ»ΡŒΠ·ΠΎΠ²Π°Ρ‚ΡŒ, Ссли ΡΠΎΡ…Ρ€Π°Π½ΡΡŽΡ‚ΡΡ условия, зафиксированныС Π½Π° рисункС.

Участок Ρ†Π΅ΠΏΠΈ с Π­Π”Π‘

Π‘Π΅Π· Π­Π”Π‘

Для Π½Π°Ρ‡Π°Π»Π° ΠΎΠΏΡ€Π΅Π΄Π΅Π»ΠΈΠΌ, Ρ‡Ρ‚ΠΎ ΠΏΠΎΠ»ΠΎΠΆΠΈΡ‚Π΅Π»ΡŒΠ½ΠΎΠ΅ Π½Π°ΠΏΡ€Π°Π²Π»Π΅Π½ΠΈΠ΅ – это Ρ‚ΠΎ, Ρ‡Ρ‚ΠΎ слСва Π½Π°ΠΏΡ€Π°Π²ΠΎ. Волько Π² этом случаС напряТСниС Π½Π° участкС Π±ΡƒΠ΄Π΅Ρ‚ Ρ€Π°Π²Π½ΡΡ‚ΡŒΡΡ разности ΠΏΠΎΡ‚Π΅Π½Ρ†ΠΈΠ°Π»ΠΎΠ².

Π Π°Π·Π½ΠΎΡΡ‚ΡŒ ΠΏΠΎΡ‚Π΅Π½Ρ†ΠΈΠ°Π»ΠΎΠ²

Если сохраняСтся условиС ΠΈ ΠΏΠΎΡ‚Π΅Π½Ρ†ΠΈΠ°Π» ΠΊΠΎΠ½Π΅Ρ‡Π½Ρ‹ΠΉ мСньшС ΠΏΠΎΡ‚Π΅Π½Ρ†ΠΈΠ°Π»Π° Π½Π°Ρ‡Π°Π»ΡŒΠ½ΠΎΠ³ΠΎ, Ρ‚ΠΎ напряТСниС Π±ΡƒΠ΄Π΅Ρ‚ большС нуля. Π—Π½Π°Ρ‡ΠΈΡ‚, ΠΊΠ°ΠΊ ΠΈ полагаСтся, Π½Π°ΠΏΡ€Π°Π²Π»Π΅Π½ΠΈΠ΅ Π»ΠΈΠ½ΠΈΠΉ напряТСнности Π² ΠΏΡ€ΠΎΠ²ΠΎΠ΄Π½ΠΈΠΊΠ΅ Π±ΡƒΠ΄Π΅Ρ‚ ΠΎΡ‚ Π½Π°Ρ‡Π°Π»Π° ΠΊ ΠΊΠΎΠ½Ρ†Ρƒ, ΡΠ»Π΅Π΄ΠΎΠ²Π°Ρ‚Π΅Π»ΡŒΠ½ΠΎ, Π½Π°ΠΏΡ€Π°Π²Π»Π΅Π½ΠΈΠ΅ Ρ‚ΠΎΠΊΠ° Π±ΡƒΠ΄Π΅Ρ‚ ΠΈΠ΄Π΅Π½Ρ‚ΠΈΡ‡Π½Ρ‹ΠΌ. ИмСнно Ρ‚Π°ΠΊΠΎΠ΅ Π½Π°ΠΏΡ€Π°Π²Π»Π΅Π½ΠΈΠ΅ Ρ‚ΠΎΠΊΠ° принято ΡΡ‡ΠΈΡ‚Π°Ρ‚ΡŒ ΠΏΠΎΠ»ΠΎΠΆΠΈΡ‚Π΅Π»ΡŒΠ½Ρ‹ΠΌ, I > O. Π”Π°Π½Π½Ρ‹ΠΉ Π²Π°Ρ€ΠΈΠ°Π½Ρ‚ самый простой для расчётов. Π€ΠΎΡ€ΠΌΡƒΠ»Π° Π΄Π΅ΠΉΡΡ‚Π²ΠΈΡ‚Π΅Π»ΡŒΠ½Π° с Π»ΡŽΠ±Ρ‹ΠΌΠΈ числами.

Π—Π°ΠΊΠΎΠ½ Ома для ΠΏΠΎΠ»Π½ΠΎΠΉ (Π·Π°ΠΌΠΊΠ½ΡƒΡ‚ΠΎΠΉ) Ρ†Π΅ΠΏΠΈ

ΠŸΡ€ΠΈ Π΄Π°Π½Π½ΠΎΠΉ Π²Π°Ρ€ΠΈΠ°Ρ†ΠΈΠΈ Π·Π°ΠΊΠΎΠ½Π° выявляСтся Π·Π½Π°Ρ‡Π΅Π½ΠΈΠ΅ Ρ‚ΠΎΠΊΠ° ΠΏΡ€ΠΈ Ρ€Π΅Π°Π»ΡŒΠ½Ρ‹Ρ… условиях, Ρ‚ΠΎ Π΅ΡΡ‚ΡŒ Π² настоящСй ΠΏΠΎΠ»Π½ΠΎΠΉ Ρ†Π΅ΠΏΠΈ. Π’Π°ΠΆΠ½ΠΎ ΡƒΡ‡ΠΈΡ‚Ρ‹Π²Π°Ρ‚ΡŒ Ρ‚ΠΎ, Ρ‡Ρ‚ΠΎ ΠΏΠΎΠ»ΡƒΡ‡ΠΈΠ²ΡˆΠ΅Π΅ΡΡ Π² Ρ€Π΅Π·ΡƒΠ»ΡŒΡ‚Π°Ρ‚Π΅ расчСтов число зависит ΠΎΡ‚ Π½Π΅ΡΠΊΠΎΠ»ΡŒΠΊΠΈΡ… ΠΏΠ°Ρ€Π°ΠΌΠ΅Ρ‚Ρ€ΠΎΠ², Π° Π½Π΅ Ρ‚ΠΎΠ»ΡŒΠΊΠΎ ΠΎΡ‚ сопротивлСния Π½Π°Π³Ρ€ΡƒΠ·ΠΊΠΈ.

Π‘ΠΎΠΏΡ€ΠΎΡ‚ΠΈΠ²Π»Π΅Π½ΠΈΠ΅ Π½Π°Π³Ρ€ΡƒΠ·ΠΊΠΈ – внСшнСС сопротивлСниС, Π° сопротивлСниС самого источника Ρ‚ΠΎΠΊΠ° – Π²Π½ΡƒΡ‚Ρ€Π΅Π½Π½Π΅Π΅ сопротивлСниС (обозначаСтся малСнькой r).

Π’Ρ‹Π²ΠΎΠ΄ Ρ„ΠΎΡ€ΠΌΡƒΠ»Ρ‹ Π·Π°ΠΊΠΎΠ½Π° Ома для Π·Π°ΠΌΠΊΠ½ΡƒΡ‚ΠΎΠΉ Ρ†Π΅ΠΏΠΈ

Если ΠΊ Ρ†Π΅ΠΏΠΈ ΠΏΠΎΠ΄ΠΊΠ»ΡŽΡ‡Π΅Π½ΠΎ напряТСниС ΠΈ Π² Ρ†Π΅ΠΏΠΈ Π·Π°ΠΌΠ΅Ρ‡Π΅Π½ΠΎ напряТСниС (Ρ‚ΠΎΠΊ), Ρ‚ΠΎ, Ρ‡Ρ‚ΠΎΠ±Ρ‹ ΠΏΠΎΠ΄Π΄Π΅Ρ€ΠΆΠ°Ρ‚ΡŒ Π΅Π³ΠΎ Π²ΠΎ внСшнСй Ρ†Π΅ΠΏΠΈ, Π½Π΅ΠΎΠ±Ρ…ΠΎΠ΄ΠΈΠΌΠΎ ΡΠΎΠ·Π΄Π°Ρ‚ΡŒ условия, ΠΏΡ€ΠΈ ΠΊΠΎΡ‚ΠΎΡ€Ρ‹Ρ… ΠΌΠ΅ΠΆΠ΄Ρƒ Π΅Ρ‘ ΠΊΠΎΠ½Ρ†Π°ΠΌΠΈ Π²ΠΎΠ·Π½ΠΈΠΊΠ½Π΅Ρ‚ Ρ€Π°Π·Π½ΠΎΡΡ‚ΡŒ ΠΏΠΎΡ‚Π΅Π½Ρ†ΠΈΠ°Π»ΠΎΠ². Π­Ρ‚ΠΎ число Π±ΡƒΠ΄Π΅Ρ‚ Ρ€Π°Π²Π½ΡΡ‚ΡŒΡΡ I Γ— R. Однако Π²Π°ΠΆΠ½ΠΎ ΠΏΠΎΠΌΠ½ΠΈΡ‚ΡŒ ΠΎ Ρ‚ΠΎΠΌ, Ρ‡Ρ‚ΠΎ Π²Ρ‹ΡˆΠ΅ΡƒΠΏΠΎΠΌΡΠ½ΡƒΡ‚Ρ‹ΠΉ Ρ‚ΠΎΠΊ Π±ΡƒΠ΄Π΅Ρ‚ ΠΈ Π²ΠΎ Π²Π½ΡƒΡ‚Ρ€Π΅Π½Π½Π΅ΠΉ Ρ†Π΅ΠΏΠΈ ΠΈ Π΅Π³ΠΎ Ρ‚Π°ΠΊΠΆΠ΅ Π½Π΅ΠΎΠ±Ρ…ΠΎΠ΄ΠΈΠΌΠΎ ΠΏΠΎΠ΄Π΄Π΅Ρ€ΠΆΠΈΠ²Π°Ρ‚ΡŒ, поэтому Π½ΡƒΠΆΠ½ΠΎ ΡΠΎΠ·Π΄Π°Ρ‚ΡŒ Ρ€Π°Π·Π½ΠΎΡΡ‚ΡŒ ΠΏΠΎΡ‚Π΅Π½Ρ†ΠΈΠ°Π»ΠΎΠ² ΠΌΠ΅ΠΆΠ΄Ρƒ ΠΊΠΎΠ½Ρ†Π°ΠΌΠΈ сопротивлСния r. Π­Ρ‚Π° Ρ€Π°Π·Π½ΠΎΡΡ‚ΡŒ равняСтся I Γ— r.

Π§Ρ‚ΠΎΠ±Ρ‹ ΠΏΠΎΠ΄Π΄Π΅Ρ€ΠΆΠ°Ρ‚ΡŒ Ρ‚ΠΎΠΊ Π² Ρ†Π΅ΠΏΠΈ, элСктродвиТущая сила (Π­Π”Π‘) аккумулятора Π΄ΠΎΠ»ΠΆΠ½Π° ΠΈΠΌΠ΅Ρ‚ΡŒ Π²Π΅Π»ΠΈΡ‡ΠΈΠ½Ρƒ:

E = I Γ— r + I Γ— R

Π­Ρ‚Π° Ρ„ΠΎΡ€ΠΌΡƒΠ»Π° ΠΏΠΎΠΊΠ°Π·Ρ‹Π²Π°Π΅Ρ‚, Ρ‡Ρ‚ΠΎ элСктродвиТущая сила Π² Ρ†Π΅ΠΏΠΈ Ρ€Π°Π²Π½Π° суммС внСшнСго ΠΈ Π²Π½ΡƒΡ‚Ρ€Π΅Π½Π½Π΅Π³ΠΎ ΠΏΠ°Π΄Π΅Π½ΠΈΠΉ напряТСния. Вынося I Π·Π° скобки, ΠΏΠΎΠ»ΡƒΡ‡ΠΈΠΌ:

E = I(r + R)

ΠΈΠ»ΠΈ

I = E / (r + R)

Π”Π²Π΅ послСдниС Ρ„ΠΎΡ€ΠΌΡƒΠ»Ρ‹ Π²Ρ‹Ρ€Π°ΠΆΠ°ΡŽΡ‚ Π·Π°ΠΊΠΎΠ½Π° Ома для ΠΏΠΎΠ»Π½ΠΎΠΉ Ρ†Π΅ΠΏΠΈ.

Π—Π°ΠΊΠΎΠ½ Ома Π² Π΄ΠΈΡ„Ρ„Π΅Ρ€Π΅Π½Ρ†ΠΈΠ°Π»ΡŒΠ½ΠΎΠΉ Ρ„ΠΎΡ€ΠΌΠ΅

Π”ΠΈΡ„Ρ„Π΅Ρ€Π΅Π½Ρ†ΠΈΠ°Π»ΡŒΠ½Π°Ρ Ρ„ΠΎΡ€ΠΌΠ° Π·Π°ΠΊΠΎΠ½Π° Ома

Π—Π°ΠΊΠΎΠ½ ΠΌΠΎΠΆΠ½ΠΎ ΠΏΡ€Π΅Π΄ΡΡ‚Π°Π²ΠΈΡ‚ΡŒ Ρ‚Π°ΠΊΠΈΠΌ ΠΎΠ±Ρ€Π°Π·ΠΎΠΌ, Ρ‡Ρ‚ΠΎΠ±Ρ‹ ΠΎΠ½ Π½Π΅ Π±Ρ‹Π» привязан ΠΊ Ρ€Π°Π·ΠΌΠ΅Ρ€Π°ΠΌ ΠΏΡ€ΠΎΠ²ΠΎΠ΄Π½ΠΈΠΊΠ°. Для этого Π²Ρ‹Π΄Π΅Π»ΠΈΠΌ участок ΠΏΡ€ΠΎΠ²ΠΎΠ΄Π½ΠΈΠΊΠ° Ξ”l, Π½Π° ΠΊΠΎΠ½Ρ†Π°Ρ… ΠΊΠΎΡ‚ΠΎΡ€ΠΎΠΉ располоТСны Ρ„1 ΠΈ Ρ„2. Π‘Ρ€Π΅Π΄Π½ΡŽΡŽ ΠΏΠ»ΠΎΡ‰Π°Π΄ΡŒ ΠΏΡ€ΠΎΠ²ΠΎΠ΄Π½ΠΈΠΊΠ° ΠΎΠ±ΠΎΠ·Π½Π°Ρ‡Π°ΡŽΡ‚ Ξ”S , Π° ΠΏΠ»ΠΎΡ‚Π½ΠΎΡΡ‚ΡŒ Ρ‚ΠΎΠΊΠ° j, ΠΏΡ€ΠΈ Ρ‚Π°ΠΊΠΈΡ… условиях сила Ρ‚ΠΎΠΊΠ° Π±ΡƒΠ΄Π΅Ρ‚ Ρ€Π°Π²Π½ΡΡ‚ΡŒΡΡ:

I = jΞ”S = (Ρ„1- Ρ„2) / R = -(((Ρ„1 β€” Ρ„2)Ξ”S) / pΞ”l , ΠΎΡ‚ΡΡŽΠ΄Π° слСдуСт, Ρ‡Ρ‚ΠΎ j = -y Γ— (Ξ”Ρ„/Ξ”l)

ΠŸΡ€ΠΈ условии, Ρ‡Ρ‚ΠΎ Ξ”l Π±ΡƒΠ΄Π΅Ρ‚ Ρ€Π°Π²Π΅Π½ 0, Ρ‚ΠΎ, взяв ΠΏΡ€Π΅Π΄Π΅Π» ΠΎΡ‚Π½ΠΎΡˆΠ΅Π½ΠΈΡ:

lim (-(Ξ”Ρ„/Ξ”l)) = -(dΡ„/dl) = Π•,

ΠΎΠΊΠΎΠ½Ρ‡Π°Ρ‚Π΅Π»ΡŒΠ½ΠΎΠ΅ Π²Ρ‹Ρ€Π°ΠΆΠ΅Π½ΠΈΠ΅ Π±ΡƒΠ΄Π΅Ρ‚ Π²Ρ‹Π³Π»ΡΠ΄Π΅Ρ‚ΡŒ Ρ‚Π°ΠΊ:

j = yE

Π”Π°Π½Π½ΠΎΠ΅ Π²Ρ‹Ρ€Π°ΠΆΠ΅Π½ΠΈΠ΅ Π·Π°ΠΊΠΎΠ½Π° Π½Π°Ρ…ΠΎΠ΄ΠΈΡ‚ силу Ρ‚ΠΎΠΊΠ° Π² ΠΏΡ€ΠΎΠΈΠ·Π²ΠΎΠ»ΡŒΠ½ΠΎΠΉ Ρ‚ΠΎΡ‡ΠΊΠ΅ ΠΏΡ€ΠΎΠ²ΠΎΠ΄Π½ΠΈΠΊΠ° Π² зависимости ΠΎΡ‚ Π΅Π³ΠΎ свойств ΠΈ элСктричСского состояния.

Π—Π°ΠΊΠΎΠ½ Ома Π² ΠΈΠ½Ρ‚Π΅Π³Ρ€Π°Π»ΡŒΠ½ΠΎΠΉ Ρ„ΠΎΡ€ΠΌΠ΅

Π’ Π΄Π°Π½Π½ΠΎΠΉ ΠΈΠ½Ρ‚Π΅Ρ€ΠΏΡ€Π΅Ρ‚Π°Ρ†ΠΈΠΈ Π·Π°ΠΊΠΎΠ½Π° Π½Π΅ содСрТится Π² условиях Π­Π”Π‘, Ρ‚ΠΎ Π΅ΡΡ‚ΡŒ Ρ„ΠΎΡ€ΠΌΡƒΠ»Π° выглядит Ρ‚Π°ΠΊ:

I = U/R

Π§Ρ‚ΠΎΠ±Ρ‹ Π½Π°ΠΉΡ‚ΠΈ Π·Π½Π°Ρ‡Π΅Π½ΠΈΠ΅ для ΠΎΠ΄Π½ΠΎΡ€ΠΎΠ΄Π½ΠΎΠ³ΠΎ Π»ΠΈΠ½Π΅ΠΉΠ½ΠΎΠ³ΠΎ ΠΏΡ€ΠΎΠ²ΠΎΠ΄Π½ΠΈΠΊΠ°, Π²Ρ‹Ρ€Π°Π·ΠΈΠΌ R Ρ‡Π΅Ρ€Π΅Π· p ΠΈ ΠΏΠΎΠ»ΡƒΡ‡ΠΈΠΌ:

R = p (l/S), Π³Π΄Π΅ Π·Π° Ρ€ ΠΏΡ€ΠΈΠ½ΠΈΠΌΠ°Π΅ΠΌ ΡƒΠ΄Π΅Π»ΡŒΠ½ΠΎΠ΅ ΠΎΠ±ΡŠΡ‘ΠΌΠ½ΠΎΠ΅ сопротивлСниС.

Π›ΠΈΠ½ΠΈΠ΅ΠΉ Ρ‚ΠΎΠΊΠ° принято Π½Π°Π·Ρ‹Π²Π°Ρ‚ΡŒ ΠΊΡ€ΠΈΠ²ΡƒΡŽ, Π² ΠΊΠ°ΠΆΠ΄ΠΎΠΉ Ρ‚ΠΎΡ‡ΠΊΠ΅ ΠΊΠΎΡ‚ΠΎΡ€ΠΎΠΉ Π²Π΅ΠΊΡ‚ΠΎΡ€ плотности Ρ‚ΠΎΠΊΠ° Π½Π°ΠΏΡ€Π°Π²Π»Π΅Π½ ΠΏΠΎ ΠΊΠ°ΡΠ°Ρ‚Π΅Π»ΡŒΠ½ΠΎΠΉ ΠΊ этой ΠΊΡ€ΠΈΠ²ΠΎΠΉ. ΠŸΡ€ΠΈ Ρ‚Π°ΠΊΠΈΡ… условиях Π²Π΅ΠΊΡ‚ΠΎΡ€ плотности находится ΠΈΠ· ΠΎΡ‚Π½ΠΎΡˆΠ΅Π½ΠΈΡ J = jt, Π³Π΄Π΅ t – это Π΅Π΄ΠΈΠ½ΠΈΡ‡Π½Ρ‹ΠΉ Π²Π΅ΠΊΡ‚ΠΎΡ€ ΠΊΠ°ΡΠ°Ρ‚Π΅Π»ΡŒΠ½ΠΎΠΉ ΠΊ Π»ΠΈΠ½ΠΈΠΈ Ρ‚ΠΎΠΊΠ°.

Для Π»ΡƒΡ‡ΡˆΠ΅Π³ΠΎ понимания ΠΏΡ€Π΅Π΄ΠΏΠΎΠ»ΠΎΠΆΠΈΠΌ, Ρ‡Ρ‚ΠΎ ΡƒΠ΄Π΅Π»ΡŒΠ½ΠΎΠ΅ сопротивлСниС, Π° Ρ‚Π°ΠΊΠΆΠ΅ Π½Π°ΠΏΡ€ΡΠΆΠ΅Π½Π½ΠΎΡΡ‚ΡŒ поля Π΄Π²ΠΈΠΆΡƒΡ‰ΠΈΡ… сил Π½Π° ΠΏΠΎΠΏΠ΅Ρ€Π΅Ρ‡Π½ΠΎΠΌ сСчСнии ΠΏΡ€ΠΎΠ²ΠΎΠ΄Π½ΠΈΠΊΠ° ΠΎΠ΄Π½ΠΎΡ€ΠΎΠ΄Π½Ρ‹. ΠŸΡ€ΠΈ Ρ‚Π°ΠΊΠΎΠΌ условии Π• ΠΎΠ΄Π½ΠΎΡ€ΠΎΠ΄Π½Π°, Π° Π·Π½Π°Ρ‡ΠΈΡ‚, ΠΈ j Ρ‚Π°ΠΊΠΆΠ΅ однородная Π²Π΅Π»ΠΈΡ‡ΠΈΠ½Π°. ΠŸΡ€ΠΈΠΌΠ΅ΠΌ ΠΏΡ€ΠΎΠΈΠ·Π²ΠΎΠ»ΡŒΠ½ΠΎΠ΅ Π·Π½Π°Ρ‡Π΅Π½ΠΈΠ΅ ΠΏΠΎΠΏΠ΅Ρ€Π΅Ρ‡Π½ΠΎΠ³ΠΎ сСчСния Ρ†Π΅ΠΏΠΈ S, Ρ‚ΠΎΠ³Π΄Π° pl/s = E. ΠŸΠΎΠ»ΡƒΡ‡ΠΈΠ²ΡˆΠ΅Π΅ΡΡ равСнство ΡƒΠΌΠ½ΠΎΠΆΠΈΠΌ Π½Π° dl. Π’ΠΎΠ³Π΄Π° Edl = (Π• эл.ст.+Π• стор.) dl = Π• эл.ст. dl + Π• стор. dl = -dΡ„ + dE. ΠžΡ‚ΡΡŽΠ΄Π° ΠΏΠΎΠ»ΡƒΡ‡ΠΈΠΌ (pI/S) dl = -dΡ„ + dE. Π’ΠΎΠ·ΡŒΠΌΡ‘ΠΌ Π² ΡƒΡ‡Ρ‘Ρ‚, Ρ‡Ρ‚ΠΎ p/s dl = dR ΠΈ запишСм Π·Π°ΠΊΠΎΠ½ Ома Π² ΠΈΠ½Ρ‚Π΅Π³Ρ€Π°Π»ΡŒΠ½ΠΎΠΉ Ρ„ΠΎΡ€ΠΌΠ΅:

IdR = -dΡ„ + dE.

Π—Π°ΠΊΠΎΠ½ Ома Π² комплСксной Ρ„ΠΎΡ€ΠΌΠ΅

Π§Ρ‚ΠΎΠ±Ρ‹ провСсти Π°Π½Π°Π»ΠΈΠ· элСктричСских Ρ†Π΅ΠΏΠ΅ΠΉ ΡΠΈΠ½ΡƒΡΠΎΠΈΠ΄Π°Π»ΡŒΠ½ΠΎΠ³ΠΎ Ρ‚ΠΎΠΊΠ°, ΠΊΠΎΠΌΡ„ΠΎΡ€Ρ‚Π½Π΅Π΅ ΠΈΡΠΏΠΎΠ»ΡŒΠ·ΠΎΠ²Π°Ρ‚ΡŒ Π·Π°ΠΊΠΎΠ½ Ома Π² комплСксной Ρ„ΠΎΡ€ΠΌΠ΅. Для Π»ΡƒΡ‡ΡˆΠ΅Π³ΠΎ понимания Π²Π²Π΅Π΄Π΅ΠΌ основноС понятиС, Ρ„ΠΈΠ³ΡƒΡ€ΠΈΡ€ΡƒΡŽΡ‰Π΅Π΅ Π² Π΄Π°Π½Π½ΠΎΠΉ ΠΈΠ½Ρ‚Π΅Ρ€ΠΏΡ€Π΅Ρ‚Π°Ρ†ΠΈΠΈ Π·Π°ΠΊΠΎΠ½Π°: ΡΠΈΠ½ΡƒΡΠΎΠΈΠ΄Π°Π»ΡŒΠ½Ρ‹ΠΉ Ρ‚ΠΎΠΊ – это Π»ΠΈΠ½Π΅ΠΉΠ½Ρ‹Π΅ Ρ†Π΅ΠΏΠΈ с ΡƒΡΡ‚Π°Π½ΠΎΠ²ΠΈΠ²ΡˆΠΈΠΌΡΡ Ρ€Π΅ΠΆΠΈΠΌΠΎΠΌ Ρ€Π°Π±ΠΎΡ‚Ρ‹, послС Ρ‚ΠΎΠ³ΠΎ, ΠΊΠ°ΠΊ ΠΏΠ΅Ρ€Π΅Ρ…ΠΎΠ΄Π½Ρ‹Π΅ процСссы Π² Π½ΠΈΡ… Π·Π°Π²Π΅Ρ€ΡˆΠ΅Π½Ρ‹, ΡƒΡ€ΠΎΠ²Π΅Π½ΡŒ напряТСния Ρ€Π΅Π·ΠΊΠΎ ΡƒΠΌΠ΅Π½ΡŒΡˆΠ°Π΅Ρ‚ΡΡ Π½Π° ΠΊΠΎΠ½ΠΊΡ€Π΅Ρ‚Π½ΠΎΠΉ дистанции, Ρ‚ΠΎΠΊΠΈ Π² вСтвях ΠΈ Π­Π”Π‘ источников ΡΠ²Π»ΡΡŽΡ‚ΡΡ ΡΠΈΠ½ΡƒΡΠΎΠΈΠ΄Π°Π»ΡŒΠ½Ρ‹ΠΌΠΈ функциями Π²Ρ€Π΅ΠΌΠ΅Π½ΠΈ. Π’ ΠΏΡ€ΠΎΡ‚ΠΈΠ²Π½ΠΎΠΌ случаС, ΠΊΠΎΠ³Π΄Π° Π΄Π°Π½Π½Ρ‹Π΅ ΠΏΠ°Ρ€Π°ΠΌΠ΅Ρ‚Ρ€Ρ‹ Π½Π΅ ΡΠΎΠ±Π»ΡŽΠ΄Π°ΡŽΡ‚ΡΡ, Π·Π°ΠΊΠΎΠ½ Π½Π΅ ΠΌΠΎΠΆΠ΅Ρ‚ Π±Ρ‹Ρ‚ΡŒ ΠΏΡ€ΠΈΠΌΠ΅Π½ΠΈΠΌ. Π§Π΅ΠΌ отличаСтся эта Ρ„ΠΎΡ€ΠΌΠ° ΠΎΡ‚ ΠΎΠ±Ρ‹Ρ‡Π½ΠΎΠΉ? ΠžΡ‚Π²Π΅Ρ‚ прост: Ρ‚ΠΎΠΊΠΈ, сопротивлСниС ΠΈ Π­Π”Π‘ Ρ„ΠΈΠΊΡΠΈΡ€ΡƒΡŽΡ‚ΡΡ ΠΊΠ°ΠΊ комплСксныС числа. Π­Ρ‚ΠΎ обусловлСно Ρ‚Π΅ΠΌ, Ρ‡Ρ‚ΠΎ ΡΡƒΡ‰Π΅ΡΡ‚Π²ΡƒΡŽΡ‚ ΠΊΠ°ΠΊ Π°ΠΊΡ‚ΠΈΠ²Π½Ρ‹Π΅ Ρ‚Π°ΠΊ ΠΈ Ρ€Π΅Π°ΠΊΡ‚ΠΈΠ²Π½Ρ‹Π΅ значСния напряТСний, Ρ‚ΠΎΠΊΠΎΠ² ΠΈ сопротивлСний, Π° Π² Ρ€Π΅Π·ΡƒΠ»ΡŒΡ‚Π°Ρ‚Π΅ этого трСбуСтся внСсСниС ΠΎΠΏΡ€Π΅Π΄Π΅Π»Π΅Π½Π½Ρ‹Ρ… ΠΊΠΎΡ€Ρ€Π΅ΠΊΡ‚ΠΈΠ²ΠΎΠ².

ВмСсто Π°ΠΊΡ‚ΠΈΠ²Π½ΠΎΠ³ΠΎ сопротивлСния ΠΈΡΠΏΠΎΠ»ΡŒΠ·ΡƒΠ΅Ρ‚ΡΡ ΠΏΠΎΠ»Π½ΠΎΠ΅, Ρ‚ΠΎ Π΅ΡΡ‚ΡŒ комплСксноС сопротивлСниС Ρ†Π΅ΠΏΠΈ Z. ПадСниС напряТСния, Ρ‚ΠΎΠΊ ΠΈ Π­Π”Π‘ Ρ‚ΠΎΠΆΠ΅ ΠΏΡ€Π΅Π²Ρ€Π°Ρ‰Π°ΡŽΡ‚ΡΡ Π² комплСксныС Π²Π΅Π»ΠΈΡ‡ΠΈΠ½Ρ‹. ΠŸΡ€ΠΈ Ρ€Π΅Π°Π»ΡŒΠ½Ρ‹Ρ… расчСтах Π»ΡƒΡ‡ΡˆΠ΅ ΠΈ ΡƒΠ΄ΠΎΠ±Π½Π΅Π΅ ΠΏΡ€ΠΈΠΌΠ΅Π½ΡΡ‚ΡŒ Π΄Π΅ΠΉΡΡ‚Π²ΡƒΡŽΡ‰ΠΈΠ΅ значСния. Π˜Ρ‚Π°ΠΊ, Π·Π°ΠΊΠΎΠ½ Π² комплСксной Ρ„ΠΎΡ€ΠΌΠ΅ выглядит Ρ‚Π°ΠΊ:

i = U/Z, i = UY

Π’ Π΄Π°Π½Π½ΠΎΠΉ Ρ„ΠΎΡ€ΠΌΡƒΠ»Π΅ Z – комплСксноС сопротивлСниС, Y – комплСксная ΠΏΡ€ΠΎΠ²ΠΎΠ΄ΠΈΠΌΠΎΡΡ‚ΡŒ.

Π§Ρ‚ΠΎΠ±Ρ‹ Π²Ρ‹ΡΠ²ΠΈΡ‚ΡŒ эти Π²Π΅Π»ΠΈΡ‡ΠΈΠ½Ρ‹, Π²Ρ‹Π²Π΅Π΄Π΅Π½Ρ‹ Ρ„ΠΎΡ€ΠΌΡƒΠ»Ρ‹. ΠŸΡ€ΠΎΠΏΡƒΡΡ‚ΠΈΠΌ шаги ΠΈΡ… создания ΠΈ ΠΏΡ€ΠΈΠ²Π΅Π΄Π΅ΠΌ Π³ΠΎΡ‚ΠΎΠ²Ρ‹Π΅ Ρ„ΠΎΡ€ΠΌΡƒΠ»Ρ‹:

Z = ze = z cosΡ„ + jz sinΡ„ = r + jx

Y = 1/ ze = ye = y cos Ρ„ β€” jy sin Ρ„ = g + jb

Π—Π°ΠΊΠΎΠ½ Ома для ΠΏΠ΅Ρ€Π΅ΠΌΠ΅Π½Π½ΠΎΠ³ΠΎ Ρ‚ΠΎΠΊΠ°

ПослС Ρ‚ΠΎΠ³ΠΎ ΠΊΠ°ΠΊ Π€Π°Ρ€Π°Π΄Π΅ΠΉ ΠΎΡ‚ΠΊΡ€Ρ‹Π» ΡΠ»Π΅ΠΊΡ‚Ρ€ΠΎΠΌΠ°Π³Π½ΠΈΡ‚Π½ΡƒΡŽ ΠΈΠ½Π΄ΡƒΠΊΡ†ΠΈΡŽ, стали Π°ΠΊΡ‚ΠΈΠ²Π½ΠΎ ΠΈΡΠΏΠΎΠ»ΡŒΠ·ΠΎΠ²Π°Ρ‚ΡŒ Π³Π΅Π½Π΅Ρ€Π°Ρ‚ΠΎΡ€Ρ‹ спСрва постоянного, Π° послС ΠΈ ΠΏΠ΅Ρ€Π΅ΠΌΠ΅Π½Π½ΠΎΠ³ΠΎ Ρ‚ΠΎΠΊΠ°.

Π˜ΡΠΏΠΎΠ»ΡŒΠ·ΡƒΠ΅Ρ‚ΡΡ ΡƒΠΆΠ΅ извСстная Ρ„ΠΎΡ€ΠΌΡƒΠ»Π°:

I = U/Z

ПолноС сопротивлСниС Ρ‚ΠΎΠΊΠ° – это ΡΠΎΠ²ΠΎΠΊΡƒΠΏΠ½ΠΎΡΡ‚ΡŒ Π°ΠΊΡ‚ΠΈΠ²Π½ΠΎΠ³ΠΎ, Π° Ρ‚Π°ΠΊΠΆΠ΅ ΠΈΠ½Π΄ΡƒΠΊΡ‚ΠΈΠ²Π½ΠΎΠ³ΠΎ ΠΈ Смкостного сопротивлСний. 2

ЦСпь

Π’ Ρ‚Π°ΠΊΠΎΠΉ Ρ†Π΅ΠΏΠΈ колСбания Ρ‚ΠΎΠΊΠ° ΠΈ напряТСния Ρ€Π°Π·Π½Ρ‹Π΅ ΠΏΠΎ Ρ„Π°Π·Π΅, Π° Ρ€Π°Π·Π½ΠΎΡΡ‚ΡŒ Ρ„Π°Π· зависит ΠΎΡ‚ индуктивности ΠΊΠ°Ρ‚ΡƒΡˆΠΊΠΈ ΠΈ ёмкости кондСнсатора:

U = Um sin (Ο‰t)

I = Im sin (Ο‰t + Ρ„)

Π—Π°ΠΊΠΎΠ½ Ома для постоянного Ρ‚ΠΎΠΊΠ°

Π’ Π΄Π°Π½Π½ΠΎΠΌ случаС частота Π±ΡƒΠ΄Π΅Ρ‚ Ρ€Π°Π²Π½ΡΡ‚ΡŒΡΡ Π½ΡƒΠ»Π΅Π²ΠΎΠΌΡƒ Π·Π½Π°Ρ‡Π΅Π½ΠΈΡŽ, поэтому ΠΎΡΡ‚Π°Π»ΡŒΠ½Ρ‹Π΅ ΠΏΠΎΠΊΠ°Π·Π°Ρ‚Π΅Π»ΠΈ Ρ‚Π°ΠΊΠΆΠ΅ Π±ΡƒΠ΄ΡƒΡ‚ Π½ΡƒΠ»Π΅Π²Ρ‹ΠΌΠΈ соотвСтствСнно, Π² Ρ‚ΠΎ врСмя ΠΊΠ°ΠΊ Π·Π½Π°Ρ‡Π΅Π½ΠΈΠ΅ ёмкости достигнСт бСсконСчности. ЦСпь разорвётся. ΠŸΠΎΡΡ‚ΠΎΠΌΡƒ ΠΎΡ‚ΡΡŽΠ΄Π° Π²Ρ‹Ρ‚Π΅ΠΊΠ°Π΅Ρ‚ Π»ΠΎΠ³ΠΈΡ‡Π½Ρ‹ΠΉ Π²Ρ‹Π²ΠΎΠ΄: Ρ€Π΅Π°ΠΊΡ‚ΠΈΠ²Π½ΠΎΠ΅ сопротивлСниС элСмСнтов Π² цСпях постоянного напряТСния отсутствуСт.

Π—Π°ΠΊΠΎΠ½ Ома для ΠΎΠ΄Π½ΠΎΡ€ΠΎΠ΄Π½ΠΎΠ³ΠΎ участка Ρ†Π΅ΠΏΠΈ

Π€ΠΎΡ€ΠΌΡƒΠ»Π° выглядит ΡƒΠΆΠ΅ извСстным ΠΎΠ±Ρ€Π°Π·ΠΎΠΌ:

I = U/R

Π’ Π΄Π°Π½Π½ΠΎΠΌ случаС Π³Π»Π°Π²Π½ΠΎΠΉ характСристикой ΠΏΡ€ΠΎΠ²ΠΎΠ΄Π½ΠΈΠΊΠ° остаётся сопротивлСниС. ΠžΡ‚ Ρ‚ΠΎΠ³ΠΎ, ΠΊΠ°ΠΊ выглядит ΠΏΡ€ΠΎΠ²ΠΎΠ΄Π½ΠΈΠΊ, зависит количСство ΡƒΠ·Π»ΠΎΠ² кристалличСской Ρ€Π΅ΡˆΡ‘Ρ‚ΠΊΠΈ ΠΈ Π°Ρ‚ΠΎΠΌΠΎΠ² примСсСй. ΠŸΠΎΡΡ‚ΠΎΠΌΡƒ элСктроны ΠΌΠΎΠ³ΡƒΡ‚ Π·Π°ΠΌΠ΅Π΄Π»ΡΡ‚ΡŒΡΡ ΠΈΠ»ΠΈ ΡƒΡΠΊΠΎΡ€ΡΡ‚ΡŒΡΡ.

Π‘ΠΎΠΏΡ€ΠΎΡ‚ΠΈΠ²Π»Π΅Π½ΠΈΠ΅ Π±ΡƒΠ΄Π΅Ρ‚ Π·Π°Π²ΠΈΡΠ΅Ρ‚ΡŒ ΠΎΡ‚ Π²ΠΈΠ΄Π° ΠΏΡ€ΠΎΠ²ΠΎΠ΄Π½ΠΈΠΊΠ°, Π° ΠΈΠΌΠ΅Π½Π½ΠΎ ΠΎΡ‚ Π΅Π³ΠΎ сСчСния, ΠΌΠ°Ρ‚Π΅Ρ€ΠΈΠ°Π»Π° ΠΈ Π΄Π»ΠΈΠ½Ρ‹:

R = p (L/S)

Π—Π°ΠΊΠΎΠ½ Ома для Π½Π΅ΠΎΠ΄Π½ΠΎΡ€ΠΎΠ΄Π½ΠΎΠ³ΠΎ участка Ρ†Π΅ΠΏΠΈ

ΠŸΡ€ΠΈ Ρ€Π΅ΡˆΠ΅Π½ΠΈΠΈ Π·Π°Π΄Π°Ρ‡ΠΈ становится понятным, Ρ‡Ρ‚ΠΎ для Ρ‚ΠΎΠ³ΠΎ, Ρ‡Ρ‚ΠΎΠ±Ρ‹ поддСрТивался ΡΡ‚Π°Π±ΠΈΠ»ΡŒΠ½Ρ‹ΠΉ Ρ‚ΠΎΠΊ Π² Π·Π°ΠΌΠΊΠ½ΡƒΡ‚ΠΎΠΉ Ρ†Π΅ΠΏΠΈ, Π½ΡƒΠΆΠ½Ρ‹ силы ΡΠΎΠ²Π΅Ρ€ΡˆΠ΅Π½Π½ΠΎΠΉ Π΄Ρ€ΡƒΠ³ΠΎΠΉ ΠΏΡ€ΠΈΡ€ΠΎΠ΄Ρ‹, Π° Π½Π΅ кулоновскиС. Π’ этом случаС ΠΌΠΎΠΆΠ½ΠΎ Π·Π°ΠΌΠ΅Ρ‚ΠΈΡ‚ΡŒ Ρ‚Π°ΠΊΡƒΡŽ Π·Π°ΠΊΠΎΠ½ΠΎΠΌΠ΅Ρ€Π½ΠΎΡΡ‚ΡŒ: заряды, ΠΊΠΎΡ‚ΠΎΡ€Ρ‹Π΅ Π½ΠΈΠΊΠ°ΠΊ Π½Π΅ ΡΠΎΠΏΡ€ΠΈΠΊΠ°ΡΠ°ΡŽΡ‚ΡΡ Π΄Ρ€ΡƒΠ³ с Π΄Ρ€ΡƒΠ³ΠΎΠΌ, Π²Ρ‹ΡΡ‚ΡƒΠΏΠ°ΡŽΡ‚ Π² Π΄Π²ΡƒΡ… ролях ΠΎΠ΄Π½ΠΎΠ²Ρ€Π΅ΠΌΠ΅Π½Π½ΠΎ, Ρ‚ΠΎ Π΅ΡΡ‚ΡŒ ΠΎΠ½ΠΈ ΡΠ²Π»ΡΡŽΡ‚ΡΡ силами элСктричСского поля ΠΈ силами ΠΈΠ½ΠΎΠ³ΠΎ Π²ΠΈΠ΄Π° – сторонними Π² это ΠΆΠ΅ врСмя. Участок, Π½Π° ΠΊΠΎΡ‚ΠΎΡ€ΠΎΠΌ Π·Π°ΠΌΠ΅Ρ‡Π΅Π½Π° данная Π·Π°ΠΊΠΎΠ½ΠΎΠΌΠ΅Ρ€Π½ΠΎΡΡ‚ΡŒ, называСтся Π½Π΅ΠΎΠ΄Π½ΠΎΡ€ΠΎΠ΄Π½Ρ‹ΠΌ.

НСоднородный участок Ρ†Π΅ΠΏΠΈ

Π€ΠΎΡ€ΠΌΡƒΠ»Π° ΠΏΡ€ΠΈΠ½ΠΈΠΌΠ°Π΅Ρ‚ Π²ΠΈΠ΄:

E = Eq + Est

Π—Π°ΠΊΠΎΠ½ Ома Π² Π΄Π°Π½Π½ΠΎΠΌ ΠΏΠΎΠ΄Ρ€Π°Π·Π΄Π΅Π»Π΅ Π±Ρ‹Π» сформулирован Ρ‚Π°ΠΊΠΈΠΌ ΠΎΠ±Ρ€Π°Π·ΠΎΠΌ: сила Ρ‚ΠΎΠΊΠ° прямо ΠΏΡ€ΠΎΠΏΠΎΡ€Ρ†ΠΈΠΎΠ½Π°Π»ΡŒΠ½Π° Π½Π°ΠΏΡ€ΡΠΆΠ΅Π½ΠΈΡŽ Π½Π° Π΄Π°Π½Π½ΠΎΠΌ участкС ΠΈ ΠΎΠ±Ρ€Π°Ρ‚Π½ΠΎ ΠΏΡ€ΠΎΠΏΠΎΡ€Ρ†ΠΈΠΎΠ½Π°Π»ΡŒΠ½Π° Π΅Π³ΠΎ ΠΏΠΎΠ»Π½ΠΎΠΌΡƒ ΡΠΎΠΏΡ€ΠΎΡ‚ΠΈΠ²Π»Π΅Π½ΠΈΡŽ.

Π˜Ρ‚Π°ΠΊ, готовая Ρ„ΠΎΡ€ΠΌΡƒΠ»Π°:

I = U12/R, Π³Π΄Π΅ U12

Π—Π°ΠΊΠΎΠ½ Ома для ΠΌΠ°Π³Π½ΠΈΡ‚Π½ΠΎΠΉ Ρ†Π΅ΠΏΠΈ

Π’ ΠΊΠ°ΠΆΠ΄ΠΎΠΌ элСктромагнитС совмСщСны нСсколько Π²Π°ΠΆΠ½Ρ‹Ρ… элСмСнтов: ΡΡ‚Π°Π»ΡŒΠ½ΠΎΠΉ сСрдСчник ΠΈ ΠΊΠ°Ρ‚ΡƒΡˆΠΊΠ°. По послСднСй ΠΏΡ€ΠΎΡ‚Π΅ΠΊΠ°Π΅Ρ‚ Ρ‚ΠΎΠΊ. ΠŸΡ€ΠΈ совмСщСнии Π½Π΅ΡΠΊΠΎΠ»ΡŒΠΊΠΈΡ… участков образуСтся магнитная Ρ†Π΅ΠΏΡŒ.

ΠŸΡ€ΠΈ ΠΊΠΎΠ»ΡŒΡ†Π΅Π²ΠΎΠΌ ΠΌΠ°Π³Π½ΠΈΡ‚ΠΎΠΏΡ€ΠΎΠ²ΠΎΠ΄Π΅ всС ΠΏΠΎΠ»Π΅ находится Π²Π½ΡƒΡ‚Ρ€ΠΈ ΠΊΠΎΠ»ΡŒΡ†Π°. Π’ΠΎΠ³Π΄Π° ΠΏΠΎΡ‚ΠΎΠΊ Π² ΠΌΠ°Π³Π½ΠΈΡ‚ΠΎΠΏΡ€ΠΎΠ²ΠΎΠ΄Π΅ Ρ€Π°Π²Π΅Π½:

Π€ = Вср S = ΞΌHср S

Π€ΠΎΡ€ΠΌΡƒΠ»Π° Π·Π°ΠΊΠΎΠ½Π° для ΠΌΠ°Π³Π½ΠΈΡ‚Π½ΠΎΠΉ Ρ†Π΅ΠΏΠΈ:

Π€ΠΎΡ€ΠΌΡƒΠ»Π° Π·Π°ΠΊΠΎΠ½Π° ΠΎΠΌΠ° для ΠΌΠ°Π³Π½ΠΈΡ‚Π½ΠΎΠΉ Ρ†Π΅ΠΏΠΈ

Π—Π°Π΄Π°Ρ‡ΠΈ с Ρ€Π΅ΡˆΠ΅Π½ΠΈΡΠΌΠΈ Π½Π° Π·Π°ΠΊΠΎΠ½ Ома

Π—Π°Π΄Π°Ρ‡Π° β„–1

Нихромовая ΠΏΡ€ΠΎΠ²ΠΎΠ»ΠΎΠΊΠ° Π΄Π»ΠΈΠ½ΠΎΠΉ 120 ΠΌ ΠΈ ΠΏΠ»ΠΎΡ‰Π°Π΄ΡŒΡŽ сСчСния 0,5 ΠΌΠΌ Π²ΠΊΠ»ΡŽΡ‡Π΅Π½Π° Π² Ρ†Π΅ΠΏΡŒ с напряТСниСм 127 Π’. ΠžΠΏΡ€Π΅Π΄Π΅Π»ΠΈΡ‚ΡŒ силу Ρ‚ΠΎΠΊΠ° Π² ΠΏΡ€ΠΎΠ²ΠΎΠ»ΠΎΠΊΠ΅.

Π”Π°Π½ΠΎ:

  • l = 120 ΠΌ,
  • S = 0,5 ΠΌΠΌ,
  • U = 127 Π’,
  • p = 1,1 Ом*ΠΌΠΌ2 /ΠΌ.

Найти: I β€” ?

РСшСниС:

  • R = p * l / S,
  • R = 1,1 Ом*ΠΌΠΌ2 /ΠΌ * 120 ΠΌ : 0,5 ΠΌΠΌ = 264 Ом,
  • I = 127 Π’ : 264 Ом = 0,48 А.

ΠžΡ‚Π²Π΅Ρ‚: I = 0,48 Ом

Π—Π°Π΄Π°Ρ‡Π° β„–2

Нихромовая ΠΏΡ€ΠΎΠ²ΠΎΠ»ΠΎΠΊΠ° Π΄Π»ΠΈΠ½ΠΎΠΉ 120 ΠΌ ΠΈ ΠΏΠ»ΠΎΡ‰Π°Π΄ΡŒΡŽ сСчСния 0,5 ΠΌΠΌ Π²ΠΊΠ»ΡŽΡ‡Π΅Π½Π° Π² Ρ†Π΅ΠΏΡŒ с напряТСниСм 220 Π’. ΠžΠΏΡ€Π΅Π΄Π΅Π»ΠΈΡ‚ΡŒ силу Ρ‚ΠΎΠΊΠ° Π² ΠΏΡ€ΠΎΠ²ΠΎΠ»ΠΎΠΊΠ΅.

Π”Π°Π½ΠΎ:

  • l = 120 ΠΌ,
  • S = 0,5 ΠΌΠΌ,
  • U = 220 Π’,
  • p = 1,1 Ом*ΠΌΠΌ2 /ΠΌ.

Найти: I β€” ?

РСшСниС:

  • R = p * l / S,
  • R = 1,1 Ом*ΠΌΠΌ2 /ΠΌ * 120 ΠΌ : 0,5 ΠΌΠΌ = 264 Ом,
  • I = 220 Π’ : 264 Ом = 0,83 А.

ΠžΡ‚Π²Π΅Ρ‚: I = 0,83 Ом

Π—Π°Π΄Π°Ρ‡Π° β„–3

Π”Π°Π½ΠΎ:

  • U = 15 Π’,
  • R1 = 3 Ом,
  • R2 = R3 = 4 Ом.

Найти: I β€” ?

РСшСниС:

  • R2 ΠΈ R3 соСдинСны ΠΏΠ°Ρ€Π°Π»Π»Π΅Π»ΡŒΠ½ΠΎ R2 = R3, R2.3 = R2 / 2 = 2 Ом, составим ΡΠΊΠ²ΠΈΠ²Π°Π»Π΅Π½Ρ‚Π½ΡƒΡŽ схСму:
  • R = R1 + R2,3
  • R = 3 Ом + 2 Ом = 5 Ом
  • НайдСм силу Ρ‚ΠΎΠΊΠ° Π½Π° участкС Ρ†Π΅ΠΏΠΈ ΠΏΠΎ Π·Π°ΠΊΠΎΠ½Ρƒ Ома I = U / R
  • I = 15 Π’ / 5 Ом = 3 А

ΠžΡ‚Π²Π΅Ρ‚: I = 3 A.

Π­Π»Π΅ΠΊΡ‚Ρ€ΠΎΡ‚Π΅Ρ…Π½ΠΈΠΊΠ°. ΠžΡΠ½ΠΎΠ²Ρ‹. Π—Π°ΠΊΠΎΠ½ Ома — Всё ΠΎΠ± энСргСтикС

Π­Π»Π΅ΠΊΡ‚Ρ€ΠΎΡ‚Π΅Ρ…Π½ΠΈΠΊΠ°. ΠžΡΠ½ΠΎΠ²Ρ‹. Π—Π°ΠΊΠΎΠ½ Ома

Π’ элСктротСхникС, ΠΊΠ°ΠΊ ΠΈ Π² любой Π΄Ρ€ΡƒΠ³ΠΎΠΉ Π½Π°ΡƒΠΊΠ΅, ΡΡƒΡ‰Π΅ΡΡ‚Π²ΡƒΡŽΡ‚ Π±Π°Π·ΠΎΠ²Ρ‹Π΅ понятия, Π±Π΅Π· понимания ΠΊΠΎΡ‚ΠΎΡ€Ρ‹Ρ… Π½Π΅ удастся ΠΎΠ²Π»Π°Π΄Π΅Ρ‚ΡŒ этой ΠΎΠ±Π»Π°ΡΡ‚ΡŒΡŽ Π·Π½Π°Π½ΠΈΠΉ. Π—Π΄Π΅ΡΡŒ Ρ‚Π°ΠΊΠΈΠΌΠΈ понятиями ΡΠ²Π»ΡΡŽΡ‚ΡΡ элСктричСскоС напряТСниС, элСктричСский Ρ‚ΠΎΠΊ ΠΈ элСктричСскоС сопротивлСниС.

Π—Π°ΠΊΠΎΠ½ Ома

Π—Π°ΠΊΠΎΠ½ Ома Π±Ρ‹Π» ΠΎΡ‚ΠΊΡ€Ρ‹Ρ‚ Π² Ρ€Π΅Π·ΡƒΠ»ΡŒΡ‚Π°Ρ‚Π΅ экспСримСнтов Π“Π΅ΠΎΡ€Π³Π° Ома с Π³Π°Π»ΡŒΠ²Π°Π½ΠΎΠΌΠ΅Ρ‚Ρ€ΠΎΠΌ ΠΈ простой элСктричСской Ρ†Π΅ΠΏΡŒΡŽ ΠΈΠ· источника Π­Π”Π‘ ΠΈ сопротивлСния. Π‘ΠΎ Π²Ρ€Π΅ΠΌΠ΅Π½Π΅ΠΌ Ρ„ΠΎΡ€ΠΌΡƒΠ»Π° получСнная Омом ΠΏΡ€Π΅Ρ‚Π΅Ρ€ΠΏΠ΅Π»Π° нСсколько ΠΈΠ·ΠΌΠ΅Π½Π΅Π½ΠΈΠΉ.

Π—Π°ΠΊΠΎΠ½ Ома для участка Ρ†Π΅ΠΏΠΈ Π±Π΅Π· Π­Π”Π‘

ΠœΠΎΠΆΠ΅Ρ‚ Π±Ρ‹Ρ‚ΡŒ сформулирован Ρ‡Π΅Ρ€Π΅Π· сопротивлСниС [1, стр.33][2, стр.15]:

\begin{equation} I = {U_{ab}\over R}; \end{equation}

Π“Π΄Π΅:

  • I — Ρ‚ΠΎΠΊ Ρ‡Π΅Ρ€Π΅Π· участок ab элСктричСской Ρ†Π΅ΠΏΠΈ;
  • Uab — напряТСниС Π½Π° участкС ab элСктричСской Ρ†Π΅ΠΏΠΈ;
  • R — сопротивлСниС участка ab элСктричСской Ρ†Π΅ΠΏΠΈ.

Или Ρ‡Π΅Ρ€Π΅Π· ΠΏΡ€ΠΎΠ²ΠΎΠ΄ΠΈΠΌΠΎΡΡ‚ΡŒ:

\begin{equation} I = U_{ab} Γ— G; \end{equation}

Π“Π΄Π΅:

  • G — ΠΏΡ€ΠΎΠ²ΠΎΠ΄ΠΈΠΌΠΎΡΡ‚ΡŒ участка ab элСктричСской Ρ†Π΅ΠΏΠΈ.

Π€ΠΎΡ€ΠΌΡƒΠ»Π° (1, 2) справСдлива для элСктричСской Ρ†Π΅ΠΏΠΈ прСдставлСнной Π½ΠΈΠΆΠ΅ Π½Π° рисункС 1.

Рисунок 1 — Участок Ρ†Π΅ΠΏΠΈ Π±Π΅Π· Π­Π”Π‘

Π—Π°ΠΊΠΎΠ½ Ома для участка Ρ†Π΅ΠΏΠΈ содСрТащСго Π­Π”Π‘

Или ΠΎΠ±ΠΎΠ±Ρ‰Ρ‘Π½Π½Ρ‹ΠΉ Π·Π°ΠΊΠΎΠ½ Ома. ЀормулируСтся ΡΠ»Π΅Π΄ΡƒΡŽΡ‰ΠΈΠΌ ΠΎΠ±Ρ€Π°Π·ΠΎΠΌ [1, стр.34][2, стр.17]:

\begin{equation} I = {U_{ab} + E\over R}; \end{equation}

Π“Π΄Π΅:

  • I — Ρ‚ΠΎΠΊ Ρ‡Π΅Ρ€Π΅Π· участок ac элСктричСской Ρ†Π΅ΠΏΠΈ;
  • Uab — напряТСниС Π½Π° участкС ab элСктричСской Ρ†Π΅ΠΏΠΈ;
  • E — Π­Π”Π‘ Π½Π° участкС bс элСктричСской Ρ†Π΅ΠΏΠΈ;
  • R — сопротивлСниС участка ab элСктричСской Ρ†Π΅ΠΏΠΈ.

Или Ρ‡Π΅Ρ€Π΅Π· ΠΏΡ€ΠΎΠ²ΠΎΠ΄ΠΈΠΌΠΎΡΡ‚ΡŒ:

\begin{equation} I = {(U_{ab} + E) Γ— G}; \end{equation}

Π“Π΄Π΅:

  • G — ΠΏΡ€ΠΎΠ²ΠΎΠ΄ΠΈΠΌΠΎΡΡ‚ΡŒ участка ab элСктричСской Ρ†Π΅ΠΏΠΈ.

Π€ΠΎΡ€ΠΌΡƒΠ»Π° (3, 4) справСдлива для элСктричСской Ρ†Π΅ΠΏΠΈ прСдставлСнной Π½ΠΈΠΆΠ΅ Π½Π° рисункС 2.

Рисунок 2 — Участок Ρ†Π΅ΠΏΠΈ содСрТащий Π­Π”Π‘

Π—Π°ΠΊΠΎΠ½ Ома для ΠΏΠΎΠ»Π½ΠΎΠΉ Ρ†Π΅ΠΏΠΈ

Π—Π°ΠΊΠΎΠ½ формулируСтся ΡΠ»Π΅Π΄ΡƒΡŽΡ‰ΠΈΠΌ ΠΎΠ±Ρ€Π°Π·ΠΎΠΌ [1, стр.34][2, стр.17]:

\begin{equation} I = {E\over {R + r}}; \end{equation}

Π“Π΄Π΅:

  • I — Ρ‚ΠΎΠΊ Π² элСктричСской Ρ†Π΅ΠΏΠΈ;
  • E — Π­Π”Π‘ элСктричСской Ρ†Π΅ΠΏΠΈ;
  • R — сопротивлСниС элСктричСской Ρ†Π΅ΠΏΠΈ;
  • r — Π²Π½ΡƒΡ‚Ρ€Π΅Π½Π½Π΅Π΅ сопротивлСниС источника Π­Π”Π‘.

Π€ΠΎΡ€ΠΌΡƒΠ»ΠΈΡ€ΠΎΠ²ΠΊΠ° выраТСния (5) Ρ‡Π΅Ρ€Π΅Π· ΠΏΡ€ΠΎΠ²ΠΎΠ΄ΠΈΠΌΠΎΡΡ‚ΡŒ Π½Π΅ΡƒΠ΄ΠΎΠ±Π½Π° ΠΈ здСсь ΠΏΡ€ΠΈΠ²Π΅Π΄Π΅Π½Π° Π½Π΅ Π±ΡƒΠ΄Π΅Ρ‚. НиТС Π½Π° рисункС 3 ΠΈΠ·ΠΎΠ±Ρ€Π°ΠΆΠ΅Π½Π° схСма элСктричСской Ρ†Π΅ΠΏΠΈ для ΠΊΠΎΡ‚ΠΎΡ€ΠΎΠΉ справСдливо Π²Ρ‹Ρ€Π°ΠΆΠ΅Π½ΠΈΠ΅ (5).

Рисунок 3 — Полная Ρ†Π΅ΠΏΡŒ

На схСмС Π²ΠΈΠ΄Π½ΠΎ, Ρ‡Ρ‚ΠΎ R ΠΈ r соСдинСны ΠΏΠΎΡΠ»Π΅Π΄ΠΎΠ²Π°Ρ‚Π΅Π»ΡŒΠ½ΠΎ, Π° Π² Ρ„ΠΎΡ€ΠΌΡƒΠ»Π΅ это ΠΎΡ‚Ρ€Π°ΠΆΠ΅Π½ΠΎ ΠΊΠ°ΠΊ сумма R (сопротивлСния Ρ†Π΅ΠΏΠΈ) ΠΈ r (Π²Π½ΡƒΡ‚Ρ€Π΅Π½Π½Π΅Π³ΠΎ сопротивлСния источника Π­Π”Π‘). Π—Π°ΠΌΠ΅Π½ΠΈΠΌ Π²Ρ‹Ρ€Π°ΠΆΠ΅Π½ΠΈΠ΅ R + r Π½Π° RΠΏ

\begin{equation} I = {E\over R_ΠΏ}; \end{equation}

Π“Π΄Π΅:

  • RΠΏ — ΠΏΠΎΠ»Π½ΠΎΠ΅ сопротивлСниС элСктричСской Ρ†Π΅ΠΏΠΈ (Π²ΠΊΠ»ΡŽΡ‡Π°Ρ сопротивлСниС источника Π­Π”Π‘).
Π—Π°ΠΊΠΎΠ½ Ома Π² Π΄ΠΈΡ„Ρ„Π΅Ρ€Π΅Π½Ρ†ΠΈΠ°Π»ΡŒΠ½ΠΎΠΉ Ρ„ΠΎΡ€ΠΌΠ΅

Π—Π°ΠΊΠΎΠ½ Ома Π² Π΄ΠΈΡ„Ρ„Π΅Ρ€Π΅Π½Ρ†ΠΈΠ°Π»ΡŒΠ½ΠΎΠΉ Ρ„ΠΎΡ€ΠΌΠ΅, прСдставлСнный Π² Π²Ρ‹Ρ€Π°ΠΆΠ΅Π½ΠΈΠΈ (7), справСдлив для Π½Π΅ΠΎΠ΄Π½ΠΎΡ€ΠΎΠ΄Π½ΠΎΠ³ΠΎ, Π½ΠΎ ΠΈΠ·ΠΎΡ‚Ρ€ΠΎΠΏΠ½ΠΎΠ³ΠΎ вСщСства [3].

\begin{equation} \vec E = {ρ Γ— \vec\jmath}; \end{equation}

Π“Π΄Π΅:

  • \(\vec\jmath\) — ΠΏΠ»ΠΎΡ‚Π½ΠΎΡΡ‚ΡŒ Ρ‚ΠΎΠΊΠ°;
  • ρ — ΡƒΠ΄Π΅Π»ΡŒΠ½ΠΎΠ΅ сопротивлСниС;
  • \(\vec E\) — Π½Π°ΠΏΡ€ΡΠΆΡ‘Π½Π½ΠΎΡΡ‚ΡŒ элСктричСского поля.

ΠŸΡ€ΠΈΠΌΠ΅Ρ€Ρ‹ примСнСния

НиТС ΠΏΡ€ΠΈΠ²Π΅Π΄Π΅Π½Ρ‹ нСсколько ΠΏΡ€ΠΈΠΌΠ΅Ρ€ΠΎΠ² для дСмонстрации примСнСния Ρ€Π°Π·Π½Ρ‹Ρ… Ρ„ΠΎΡ€ΠΌΡƒΠ»ΠΈΡ€ΠΎΠ²ΠΎΠΊ Π·Π°ΠΊΠΎΠ½Π° Ома.

ΠŸΡ€ΠΈΠΌΠ΅Ρ€ 1

Π‘Ρ…Π΅ΠΌΠ° задания ΠΏΡ€ΠΈΠ²Π΅Π΄Π΅Π½Π° Π½Π° рисункС 4. На схСмС R = 5,2 Ом, U = 26 Π’. ΠžΠΏΡ€Π΅Π΄Π΅Π»ΠΈΡ‚ΡŒ I.

Рисунок 4 — Π‘Ρ…Π΅ΠΌΠ° ΠΊ 1 ΠΈ 2-ΠΌΡƒ ΠΏΡ€ΠΈΠΌΠ΅Ρ€Ρƒ

Для Ρ€Π΅ΡˆΠ΅Π½ΠΈΡ задания Π²ΠΎΡΠΏΠΎΠ»ΡŒΠ·ΡƒΠ΅ΠΌΡΡ Π²Ρ‹Ρ€Π°ΠΆΠ΅Π½ΠΈΠ΅ΠΌ (1):

\begin{equation} I = {U\over R} = {26\over 5,2} = {5 \ А;} \end{equation}
ΠŸΡ€ΠΈΠΌΠ΅Ρ€ 2

Π‘Ρ…Π΅ΠΌΠ° задания ΠΏΡ€ΠΈΠ²Π΅Π΄Π΅Π½Π° Π½Π° рисункС 4. К Π΄Π°Π½Π½ΠΎΠΌΡƒ участку Ρ†Π΅ΠΏΠΈ ΠΏΡ€ΠΈΠ»ΠΎΠΆΠ΅Π½ΠΎ напряТСниС 24 Π’ ΠΈ ΠΏΠΎ Π½Π΅ΠΌΡƒ ΠΏΡ€ΠΎΡ‚Π΅ΠΊΠ°Π΅Ρ‚ Ρ‚ΠΎΠΊ 1,5 А. ΠžΠΏΡ€Π΅Π΄Π΅Π»ΠΈΡ‚ΡŒ ΠΏΡ€ΠΎΠ²ΠΎΠ΄ΠΈΠΌΠΎΡΡ‚ΡŒ участка Ρ†Π΅ΠΏΠΈ.

Для Ρ€Π΅ΡˆΠ΅Π½ΠΈΡ задания ΠΏΡ€Π΅ΠΎΠ±Ρ€Π°Π·ΡƒΠ΅ΠΌ Π²Ρ‹Ρ€Π°ΠΆΠ΅Π½ΠΈΠ΅ (2) ΠΎΡ‚Π½ΠΎΡΠΈΡ‚Π΅Π»ΡŒΠ½ΠΎ G:

\begin{equation} I = {U Γ— G} \ \Rightarrow \ G = {I\over U} = {1,5\over 24} = {0,0625 \ Π‘ΠΌ;} \end{equation}
ΠŸΡ€ΠΈΠΌΠ΅Ρ€ 3

Π‘Ρ…Π΅ΠΌΠ° задания ΠΏΡ€ΠΈΠ²Π΅Π΄Π΅Π½Π° Π½Π° рисункС 5. На схСмС U = 220 Π’, I = 0,5 А, R = 140 Ом. ΠžΠΏΡ€Π΅Π΄Π΅Π»ΠΈΡ‚ΡŒ E.

Рисунок 5 — Π‘Ρ…Π΅ΠΌΠ° ΠΊ 3-ΠΌΡƒ ΠΏΡ€ΠΈΠΌΠ΅Ρ€Ρƒ

Для Ρ€Π΅ΡˆΠ΅Π½ΠΈΡ задания ΠΏΡ€Π΅ΠΎΠ±Ρ€Π°Π·ΡƒΠ΅ΠΌ Π²Ρ‹Ρ€Π°ΠΆΠ΅Π½ΠΈΠ΅ (3) ΠΎΡ‚Π½ΠΎΡΠΈΡ‚Π΅Π»ΡŒΠ½ΠΎ E:

\begin{equation} I = {U — E\over R} \ \Rightarrow \ {I Γ— R} = {U — E} \ \Rightarrow \ E = {U — I Γ— R}; \end{equation}

ΠŸΠΎΠ΄ΡΡ‚Π°Π²ΠΈΠΌ Π² Π²Ρ‹Ρ€Π°ΠΆΠ΅Π½ΠΈΠ΅ (10) извСстныС Π²Π΅Π»ΠΈΡ‡ΠΈΠ½Ρ‹:

\begin{equation} E = {U — I Γ— R} = {220 — 0,5 Γ— 140} = {150 \ Π’;} \end{equation}

ΠŸΡ€ΠΈΠΌΠ΅Ρ€ 4

Π‘ΠΎΠΏΡ€ΠΎΡ‚ΠΈΠ²Π»Π΅Π½ΠΈΠ΅ элСктричСской Ρ†Π΅ΠΏΠΈ, ΠΏΡ€ΠΈΠ²Π΅Π΄Π΅Π½Π½ΠΎΠΉ Π½Π° рисункС 3 составляСт 12 Ом, напряТСниС источника Π­Π”Π‘ Π²ΠΊΠ»ΡŽΡ‡Π΅Π½Π½ΠΎΠ³ΠΎ Π² Ρ†Π΅ΠΏΡŒ — 9 Π’. Π˜Π·ΠΌΠ΅Ρ€Π΅Π½ΠΈΡ ΠΏΠΎΠΊΠ°Π·Π°Π»ΠΈ, Ρ‡Ρ‚ΠΎ ΠΏΠΎ Ρ†Π΅ΠΏΠΈ ΠΏΡ€ΠΎΡ‚Π΅ΠΊΠ°Π΅Ρ‚ Ρ‚ΠΎΠΊ 0,72 А. НСобходимо ΠΎΠΏΡ€Π΅Π΄Π΅Π»ΠΈΡ‚ΡŒ Π²Π½ΡƒΡ‚Ρ€Π΅Π½Π½Π΅Π΅ сопротивлСниС источника Π­Π”Π‘.

ΠŸΡ€Π΅ΠΎΠ±Ρ€Π°Π·ΡƒΠ΅ΠΌ Π²Ρ‹Ρ€Π°ΠΆΠ΅Π½ΠΈΠ΅ (5) ΠΎΡ‚Π½ΠΎΡΠΈΡ‚Π΅Π»ΡŒΠ½ΠΎ r:

\begin{equation} I = {E\over {R + r}} \ \Rightarrow \ {I Γ— (R + r)} = E \ \Rightarrow \ {I Γ— r} = {E — I Γ— R} \ \Rightarrow \ r = {E — I Γ— R\over I}; \end{equation}

ΠžΠΏΡ€Π΅Π΄Π΅Π»ΠΈΠΌ Π²Π½ΡƒΡ‚Ρ€Π΅Π½Π½Π΅ΠΉ сопротивлСниС источника Π­Π”Π‘, подставив Π² Π²Ρ‹Ρ€Π°ΠΆΠ΅Π½ΠΈΠ΅ (10) извСстныС Π²Π΅Π»ΠΈΡ‡ΠΈΠ½Ρ‹:

\begin{equation} r = {E — I Γ— R\over I} = {9 — 0,72 Γ— 12\over 0,72} = {0,36\over 0,72} = {0,5 \ Ом;} \end{equation}

Π˜ΡΠΏΠΎΠ»ΡŒΠ·ΠΎΠ²Π°Π½Π½Ρ‹Π΅ Ρ‚Π΅Ρ€ΠΌΠΈΠ½Ρ‹

ЭлСктричСский ΠΏΠΎΡ‚Π΅Π½Ρ†ΠΈΠ°Π» Ρ‚ΠΎΡ‡ΠΊΠΈ:

ЀизичСская Π²Π΅Π»ΠΈΡ‡ΠΈΠ½Π°, равная ΠΏΠΎΡ‚Π΅Π½Ρ†ΠΈΠ°Π»ΡŒΠ½ΠΎΠΉ энСргии, ΠΊΠΎΡ‚ΠΎΡ€ΠΎΠΉ ΠΎΠ±Π»Π°Π΄Π°Π΅Ρ‚ элСмСнтарный ΠΏΠΎΠ»ΠΎΠΆΠΈΡ‚Π΅Π»ΡŒΠ½Ρ‹ΠΉ заряд, ΠΏΠΎΠΌΠ΅Ρ‰Π΅Π½Π½Ρ‹ΠΉ Π² элСктричСскоС ΠΏΠΎΠ»Π΅.

ΠŸΠΎΡ‚Π΅Π½Ρ†ΠΈΠ°Π» обозначаСтся Π±ΡƒΠΊΠ²ΠΎΠΉ Ο† грСчСского Π°Π»Ρ„Π°Π²ΠΈΡ‚Π° ΠΈ измСряСтся Π² Π²ΠΎΠ»ΡŒΡ‚Π°Ρ… (Π’). Он Π½Π΅ ΠΈΠΌΠ΅Π΅Ρ‚ направлСния ΠΈ записываСтся ΠΊΠ°ΠΊ скаляр.

ЭлСктричСскоС напряТСниС:

ЀизичСская Π²Π΅Π»ΠΈΡ‡ΠΈΠ½Π°, равная количСству энСргии, Π·Π°Ρ‚Ρ€Π°Ρ‡Π΅Π½Π½ΠΎΠΉ Π½Π° пСрСнос Π΅Π΄ΠΈΠ½ΠΈΡ‡Π½ΠΎΠ³ΠΎ заряда ΠΈΠ· Ρ‚ΠΎΡ‡ΠΊΠΈ А Π² Ρ‚ΠΎΡ‡ΠΊΡƒ Π‘ элСктромагнитного поля, опрСдСляСмая ΠΊΠ°ΠΊ Ρ€Π°Π·Π½ΠΎΡΡ‚ΡŒ ΠΏΠΎΡ‚Π΅Π½Ρ†ΠΈΠ°Π»ΠΎΠ² этих Ρ‚ΠΎΡ‡Π΅ΠΊ: Uab = Ο†a — Ο†b.

НапряТСниС обозначаСтся Π±ΡƒΠΊΠ²ΠΎΠΉ U (u) латинского Π°Π»Ρ„Π°Π²ΠΈΡ‚Π° ΠΈ измСряСтся Π² Π²ΠΎΠ»ΡŒΡ‚Π°Ρ… (Π’). НапряТСниС — скалярная Π²Π΅Π»ΠΈΡ‡ΠΈΠ½Π°, Π½ΠΎ Π½Π° элСктричСских схСмах ΡƒΠΊΠ°Π·Ρ‹Π²Π°ΡŽΡ‚ Π΅Π³ΠΎ ΠΏΠΎΠ»ΠΎΠΆΠΈΡ‚Π΅Π»ΡŒΠ½ΠΎΠ΅ Π½Π°ΠΏΡ€Π°Π²Π»Π΅Π½ΠΈΠ΅.

ЭлСктродвиТущая сила (Π­Π”Π‘):

Π’Π°ΠΊΠΆΠ΅ ΠΊΠ°ΠΊ ΠΈ напряТСниС это физичСская Π²Π΅Π»ΠΈΡ‡ΠΈΠ½Π°, равная количСству энСргии, Π·Π°Ρ‚Ρ€Π°Ρ‡Π΅Π½Π½ΠΎΠΉ Π½Π° пСрСнос Π΅Π΄ΠΈΠ½ΠΈΡ‡Π½ΠΎΠ³ΠΎ заряда ΠΈΠ· ΠΎΠ΄Π½ΠΎΠΉ Ρ‚ΠΎΡ‡ΠΊΠΈ элСктромагнитного поля Π² Π΄Ρ€ΡƒΠ³ΡƒΡŽ.

Π­Π”Π‘ обозначаСтся Π±ΡƒΠΊΠ²ΠΎΠΉ E (e) латинского Π°Π»Ρ„Π°Π²ΠΈΡ‚Π° ΠΈ измСряСтся Π² Π²ΠΎΠ»ΡŒΡ‚Π°Ρ… (Π’). Π­Π”Π‘ — скалярная Π²Π΅Π»ΠΈΡ‡ΠΈΠ½Π°, Π½ΠΎ Π½Π° элСктричСских схСмах ΡƒΠΊΠ°Π·Ρ‹Π²Π°ΡŽΡ‚ Π΅Ρ‘ ΠΏΠΎΠ»ΠΎΠΆΠΈΡ‚Π΅Π»ΡŒΠ½ΠΎΠ΅ Π½Π°ΠΏΡ€Π°Π²Π»Π΅Π½ΠΈΠ΅. Она числСнно Ρ€Π°Π²Π½Π° Π½Π°ΠΏΡ€ΡΠΆΠ΅Π½ΠΈΡŽ Π½Π° Π·Π°ΠΆΠΈΠΌΠ°Ρ… Π½Π΅ ΠΏΠΎΠ΄ΠΊΠ»ΡŽΡ‡Π΅Π½Π½ΠΎΠ³ΠΎ источника.

ЭлСктричСскоС Ρ‚ΠΎΠΊ:

ЀизичСская Π²Π΅Π»ΠΈΡ‡ΠΈΠ½Π°, равная количСству заряТСнных частиц ΠΏΡ€ΠΎΡˆΠ΅Π΄ΡˆΠΈΡ… Ρ‡Π΅Ρ€Π΅Π· ΠΏΠΎΠΏΠ΅Ρ€Π΅Ρ‡Π½ΠΎΠ΅ сСчСниС ΠΏΡ€ΠΎΠ²ΠΎΠ΄Π½ΠΈΠΊΠ° Π·Π° Π΅Π΄ΠΈΠ½ΠΈΡ†Ρƒ Π²Ρ€Π΅ΠΌΠ΅Π½ΠΈ. Как явлСниС — Π½Π°ΠΏΡ€Π°Π²Π»Π΅Π½Π½ΠΎΠ΅ Π΄Π²ΠΈΠΆΠ΅Π½ΠΈΠ΅ заряТСнных частиц.

НапряТСниС обозначаСтся Π±ΡƒΠΊΠ²ΠΎΠΉ I (i) латинского Π°Π»Ρ„Π°Π²ΠΈΡ‚Π° ΠΈ измСряСтся Π² Π°ΠΌΠΏΠ΅Ρ€Π°Ρ… (А). Π’ΠΎΠΊ, Ρ‚Π°ΠΊ ΠΆΠ΅ ΠΊΠ°ΠΊ ΠΈ напряТСниС, Π²Π΅Π»ΠΈΡ‡ΠΈΠ½Π° скалярная, ΠΈ Π½Π° элСктричСских схСмах Ρ‚ΠΎΠΆΠ΅ ΡƒΠΊΠ°Π·Ρ‹Π²Π°ΡŽΡ‚ Π΅Π³ΠΎ ΠΏΠΎΠ»ΠΎΠΆΠΈΡ‚Π΅Π»ΡŒΠ½ΠΎΠ΅ Π½Π°ΠΏΡ€Π°Π²Π»Π΅Π½ΠΈΠ΅ [2, стр.11].

ΠŸΠ»ΠΎΡ‚Π½ΠΎΡΡ‚ΡŒ Ρ‚ΠΎΠΊΠ°:

ЀизичСская Π²Π΅Π»ΠΈΡ‡ΠΈΠ½Π°, ΠΈΠΌΠ΅ΡŽΡ‰Π°Ρ смысл силы элСктричСского Ρ‚ΠΎΠΊΠ°, ΠΏΡ€ΠΎΡ‚Π΅ΠΊΠ°ΡŽΡ‰Π΅Π³ΠΎ Ρ‡Π΅Ρ€Π΅Π· элСмСнт повСрхности Π΅Π΄ΠΈΠ½ΠΈΡ‡Π½ΠΎΠΉ ΠΏΠ»ΠΎΡ‰Π°Π΄ΠΈ.

ΠŸΠ»ΠΎΡ‚Π½ΠΎΡΡ‚ΡŒ Ρ‚ΠΎΠΊΠ° обозначаСтся Π±ΡƒΠΊΠ²ΠΎΠΉ \(\vec\jmath\) латинского Π°Π»Ρ„Π°Π²ΠΈΡ‚Π° ΠΈ измСряСтся Π² Π°ΠΌΠΏΠ΅Ρ€Π°Ρ… Π½Π° ΠΌΠ΅Ρ‚Ρ€ ΠΊΠ²Π°Π΄Ρ€Π°Ρ‚Π½Ρ‹ΠΉ (А/ΠΌ2). ΠŸΠ»ΠΎΡ‚Π½ΠΎΡΡ‚ΡŒ Ρ‚ΠΎΠΊΠ° — вСкторная Π²Π΅Π»ΠΈΡ‡ΠΈΠ½Π° [4].

ЭлСктричСскоС сопротивлСниС:

ЀизичСская Π²Π΅Π»ΠΈΡ‡ΠΈΠ½Π°, Ρ…Π°Ρ€Π°ΠΊΡ‚Π΅Ρ€ΠΈΠ·ΡƒΡŽΡ‰Π°Ρ ΡΠΏΠΎΡΠΎΠ±Π½ΠΎΡΡ‚ΡŒ ΠΏΡ€ΠΎΠ²ΠΎΠ΄Π½ΠΈΠΊΠ° ΠΏΡ€Π΅ΠΏΡΡ‚ΡΡ‚Π²ΠΎΠ²Π°Ρ‚ΡŒ ΠΏΡ€ΠΎΡ…ΠΎΠΆΠ΄Π΅Π½ΠΈΡŽ ΠΏΠΎ Π½Π΅ΠΌΡƒ Ρ‚ΠΎΠΊΠ°.

Π‘ΠΎΠΏΡ€ΠΎΡ‚ΠΈΠ²Π»Π΅Π½ΠΈΠ΅ обозначаСтся Π±ΡƒΠΊΠ²Π°ΠΌΠΈ R (r), X (x) ΠΈΠ»ΠΈ Z (z) латинского Π°Π»Ρ„Π°Π²ΠΈΡ‚Π° (послСдниС Π΄Π²Π° обозначСния ΠΏΡ€ΠΈΠΌΠ΅Π½ΡΡŽΡ‚ΡΡ для Ρ€Π΅Π°ΠΊΡ‚ΠΈΠ²Π½ΠΎΠ³ΠΎ ΠΈ комплСксного сопротивлСния соотвСтствСнно) ΠΈ измСряСтся Π² ΠΎΠΌΠ°Ρ… (Ом). Как ΠΈ ΠΏΡ€Π΅Π΄Ρ‹Π΄ΡƒΡ‰ΠΈΠ΅, сопротивлСниС — скалярная Π²Π΅Π»ΠΈΡ‡ΠΈΠ½Π°.

ЭлСктричСская ΠΏΡ€ΠΎΠ²ΠΎΠ΄ΠΈΠΌΠΎΡΡ‚ΡŒ:

ЀизичСская Π²Π΅Π»ΠΈΡ‡ΠΈΠ½Π°, Ρ…Π°Ρ€Π°ΠΊΡ‚Π΅Ρ€ΠΈΠ·ΡƒΡŽΡ‰Π°Ρ насколько Ρ…ΠΎΡ€ΠΎΡˆΠΎ ΠΏΡ€ΠΎΠ²ΠΎΠ΄Π½ΠΈΠΊ ΠΏΡ€ΠΎΠ²ΠΎΠ΄ΠΈΡ‚ элСктричСский Ρ‚ΠΎΠΊ, являСтся ΠΎΠ±Ρ€Π°Ρ‚Π½ΠΎΠΉ ΡΠΎΠΏΡ€ΠΎΡ‚ΠΈΠ²Π»Π΅Π½ΠΈΡŽ: G = 1/R.

ΠŸΡ€ΠΎΠ²ΠΎΠ΄ΠΈΠΌΠΎΡΡ‚ΡŒ обозначаСтся Π±ΡƒΠΊΠ²Π°ΠΌΠΈ G (g) латинского Π°Π»Ρ„Π°Π²ΠΈΡ‚Π° ΠΈ измСряСтся Π² симСнсах (Π‘ΠΌ). Π’Π°ΠΊ ΠΆΠ΅ ΠΊΠ°ΠΊ ΠΈ сопротивлСниС ΠΏΡ€ΠΎΠ²ΠΎΠ΄ΠΈΠΌΠΎΡΡ‚ΡŒ — скалярная Π²Π΅Π»ΠΈΡ‡ΠΈΠ½Π°.

УдСльноС сопротивлСниС:

ЀизичСская Π²Π΅Π»ΠΈΡ‡ΠΈΠ½Π°, числСнно равная ΡΠΎΠΏΡ€ΠΎΡ‚ΠΈΠ²Π»Π΅Π½ΠΈΡŽ участка элСктричСской Ρ†Π΅ΠΏΠΈ, Π²Ρ‹ΠΏΠΎΠ»Π½Π΅Π½Π½ΠΎΠ³ΠΎ ΠΈΠ· Π΄Π°Π½Π½ΠΎΠ³ΠΎ вСщСства, Π΄Π»ΠΈΠ½ΠΎΠΉ 1 ΠΌ ΠΈ ΠΏΠ»ΠΎΡ‰Π°Π΄ΡŒΡŽ ΠΏΠΎΠΏΠ΅Ρ€Π΅Ρ‡Π½ΠΎΠ³ΠΎ сСчСния 1 ΠΌ2.

УдСльная ΠΏΡ€ΠΎΠ²ΠΎΠ΄ΠΈΠΌΠΎΡΡ‚ΡŒ обозначаСтся Π±ΡƒΠΊΠ²Π°ΠΌΠΈ ρ грСчСского Π°Π»Ρ„Π°Π²ΠΈΡ‚Π° ΠΈ измСряСтся Π² ΠΎΠΌΠ°Ρ… Π½Π° ΠΌΠ΅Ρ‚Ρ€ (ΠžΠΌΓ—ΠΌ). ЯвляСтся скалярной Π²Π΅Π»ΠΈΡ‡ΠΈΠ½ΠΎΠΉ. [3].

Π’ дальнСйшСм ΠΏΡ€ΠΈ использовании Π²Ρ‹ΡˆΠ΅ΠΏΠ΅Ρ€Π΅Ρ‡ΠΈΡΠ»Π΅Π½Π½Ρ‹Ρ… Ρ‚Π΅Ρ€ΠΌΠΈΠ½ΠΎΠ² слово «ΡΠ»Π΅ΠΊΡ‚Ρ€ΠΈΡ‡Π΅ΡΠΊΠΈΠΉ» Π±ΡƒΠ΄Π΅Ρ‚ ΡƒΠΏΡƒΡΠΊΠ°Ρ‚ΡŒΡΡ.

Бписок ΠΈΡΠΏΠΎΠ»ΡŒΠ·ΠΎΠ²Π°Π½Π½Ρ‹Ρ… источников

  1. БСссонов, Π›.А. ВСорСтичСскиС основы элСктротСхники: ΡƒΡ‡Π΅Π±Π½ΠΈΠΊ / Π›.А. БСссонов — Москва: Π’Ρ‹ΡΡˆΠ°Ρ школа, 1996. — 623 с.
  2. Иванова, Π‘.Π“. ВСорСтичСскиС основы элСктротСхники: ВСрсия 1.0 [Π­Π»Π΅ΠΊΡ‚Ρ€ΠΎΠ½Π½Ρ‹ΠΉ рСсурс] : ΡƒΡ‡Π΅Π±. пособиС / Π‘. Π“. Иванова, Π’. Π’. Новиков – ΠšΡ€Π°ΡΠ½ΠΎΡΡ€ΡΠΊ: ИПК Π‘Π€Π£, 2008. — 318 с.
  3. ВикипСдия — УдСльноС элСктричСскоС сопротивлСниС [элСктронный рСсурс] — Π Π΅ΠΆΠΈΠΌ доступа: https://ru.wikipedia.org/wiki/УдСльноС_элСктричСскоС_сопротивлСниС
  4. ВикипСдия — ΠŸΠ»ΠΎΡ‚Π½ΠΎΡΡ‚ΡŒ Ρ‚ΠΎΠΊΠ° [элСктронный рСсурс] — Π Π΅ΠΆΠΈΠΌ доступа: https://ru.wikipedia.org/wiki/ΠŸΠ»ΠΎΡ‚Π½ΠΎΡΡ‚ΡŒ_Ρ‚ΠΎΠΊΠ°

Π’Ρ€ΠΈ Π·Π°ΠΊΠΎΠ½Π° Ома

НСдавно ΠΌΡ‹ выпустили ΠΏΠ΅Ρ€Π΅ΠΈΠ·Π΄Π°Π½ΠΈΠ΅ ΠΊΠ½ΠΈΠ³ΠΈ Ѐрэнка Π’ΠΈΠ»ΡŒΡ‡Π΅ΠΊΠ° (Frank Wilczek) «Вонкая Ρ„ΠΈΠ·ΠΈΠΊΠ°. Масса, эфир и объСдинСниС всСмирных сил» Π² мягком ΠΏΠ΅Ρ€Π΅ΠΏΠ»Π΅Ρ‚Π΅. Автор, Π»Π°ΡƒΡ€Π΅Π°Ρ‚ НобСлСвской ΠΏΡ€Π΅ΠΌΠΈΠΈ ΠΏΠΎ Ρ„ΠΈΠ·ΠΈΠΊΠ΅, ΠΈΠ·Π»Π°Π³Π°Π΅Ρ‚ соврСмСнныС взгляды Π½Π°Β Π½Π°ΡˆΡƒ Π½Π΅Π²Π΅Ρ€ΠΎΡΡ‚Π½ΡƒΡŽ Π’ΡΠ΅Π»Π΅Π½Π½ΡƒΡŽ ΠΈΒ ΠΏΡ€ΠΎΠ³Π½ΠΎΠ·ΠΈΡ€ΡƒΠ΅Ρ‚ Π½ΠΎΠ²Ρ‹ΠΉ Π·ΠΎΠ»ΠΎΡ‚ΠΎΠΉ Π²Π΅ΠΊ Ρ„ΡƒΠ½Π΄Π°ΠΌΠ΅Π½Ρ‚Π°Π»ΡŒΠ½ΠΎΠΉ физичСской Π½Π°ΡƒΠΊΠΈ. Π₯Π₯2 ΡƒΠΆΠ΅ ΠΏΡƒΠ±Π»ΠΈΠΊΠΎΠ²Π°Π»ΠΈΒ ΠΎΠ΄ΠΈΠ½ ΠΎΡ‚Ρ€Ρ‹Π²ΠΎΠΊΒ ΠΈΠ· этой Π·Π°ΠΌΠ΅Ρ‡Π°Ρ‚Π΅Π»ΡŒΠ½ΠΎΠΉ ΠΊΠ½ΠΈΠ³ΠΈ, Ρ‚Π΅ΠΏΠ΅Ρ€ΡŒ ΠΏΡƒΠ±Π»ΠΈΠΊΡƒΡŽΡ‚ Π½ΠΎΠ²Ρ‹ΠΉ β€” ΠΎΒ Ρ‚ΠΎΠΌ, ΠΏΠΎΡ‡Π΅ΠΌΡƒ эквивалСнтныС равСнства ΠΌΠΎΠ³ΡƒΡ‚ ΠΏΠΎΠ²Π΅Π΄Π°Ρ‚ΡŒ Π½Π°ΠΌ ΠΎΒ Ρ€Π°Π·Π½Ρ‹Ρ… явлСниях.

Из Ρ‚Ρ€Π΅Ρ‚ΡŒΠ΅ΠΉ Π³Π»Π°Π²Ρ‹. Π’Ρ‚ΠΎΡ€ΠΎΠΉ Π·Π°ΠΊΠΎΠ½ Ома

Π’Ρ‚ΠΎΡ€ΠΎΠΉ Π·Π°ΠΊΠΎΠ½ Π­ΠΉΠ½ΡˆΡ‚Π΅ΠΉΠ½Π°, m = E/c2, ΠΏΠΎΠ΄Π½ΠΈΠΌΠ°Π΅Ρ‚ вопрос ΠΎΒ Ρ‚ΠΎΠΌ, ΠΌΠΎΠΆΠ΅Ρ‚ Π»ΠΈ масса Π±Ρ‹Ρ‚ΡŒ понята Π±ΠΎΠ»Π΅Π΅ Π³Π»ΡƒΠ±ΠΎΠΊΠΎΒ β€” ΠΊΠ°ΠΊ энСргия. МоТСм Π»ΠΈ ΠΌΡ‹ ΡΠΎΠ·Π΄Π°Ρ‚ΡŒ, ΠΊΠ°ΠΊ выразился Π£ΠΈΠ»Π΅Ρ€, «массу Π±Π΅Π· массы»?

Когда я Π΅Ρ‰Ρ‘ Ρ‚ΠΎΠ»ΡŒΠΊΠΎ собирался Π½Π°Ρ‡Π°Ρ‚ΡŒ ΠΏΡ€Π΅ΠΏΠΎΠ΄Π°Π²Π°Ρ‚ΡŒ Π²Β ΠŸΡ€ΠΈΠ½ΡΡ‚ΠΎΠ½Π΅, ΠΌΠΎΠΉ Π΄Ρ€ΡƒΠ³ и наставник Бэм Π’Ρ€Π΅ΠΉΠΌΠ°Π½ ΠΏΠΎΠ·Π²Π°Π» мСня в свой ΠΊΠ°Π±ΠΈΠ½Π΅Ρ‚. Он Ρ…ΠΎΡ‚Π΅Π» ΠΏΠΎΠ΄Π΅Π»ΠΈΡ‚ΡŒΡΡ со ΠΌΠ½ΠΎΠΉ своСй ΠΌΡƒΠ΄Ρ€ΠΎΡΡ‚ΡŒΡŽ. Бэм Π²Ρ‹Ρ‚Π°Ρ‰ΠΈΠ» ΠΈΠ· ящика стола ΠΏΠΎΡ‚Ρ€Ρ‘ΠΏΠ°Π½Π½ΠΎΠ΅ руководство в мягкой ΠΎΠ±Π»ΠΎΠΆΠΊΠ΅ и сказал ΠΌΠ½Π΅: Β«Π’ΠΎ врСмя Π’Ρ‚ΠΎΡ€ΠΎΠΉ ΠΌΠΈΡ€ΠΎΠ²ΠΎΠΉ Π²ΠΎΠΉΠ½Ρ‹ Π’ΠœΠ‘ ΠΏΡ€ΠΈΡ…ΠΎΠ΄ΠΈΠ»ΠΎΡΡŒ в спСшкС ΠΎΠ±ΡƒΡ‡Π°Ρ‚ΡŒ Π½ΠΎΠ²ΠΎΠ±Ρ€Π°Π½Ρ†Π΅Π² налаТиванию и использованию радиосвязи. МногиС ΠΈΠ· этих Π½ΠΎΠ²ΠΎΠ±Ρ€Π°Π½Ρ†Π΅Π² ΠΏΡ€ΠΈΠ±Ρ‹Π²Π°Π»ΠΈ прямо с фСрм, Ρ‚Π°ΠΊ Ρ‡Ρ‚ΠΎ быстро ввСсти ΠΈΡ… в курс Π΄Π΅Π»Π° Π±Ρ‹Π»ΠΎ ΠΎΡ‡Π΅Π½ΡŒ Ρ‚Ρ€ΡƒΠ΄Π½ΠΎ. Π‘Β ΠΏΠΎΠΌΠΎΡ‰ΡŒΡŽ Ρ‚ΠΎΠΉ Π²Π΅Π»ΠΈΠΊΠΎΠ»Π΅ΠΏΠ½ΠΎΠΉ ΠΊΠ½ΠΈΠ³ΠΈ командованию Π²ΠΎΠ΅Π½Π½ΠΎ-морского Ρ„Π»ΠΎΡ‚Π° это ΡƒΠ΄Π°Π»ΠΎΡΡŒ. Π­Ρ‚ΠΎ ΡˆΠ΅Π΄Π΅Π²Ρ€ ΠΏΠ΅Π΄Π°Π³ΠΎΠ³ΠΈΠΊΠΈ. ОсобСнно пСрвая Π³Π»Π°Π²Π°. Взгляни».

Он Π²Ρ€ΡƒΡ‡ΠΈΠ» ΠΌΠ½Π΅ ΠΊΠ½ΠΈΠ³Ρƒ, ΠΎΡ‚ΠΊΡ€Ρ‹Ρ‚ΡƒΡŽ Π½Π°Β ΠΏΠ΅Ρ€Π²ΠΎΠΉ Π³Π»Π°Π²Π΅. Она Π½Π°Π·Ρ‹Π²Π°Π»Π°ΡΡŒ Β«Π’Ρ€ΠΈ Π·Π°ΠΊΠΎΠ½Π° Ома». Π― Π±Ρ‹Π» Π·Π½Π°ΠΊΠΎΠΌ с одним Π·Π°ΠΊΠΎΠ½ΠΎΠΌ Ома, извСстным ΡΠΎΠΎΡ‚Π½ΠΎΡˆΠ΅Π½ΠΈΠ΅ΠΌ V = IR, ΠΊΠΎΡ‚ΠΎΡ€Ρ‹ΠΉ связываСт напряТСниС (V), силу Ρ‚ΠΎΠΊΠ° (I) и сопротивлСниС (R) в элСктричСской Ρ†Π΅ΠΏΠΈ.

Π­Ρ‚ΠΎ оказалось ΠΏΠ΅Ρ€Π²Ρ‹ΠΌ Π·Π°ΠΊΠΎΠ½ΠΎΠΌ Ома.

МнС Π±Ρ‹Π»ΠΎ ΠΎΡ‡Π΅Π½ΡŒ интСрСсно ΡƒΠ·Π½Π°Ρ‚ΡŒ, ΠΊΠ°ΠΊΠΎΠ²Ρ‹ Π΄Π²Π° Π΄Ρ€ΡƒΠ³ΠΈΡ… Π·Π°ΠΊΠΎΠ½Π° Ома. ΠŸΠ΅Ρ€Π΅Π²Π΅Ρ€Π½ΡƒΠ² нСсколько Ρ…Ρ€ΡƒΠΏΠΊΠΈΡ… ΠΏΠΎΠΆΠ΅Π»Ρ‚Π΅Π²ΡˆΠΈΡ… страниц, я ΠΎΠ±Π½Π°Ρ€ΡƒΠΆΠΈΠ» Π²Ρ‚ΠΎΡ€ΠΎΠΉ Π·Π°ΠΊΠΎΠ½ Ома: I = V/R. Π― ΠΏΡ€Π΅Π΄ΠΏΠΎΠ»ΠΎΠΆΠΈΠ», Ρ‡Ρ‚ΠΎ Ρ‚Ρ€Π΅Ρ‚ΠΈΠΉ Π·Π°ΠΊΠΎΠ½ Ома формулируСтся ΠΊΠ°ΠΊ R = I/V, и оказался ΠΏΡ€Π°Π².

ΠžΡ‚ΠΊΡ€Ρ‹Π²Π°Ρ‚ΡŒ Π½ΠΎΠ²Ρ‹Π΅ Π·Π°ΠΊΠΎΠ½Ρ‹ Π»Π΅Π³ΠΊΠΎ

Π’Π΅ΠΌ, ΠΊΡ‚ΠΎ Π·Π½Π°ΠΊΠΎΠΌ с элСмСнтарной Π°Π»Π³Π΅Π±Ρ€ΠΎΠΉ, Ρ‚Π°ΠΊ ΠΎΡ‡Π΅Π²ΠΈΠ΄Π½ΠΎ, Ρ‡Ρ‚ΠΎ эти Ρ‚Ρ€ΠΈ Π·Π°ΠΊΠΎΠ½Π° эквивалСнтны Π΄Ρ€ΡƒΠ³ Π΄Ρ€ΡƒΠ³Ρƒ, Ρ‡Ρ‚ΠΎ данная история воспринимаСтся ΠΊΠ°ΠΊ ΡˆΡƒΡ‚ΠΊΠ°. Однако Π²Β Π½Π΅ΠΉ Π·Π°ΠΊΠ»ΡŽΡ‡Ρ‘Π½ Π³Π»ΡƒΠ±ΠΎΠΊΠΈΠΉ смысл. (ΠšΡ€ΠΎΠΌΠ΅ Ρ‚ΠΎΠ³ΠΎ, Π²Β Π½Π΅ΠΉ Π΅ΡΡ‚ΡŒ ΠΈΒ Π½Π΅Π³Π»ΡƒΠ±ΠΎΠΊΠΈΠΉ смысл, ΠΊΠΎΡ‚ΠΎΡ€Ρ‹ΠΉ, ΠΊΠ°ΠΊ ΠΌΠ½Π΅ каТСтся, Бэм Ρ…ΠΎΡ‚Π΅Π» Π΄ΠΎ мСня донСсти. ΠŸΡ€ΠΈ ΠΎΠ±ΡƒΡ‡Π΅Π½ΠΈΠΈ Π½Π°Ρ‡ΠΈΠ½Π°ΡŽΡ‰ΠΈΡ… Π²Ρ‹ Π΄ΠΎΠ»ΠΆΠ½Ρ‹ нСсколько Ρ€Π°Π· ΡΠΊΠ°Π·Π°Ρ‚ΡŒ ΠΎΠ΄Π½ΠΎ ΠΈΒ Ρ‚ΠΎ ΠΆΠ΅, Π½ΠΎ ΠΏΠΎ-Ρ€Π°Π·Π½ΠΎΠΌΡƒ. Π‘ΠΎΠΎΡ‚Π½ΠΎΡˆΠ΅Π½ΠΈΡ, ΠΊΠΎΡ‚ΠΎΡ€Ρ‹Π΅ бСсспорны для профСссионала, ΠΌΠΎΠ³ΡƒΡ‚ Π½Π΅Β Π±Ρ‹Ρ‚ΡŒ Ρ‚Π°ΠΊΠΎΠ²Ρ‹ΠΌΠΈ для Π½ΠΎΠ²ΠΈΡ‡ΠΊΠ°. Π‘Ρ‚ΡƒΠ΄Π΅Π½Ρ‚Ρ‹ Π½Π΅Β Π±ΡƒΠ΄ΡƒΡ‚ Π²ΠΎΠ·Ρ€Π°ΠΆΠ°Ρ‚ΡŒ ΠΏΡ€ΠΎΡ‚ΠΈΠ² объяснСния ΠΎΡ‡Π΅Π²ΠΈΠ΄Π½ΠΎΠ³ΠΎ. ΠžΡ‡Π΅Π½ΡŒ Π½Π΅ΠΌΠ½ΠΎΠ³ΠΈΠ΅ люди ΠΎΠ±ΠΈΠΆΠ°ΡŽΡ‚ΡΡ, ΠΊΠΎΠ³Π΄Π° Π²Ρ‹ позволяСтС ΠΈΠΌ ΠΏΠΎΡ‡ΡƒΠ²ΡΡ‚Π²ΠΎΠ²Π°Ρ‚ΡŒ сСбя ΡƒΠΌΠ½Ρ‹ΠΌΠΈ.)

Π“Π»ΡƒΠ±ΠΎΠΊΠΈΠΉ смысл содСрТит заявлСниС Π²Π΅Π»ΠΈΠΊΠΎΠ³ΠΎ Ρ„ΠΈΠ·ΠΈΠΊΠ°-Ρ‚Π΅ΠΎΡ€Π΅Ρ‚ΠΈΠΊΠ° Поля Π”ΠΈΡ€Π°ΠΊΠ°. Когда Π΅Π³ΠΎ спросили, ΠΊΠ°ΠΊ ΠΎΠ½ ΠΎΡ‚ΠΊΡ€Ρ‹Π²Π°Π΅Ρ‚ Π½ΠΎΠ²Ρ‹Π΅ Π·Π°ΠΊΠΎΠ½Ρ‹ ΠΏΡ€ΠΈΡ€ΠΎΠ΄Ρ‹, Π”ΠΈΡ€Π°ΠΊ ΠΎΡ‚Π²Π΅Ρ‚ΠΈΠ»: Β«Π― ΠΈΠ³Ρ€Π°ΡŽ с уравнСниями». Π‘ΡƒΡ‚ΡŒ Π²Β Ρ‚ΠΎΠΌ, Ρ‡Ρ‚ΠΎ Ρ€Π°Π·Π»ΠΈΡ‡Π½Ρ‹Π΅ способы написания ΠΎΠ΄Π½ΠΎΠ³ΠΎ ΠΈΒ Ρ‚ΠΎΠ³ΠΎ ΠΆΠ΅ уравнСния ΠΌΠΎΠ³ΡƒΡ‚ Π³ΠΎΠ²ΠΎΡ€ΠΈΡ‚ΡŒ ΠΎΒ ΡΠΎΠ²Π΅Ρ€ΡˆΠ΅Π½Π½ΠΎ Ρ€Π°Π·Π½Ρ‹Ρ… Π²Π΅Ρ‰Π°Ρ…, Π΄Π°ΠΆΠ΅ Ссли ΠΎΠ½ΠΈ ΡΠ²Π»ΡΡŽΡ‚ΡΡ логичСски эквивалСнтными.

Π’Ρ‚ΠΎΡ€ΠΎΠΉ Π·Π°ΠΊΠΎΠ½ Π­ΠΉΠ½ΡˆΡ‚Π΅ΠΉΠ½Π°

Π’Ρ‚ΠΎΡ€ΠΎΠΉ Π·Π°ΠΊΠΎΠ½ Π­ΠΉΠ½ΡˆΡ‚Π΅ΠΉΠ½Π° формулируСтся ΡΠ»Π΅Π΄ΡƒΡŽΡ‰ΠΈΠΌ ΠΎΠ±Ρ€Π°Π·ΠΎΠΌ:

m = E/c2.

ΠŸΠ΅Ρ€Π²Ρ‹ΠΉ Π·Π°ΠΊΠΎΠ½ Π­ΠΉΠ½ΡˆΡ‚Π΅ΠΉΠ½Π°Β β€” это, разумССтся, E = mc2. Π—Π΄ΠΎΡ€ΠΎΠ²ΠΎ, Ρ‡Ρ‚ΠΎ ΠΏΠ΅Ρ€Π²Ρ‹ΠΉ Π·Π°ΠΊΠΎΠ½ ΠΏΡ€Π΅Π΄ΠΏΠΎΠ»Π°Π³Π°Π΅Ρ‚ Π²ΠΎΠ·ΠΌΠΎΠΆΠ½ΠΎΡΡ‚ΡŒ получСния большого количСства энСргии ΠΈΠ· нСбольшого количСства массы. Он Π½Π°Π²ΠΎΠ΄ΠΈΡ‚ Π½Π°Β ΠΌΡ‹ΡΠ»ΡŒ о ядСрных Ρ€Π΅Π°ΠΊΡ‚ΠΎΡ€Π°Ρ… и ядСрных Π±ΠΎΠΌΠ±Π°Ρ….

Π’Ρ‚ΠΎΡ€ΠΎΠΉ Π·Π°ΠΊΠΎΠ½ Π­ΠΉΠ½ΡˆΡ‚Π΅ΠΉΠ½Π° ΠΏΡ€Π΅Π΄ΠΏΠΎΠ»Π°Π³Π°Π΅Ρ‚ Π½Π΅Ρ‡Ρ‚ΠΎ ΡΠΎΠ²Π΅Ρ€ΡˆΠ΅Π½Π½ΠΎ ΠΈΠ½ΠΎΠ΅.

Он ΠΏΡ€Π΅Π΄ΠΏΠΎΠ»Π°Π³Π°Π΅Ρ‚ Π²ΠΎΠ·ΠΌΠΎΠΆΠ½ΠΎΡΡ‚ΡŒ объяснСния Ρ‚ΠΎΠ³ΠΎ, ΠΊΠ°ΠΊ масса Π²ΠΎΠ·Π½ΠΈΠΊΠ°Π΅Ρ‚ ΠΈΠ· энСргии.

На самом Π΄Π΅Π»Π΅ этот Π·Π°ΠΊΠΎΠ½ Π½Π΅ΠΏΡ€Π°Π²ΠΈΠ»ΡŒΠ½ΠΎ Π½Π°Π·Ρ‹Π²Π°Ρ‚ΡŒ Β«Π²Ρ‚ΠΎΡ€Ρ‹ΠΌΒ».

Π’ ΠΎΡ€ΠΈΠ³ΠΈΠ½Π°Π»ΡŒΠ½ΠΎΠΉ Ρ€Π°Π±ΠΎΡ‚Π΅ Π­ΠΉΠ½ΡˆΡ‚Π΅ΠΉΠ½Π° 1905 Π³ΠΎΠ΄Π° Π²Ρ‹ Π½Π΅Β Π½Π°ΠΉΠ΄Ρ‘Ρ‚Π΅ уравнСния E = mc2. Π’Ρ‹ встрСтитС ΡƒΡ€Π°Π²Π½Π΅Π½ΠΈΠ΅ m = E/c2. (ΠŸΠΎΡΡ‚ΠΎΠΌΡƒ, Π²ΠΎΠ·ΠΌΠΎΠΆΠ½ΠΎ, Π½Π°ΠΌ слСдуСт Π½Π°Π·Π²Π°Ρ‚ΡŒ Π΅Π³ΠΎ Π½ΡƒΠ»Π΅Π²Ρ‹ΠΌ Π·Π°ΠΊΠΎΠ½ΠΎΠΌ Π­ΠΉΠ½ΡˆΡ‚Π΅ΠΉΠ½Π°.)

На самом Π΄Π΅Π»Π΅ в качСствС названия этой ΡΡ‚Π°Ρ‚ΡŒΠΈ ΠΈΡΠΏΠΎΠ»ΡŒΠ·ΡƒΠ΅Ρ‚ΡΡ вопрос: «Зависит Π»ΠΈ инСрция Ρ‚Π΅Π»Π° ΠΎΡ‚ содСрТащСйся Π²Β Π½Π΅ΠΌ энСргии?Β»

Π”Ρ€ΡƒΠ³ΠΈΠΌΠΈ словами, ΠΌΠΎΠΆΠ΅Ρ‚ Π»ΠΈ Π½Π΅ΠΊΠΎΡ‚ΠΎΡ€ΠΎΠ΅ количСство массы Ρ‚Π΅Π»Π° Π²ΠΎΠ·Π½ΠΈΠΊΠ°Ρ‚ΡŒ ΠΈΠ· энСргии содСрТащСгося Π²Β Π½Π΅ΠΌ вСщСства? Б самого Π½Π°Ρ‡Π°Π»Π° Π­ΠΉΠ½ΡˆΡ‚Π΅ΠΉΠ½ Ρ€Π°Π·ΠΌΡ‹ΡˆΠ»ΡΠ» ΠΎΒ ΠΊΠΎΠ½Ρ†Π΅ΠΏΡ‚ΡƒΠ°Π»ΡŒΠ½Ρ‹Ρ… основах Ρ„ΠΈΠ·ΠΈΠΊΠΈ, Π°Β Π½Π΅ о возмоТности создания Π±ΠΎΠΌΠ± ΠΈΠ»ΠΈ Ρ€Π΅Π°ΠΊΡ‚ΠΎΡ€ΠΎΠ².

ΠŸΠΎΠ½ΡΡ‚ΠΈΠ΅ энСргии ΠΈΠ³Ρ€Π°Π΅Ρ‚ Π³ΠΎΡ€Π°Π·Π΄ΠΎ Π±ΠΎΠ»Π΅Π΅ Π²Π°ΠΆΠ½ΡƒΡŽ Ρ€ΠΎΠ»ΡŒ в соврСмСнной Ρ„ΠΈΠ·ΠΈΠΊΠ΅, Ρ‡Π΅ΠΌ понятиС массы. Π­Ρ‚ΠΎ проявляСтся Π²ΠΎ ΠΌΠ½ΠΎΠ³ΠΈΡ… ΠΎΡ‚Π½ΠΎΡˆΠ΅Π½ΠΈΡΡ…. БохраняСтся ΠΈΠΌΠ΅Π½Π½ΠΎ энСргия, Π°Β Π½Π΅ масса. ИмСнно энСргия Ρ„ΠΈΠ³ΡƒΡ€ΠΈΡ€ΡƒΠ΅Ρ‚ Π²Β Ρ‚Π°ΠΊΠΈΡ… Ρ„ΡƒΠ½Π΄Π°ΠΌΠ΅Π½Ρ‚Π°Π»ΡŒΠ½Ρ‹Ρ… уравнСниях, ΠΊΠ°ΠΊ ΡƒΡ€Π°Π²Π½Π΅Π½ΠΈΠ΅ Π‘ΠΎΠ»ΡŒΡ†ΠΌΠ°Π½Π° для статистичСской ΠΌΠ΅Ρ…Π°Π½ΠΈΠΊΠΈ, уравнСния Π¨Ρ€Ρ‘Π΄ΠΈΠ½Π³Π΅Ρ€Π° для ΠΊΠ²Π°Π½Ρ‚ΠΎΠ²ΠΎΠΉ ΠΌΠ΅Ρ…Π°Π½ΠΈΠΊΠΈ ΠΈΒ ΡƒΡ€Π°Π²Π½Π΅Π½ΠΈΠ΅ Π­ΠΉΠ½ΡˆΡ‚Π΅ΠΉΠ½Π° для Π³Ρ€Π°Π²ΠΈΡ‚Π°Ρ†ΠΈΠΈ. Масса Π²Β Π±ΠΎΠ»Π΅Π΅ тСхничСском смыслС проявляСтся в качСствС ΠΌΠ΅Ρ‚ΠΊΠΈ для Π½Π΅ΠΏΡ€ΠΈΠ²ΠΎΠ΄ΠΈΠΌΡ‹Ρ… прСдставлСний Π³Ρ€ΡƒΠΏΠΏΡ‹ ΠŸΡƒΠ°Π½ΠΊΠ°Ρ€Π΅.

(Π― Π΄Π°ΠΆΠ΅ Π½Π΅Β Π±ΡƒΠ΄Ρƒ ΠΏΡ‹Ρ‚Π°Ρ‚ΡŒΡΡ ΠΎΠ±ΡŠΡΡΠ½ΠΈΡ‚ΡŒ, Ρ‡Ρ‚ΠΎ ΠΎΠ·Π½Π°Ρ‡Π°Π΅Ρ‚ ΠΏΡ€Π΅Π΄Ρ‹Π΄ΡƒΡ‰Π΅Π΅ ΡƒΡ‚Π²Π΅Ρ€ΠΆΠ΄Π΅Π½ΠΈΠ΅, ΠΊΒ ΡΡ‡Π°ΡΡ‚ΡŒΡŽ, ΡΡƒΡ‚ΡŒ Π·Π°ΠΊΠ»ΡŽΡ‡Π°Π΅Ρ‚ΡΡ в самом Ρ„Π°ΠΊΡ‚Π΅ утвСрТдСния.)

Π’Π°ΠΊΠΈΠΌ ΠΎΠ±Ρ€Π°Π·ΠΎΠΌ, вопрос Π­ΠΉΠ½ΡˆΡ‚Π΅ΠΉΠ½Π° бросаСт Π²Ρ‹Π·ΠΎΠ². Если ΠΌΡ‹ смоТСм ΠΎΠ±ΡŠΡΡΠ½ΠΈΡ‚ΡŒ массу Π²Β Ρ‚Π΅Ρ€ΠΌΠΈΠ½Π°Ρ… энСргии, ΠΌΡ‹ ΡƒΠ»ΡƒΡ‡ΡˆΠΈΠΌ нашС описаниС ΠΌΠΈΡ€Π°. В этом случаС в нашСм Ρ€Π΅Ρ†Π΅ΠΏΡ‚Π΅ Π½Π°ΠΌ потрСбуСтся мСньшСС количСство ΠΈΠ½Π³Ρ€Π΅Π΄ΠΈΠ΅Π½Ρ‚ΠΎΠ². Π’Ρ‚ΠΎΡ€ΠΎΠΉ Π·Π°ΠΊΠΎΠ½ Π­ΠΉΠ½ΡˆΡ‚Π΅ΠΉΠ½Π° позволяСт Π΄Π°Ρ‚ΡŒ Ρ…ΠΎΡ€ΠΎΡˆΠΈΠΉ ΠΎΡ‚Π²Π΅Ρ‚ на вопрос, ΠΊΠΎΡ‚ΠΎΡ€Ρ‹ΠΉ ΠΌΡ‹ Π·Π°Π΄Π°Π»ΠΈ Ρ€Π°Π½Π΅Π΅. ΠžΡ‚ΠΊΡƒΠ΄Π° бСрётся масса? ΠœΠΎΠΆΠ΅Ρ‚ Π±Ρ‹Ρ‚ΡŒ, ΠΈΠ· энСргии. На самом Π΄Π΅Π»Π΅, ΠΊΠ°ΠΊ ΠΌΡ‹ ΡƒΠ²ΠΈΠ΄ΠΈΠΌ Π΄Π°Π»Π΅Π΅, в основном Ρ‚Π°ΠΊ ΠΈΒ Π΅ΡΡ‚ΡŒ.

Π˜ΡΡ‚ΠΎΡ‡Π½ΠΈΠΊ:Β https://22century.ru/popular-science-publications/tonkaya-fizika-massa-efir-i-obedinenie-vsemirnyh-sil?

ΠžΠ±ΠΎΠ±Ρ‰Π΅Π½Π½Ρ‹ΠΉ Π·Π°ΠΊΠΎΠ½ Ома

Π‘ΠΎΠ΄Π΅Ρ€ΠΆΠ°Π½ΠΈΠ΅:

ΠžΠ±ΠΎΠ±Ρ‰Π΅Π½Π½Ρ‹ΠΉ Π·Π°ΠΊΠΎΠ½ Ома

Π—Π°ΠΊΠΎΠ½ Ома, Π²Ρ‹Ρ€Π°ΠΆΠ΅Π½Π½Ρ‹ΠΉ Π² Π²ΠΈΠ΄Π΅ уравнСния, опрСдСляСт ΡΠΎΠΎΡ‚Π½ΠΎΡˆΠ΅Π½ΠΈΠ΅ ΠΌΠ΅ΠΆΠ΄Ρƒ Ρ‚ΠΎΠΊΠΎΠΌ ΠΈ напряТСниСм Π² пассивной части элСктричСской Ρ†Π΅ΠΏΠΈ.

На Π°ΠΊΡ‚ΠΈΠ²Π½ΠΎΠΉ Ρ†Π΅ΠΏΠΈ, ΠΊΠΎΡ‚ΠΎΡ€Ρ‹ΠΉ опрСдСляСт Ρ‚ΠΎΠΊ, напряТСниС ΠΈ ΠΎΡ‚Π½ΠΎΡˆΠ΅Π½ΠΈΡ Π­Π”Π‘ ΠΈΠ· уравнСния. ΠΏΠ»ΠΎΡ‰Π°Π΄ΡŒ ΠΏΠΎΠ»ΠΎΠΆΠΈΡ‚Π΅Π»ΡŒΠ½ΠΎΠ³ΠΎ напряТСния.

Π­Ρ‚Π° Ρ„ΠΎΡ€ΠΌΡƒΠ»Π° прСдставляСт собой ΠΎΠ±ΠΎΠ±Ρ‰Π΅Π½Π½Ρ‹ΠΉ Π·Π°ΠΊΠΎΠ½ Ома ΠΈΠ»ΠΈ Π·Π°ΠΊΠΎΠ½ Ома для Ρ†Π΅ΠΏΠΈ, которая содСрТат Π­Π”Π‘.

Π£Ρ€Π°Π²Π½Π΅Π½ΠΈΠ΅ Π²Ρ…ΠΎΠ΄ΠΈΡ‚ Π² Π·Π°ΠΊΠΎΠ½ Ома с Ρ‚Π΅ΠΌ ΠΆΠ΅ символом, ΠΊΠΎΠ³Π΄Π° направлСния Ρ‚ΠΎΠΊΠ°, напряТСния ΠΈ Π­Π”Π‘ ΡΠΎΠ²ΠΏΠ°Π΄Π°ΡŽΡ‚. Π’ случаС Π­Π”Π‘ ΠΏΡ€ΠΈ дСйствии Π² Π½Π°ΠΏΡ€Π°Π²Π»Π΅Π½ΠΈΠΈ, ΠΏΡ€ΠΎΡ‚ΠΈΠ²ΠΎΠΏΠΎΠ»ΠΎΠΆΠ½ΠΎΠΌ ΠΏΠΎΠ»ΠΎΠΆΠΈΡ‚Π΅Π»ΡŒΠ½ΠΎΠΌΡƒ Π½Π°ΠΏΡ€Π°Π²Π»Π΅Π½ΠΈΡŽ Ρ‚ΠΎΠΊΠ°, Π² Π²Ρ‹Ρ€Π°ΠΆΠ΅Π½ΠΈΠΈ вводится символ Β«-Β».

Π—Π°ΠΊΠΎΠ½ Ома распространяСтся Π½Π° отвСтвлСния ΠΈ ΠΎΠ΄Π½ΠΎΠΊΠΎΠ½Ρ‚ΡƒΡ€Π½Ρ‹Π΅ Π·Π°ΠΌΠΊΠ½ΡƒΡ‚Ρ‹Π΅ Ρ†Π΅ΠΏΠΈ.

ΠŸΡ€ΠΈΠΌΠ΅Ρ€ 1 создания ΠΏΠΎΡ‚Π΅Π½Ρ†ΠΈΠ°Π»ΡŒΠ½ΠΎΠ³ΠΎ Π³Ρ€Π°Ρ„ΠΈΠΊΠ°:

Π‘ΠΎΠ·Π΄Π°ΠΉΡ‚Π΅ Π΄ΠΈΠ°Π³Ρ€Π°ΠΌΠΌΡƒ ΠΏΠΎΡ‚Π΅Π½Ρ†ΠΈΠ°Π»Π° ΠΎΠ΄Π½ΠΎΠΉ Ρ†Π΅ΠΏΠΈ.

РСшСниС: ΠŸΠ΅Ρ€Π΅Ρ€ΠΈΡΠΎΠ²Π°Ρ‚ΡŒ ΡƒΠΊΠ°Π·Π°Π½Π½Ρ‹ΠΉ ΠΊΠΎΠ½Ρ‚ΡƒΡ€ ΠΈ вывСсти Π²Π½ΡƒΡ‚Ρ€Π΅Π½Π½Π΅Π΅ сопротивлСниС Π­Π”Π‘ Ρ‡Π΅Ρ€Π΅Π· Π³Ρ€Π°Π½ΠΈΡ†Ρƒ. Π£ΠΊΠ°Π·Ρ‹Π²Π°Π΅Ρ‚ Ρ‚ΠΎΡ‡ΠΊΠΈ ΠΊΠΎΠ½Ρ‚ΡƒΡ€Π°.

  1. Π’Ρ‹Π±Π΅Ρ€ΠΈΡ‚Π΅ ΠΏΠΎΠ»ΠΎΠΆΠΈΡ‚Π΅Π»ΡŒΠ½ΠΎΠ΅ Π½Π°ΠΏΡ€Π°Π²Π»Π΅Π½ΠΈΠ΅ Ρ‚ΠΎΠΊΠ° I ΠΈ ΠΎΠΏΡ€Π΅Π΄Π΅Π»ΠΈΡ‚Π΅ Π΅Π³ΠΎ Π·Π½Π°Ρ‡Π΅Π½ΠΈΠ΅, ΠΈΡΠΏΠΎΠ»ΡŒΠ·ΡƒΡ ΠΎΠ±ΠΎΠ±Ρ‰Π΅Π½Π½Ρ‹ΠΉ Π·Π°ΠΊΠΎΠ½ Ома.
  2. Для Π±Π°Π·ΠΎΠ²ΠΎΠΉ Ρ‚ΠΎΡ‡ΠΊΠΈ Π²ΠΎΠ·ΡŒΠΌΠΈΡ‚Π΅ Ρ‚ΠΎΡ‡ΠΊΡƒ Π°. Найти ΠΏΠΎΡ‚Π΅Π½Ρ†ΠΈΠ°Π» ΠΎΡΡ‚Π°Π²ΡˆΠΈΡ…ΡΡ Ρ‚ΠΎΡ‡Π΅ΠΊ.

Π’ систСмС ΠΊΠΎΠΎΡ€Π΄ΠΈΠ½Π°Ρ‚ создайтС ΠΏΠΎΡ‚Π΅Π½Ρ†ΠΈΠ°Π»ΡŒΠ½ΡƒΡŽ Π΄ΠΈΠ°Π³Ρ€Π°ΠΌΠΌΡƒ.

РаспрСдСлСниС Ρ‚ΠΎΠΊΠ° ΠΏΠΎ Π²Π΅Ρ‚Π²ΠΈ элСктричСской Ρ†Π΅ΠΏΠΈ слСдуСт ΠΏΠ΅Ρ€Π²ΠΎΠΌΡƒ Π·Π°ΠΊΠΎΠ½Ρƒ, Π° распрСдСлСниС напряТСния ΠΏΠΎ части Ρ†Π΅ΠΏΠΈ β€” ΠΏΠΎ Π²Ρ‚ΠΎΡ€ΠΎΠΌΡƒ Π·Π°ΠΊΠΎΠ½Ρƒ.

Π’ соотвСтствии с Π·Π°ΠΊΠΎΠ½ΠΎΠΌ Ома, Π΅ΡΡ‚ΡŒ основа для Ρ‚Π΅ΠΎΡ€ΠΈΠΈ элСктричСских Ρ†Π΅ΠΏΠ΅ΠΉ.

  • АлгСбраичСская сумма Ρ‚ΠΎΠΊΠΎΠ² Π² ΡƒΠ·Π»Π΅ Ρ€Π°Π²Π½Π° Π½ΡƒΠ»ΡŽ:
  • Π“Π΄Π΅ я β€” число Π²Π΅Ρ‚Π²Π΅ΠΉ, ΠΊΠΎΡ‚ΠΎΡ€Ρ‹Π΅ сходятся ΠΊ ΠΊΠΎΠ½ΠΊΡ€Π΅Ρ‚Π½ΠΎΠΌΡƒ ΡƒΠ·Π»Ρƒ.
  • Π’ΠΎ Π΅ΡΡ‚ΡŒ сумма распространяСтся Π½Π° Ρ‚ΠΎΠΊ Π²Π΅Ρ‚Π²ΠΈ, ΠΊΠΎΡ‚ΠΎΡ€Ρ‹ΠΉ сходится ΠΊ рассматриваСмому ΡƒΠ·Π»Ρƒ.

ΠŸΡ€ΠΈΠΌΠ΅Ρ€Ρ‹ ΠΏΠ΅Ρ€Π²ΠΎΠ³ΠΎ Π·Π°ΠΊΠΎΠ½Π°.
ΠšΠΎΠ»ΠΈΡ‡Π΅ΡΡ‚Π²ΠΎ ΡƒΡ€Π°Π²Π½Π΅Π½ΠΈΠΉ, составлСнных ΠΏΠΎ ΠΏΠ΅Ρ€Π²ΠΎΠΌΡƒ Π·Π°ΠΊΠΎΠ½Ρƒ ΠšΠΈΡ€Ρ…Π³ΠΎΡ„Π°, опрСдСляСтся ΡΠ»Π΅Π΄ΡƒΡŽΡ‰ΠΈΠΌ ΠΎΠ±Ρ€Π°Π·ΠΎΠΌ:

ΠšΠΎΠ»ΠΈΡ‡Π΅ΡΡ‚Π²ΠΎ ΡƒΠ·Π»ΠΎΠ² Π² рассматриваСмой Ρ†Π΅ΠΏΠΎΡ‡ΠΊΠ΅.

Π—Π½Π°ΠΊ Ρ‚ΠΎΠΊΠ° Π² ΡƒΡ€Π°Π²Π½Π΅Π½ΠΈΠΈ получаСтся с ΡƒΡ‡Π΅Ρ‚ΠΎΠΌ Π²Ρ‹Π±Ρ€Π°Π½Π½ΠΎΠ³ΠΎ ΠΏΠΎΠ»ΠΎΠΆΠΈΡ‚Π΅Π»ΡŒΠ½ΠΎΠ³ΠΎ направлСния. Π—Π½Π°ΠΊ Ρ‚ΠΎΠΊΠ° ΠΎΠ΄ΠΈΠ½Π°ΠΊΠΎΠ², Ссли Ρ‚ΠΎΠΊ Π½Π°ΠΏΡ€Π°Π²Π»Π΅Π½ ΠΎΠ΄ΠΈΠ½Π°ΠΊΠΎΠ²ΠΎ Π½Π° ΠΊΠΎΠ½ΠΊΡ€Π΅Ρ‚Π½Ρ‹ΠΉ ΡƒΠ·Π΅Π».

НапримСр, для ΡƒΠ·Π»Π°, ΠΏΠΎΠΊΠ°Π·Π°Π½Π½ΠΎΠ³ΠΎ Π²Ρ‹ΡˆΠ΅, Π½Π°Π·Π½Π°Ρ‡ΡŒΡ‚Π΅ Π·Π½Π°ΠΊ Β«+Β» для Ρ‚ΠΎΠΊΠ°, ΠΏΡ€ΠΎΡ‚Π΅ΠΊΠ°ΡŽΡ‰Π΅Π³ΠΎ Ρ‡Π΅Ρ€Π΅Π· ΡƒΠ·Π΅Π», ΠΈ Π·Π½Π°ΠΊ Β«-Β» для Ρ‚ΠΎΠΊΠ°, ΠΏΡ€ΠΎΡ‚Π΅ΠΊΠ°ΡŽΡ‰Π΅Π³ΠΎ Ρ‡Π΅Ρ€Π΅Π· ΡƒΠ·Π΅Π».

  • Π”Π°Π»Π΅Π΅ ΠΏΠ΅Ρ€Π²ΠΎΠ΅ ΡƒΡ€Π°Π²Π½Π΅Π½ΠΈΠ΅ Π·Π°ΠΊΠΎΠ½Π° ΠšΠΈΡ€Ρ…Π³ΠΎΡ„Π° записываСтся ΡΠ»Π΅Π΄ΡƒΡŽΡ‰ΠΈΠΌ ΠΎΠ±Ρ€Π°Π·ΠΎΠΌ:
  • Π£Ρ€Π°Π²Π½Π΅Π½ΠΈΠ΅, составлСнноС ΠΏΠΎ ΠΏΠ΅Ρ€Π²ΠΎΠΌΡƒ Π·Π°ΠΊΠΎΠ½Ρƒ, называСтся ΡƒΠ·Π»ΠΎΠΌ.

Π­Ρ‚ΠΎΡ‚ Π·Π°ΠΊΠΎΠ½ прСдставляСт Ρ‚ΠΎΡ‚ Ρ„Π°ΠΊΡ‚, Ρ‡Ρ‚ΠΎ заряд Π½Π΅ хранится ΠΈ Π½Π΅ потрСбляСтся Π² ΡƒΠ·Π»Π΅. ΠžΠ±Ρ‰ΠΈΠΉ заряд, Π΄ΠΎΡΡ‚ΠΈΠ³Π°ΡŽΡ‰ΠΈΠΉ ΡƒΠ·Π»Π°, Ρ€Π°Π²Π΅Π½ ΠΎΠ±Ρ‰Π΅ΠΌΡƒ заряду, ΠΏΠΎΠΊΠΈΠ΄Π°ΡŽΡ‰Π΅ΠΌΡƒ ΡƒΠ·Π΅Π» Π² Ρ‚Π΅Ρ‡Π΅Π½ΠΈΠ΅ Ρ‚ΠΎΠ³ΠΎ ΠΆΠ΅ ΠΏΠ΅Ρ€ΠΈΠΎΠ΄Π°.

Π’ Π·Π°ΠΌΠΊΠ½ΡƒΡ‚ΠΎΠΉ Ρ†Π΅ΠΏΠΈ алгСбраичСской суммы Π­Π”Π‘ эта схСма Ρ€Π°Π²Π½Π° алгСбраичСской суммС ΠΏΠ°Π΄Π΅Π½ΠΈΠΉ напряТСния элСмСнтов этой схСмы.

НомСр элСмСнта (рСзистора ΠΈΠ»ΠΈ источника напряТСния) Π² рассматриваСмой Ρ†Π΅ΠΏΠΈ.
ΠšΠΎΠ»ΠΈΡ‡Π΅ΡΡ‚Π²ΠΎ ΡƒΡ€Π°Π²Π½Π΅Π½ΠΈΠΉ, составлСнных ΠΏΠΎ Π²Ρ‚ΠΎΡ€ΠΎΠΌΡƒ Π·Π°ΠΊΠΎΠ½Ρƒ ΠšΠΈΡ€Ρ…Π³ΠΎΡ„Π°, опрСдСляСтся ΡΠ»Π΅Π΄ΡƒΡŽΡ‰ΠΈΠΌ ΠΎΠ±Ρ€Π°Π·ΠΎΠΌ:

Π“Π΄Π΅ число Π²Π΅Ρ‚Π²Π΅ΠΉ Π² элСктричСской Ρ†Π΅ΠΏΠΈ?

ΠšΠΎΠ»ΠΈΡ‡Π΅ΡΡ‚Π²ΠΎ ΠΈΠ΄Π΅Π°Π»ΡŒΠ½Ρ‹Ρ… источников Π­Π”Π‘. Для ΠΊΠ°ΠΆΠ΄ΠΎΠ³ΠΎ ΠΏΡ€ΠΎΠ²ΠΎΠ΄Π½ΠΈΠΊΠ° β€” Ρ‚Π²Π΅Ρ€Π΄Ρ‹ΠΉ, ΠΆΠΈΠ΄ΠΊΠΈΠΉ, Π³Π°Π· β€” сущСствуСт опрСдСлСнная Π·Π°Π²ΠΈΡΠΈΠΌΠΎΡΡ‚ΡŒ силы Ρ‚ΠΎΠΊΠ° ΠΎΡ‚ ΠΏΡ€ΠΈΠ»ΠΎΠΆΠ΅Π½Π½ΠΎΠΉ характСристики напряТСниС-Π²ΠΎΠ»ΡŒΡ‚-Π°ΠΌΠΏΠ΅Ρ€ (CVC). Он ΠΈΠΌΠ΅Π΅Ρ‚ ΠΏΡ€ΠΎΡΡ‚Π΅ΠΉΡˆΡƒΡŽ Ρ„ΠΎΡ€ΠΌΡƒ мСталличСского ΠΏΡ€ΠΎΠ²ΠΎΠ΄Π½ΠΈΠΊΠ° ΠΈ раствора элСктролита (рис. 5.2) ΠΈ опрСдСляСтся ΠΏΠΎ Π·Π°ΠΊΠΎΠ½Ρƒ Ома.

Богласно Π·Π°ΠΊΠΎΠ½Ρƒ Ома для ΠΎΠ΄Π½ΠΎΡ€ΠΎΠ΄Π½ΠΎΠ³ΠΎ (Π±Π΅Π· внСшнСй силы) участка Ρ†Π΅ΠΏΠΈ, сила Ρ‚ΠΎΠΊΠ° прямо ΠΏΡ€ΠΎΠΏΠΎΡ€Ρ†ΠΈΠΎΠ½Π°Π»ΡŒΠ½Π° ΠΏΡ€ΠΈΠ»ΠΎΠΆΠ΅Π½Π½ΠΎΠΌΡƒ Π½Π°ΠΏΡ€ΡΠΆΠ΅Π½ΠΈΡŽ U ΠΈ ΠΎΠ±Ρ€Π°Ρ‚Π½ΠΎ ΠΏΡ€ΠΎΠΏΠΎΡ€Ρ†ΠΈΠΎΠ½Π°Π»ΡŒΠ½Π° ΡΠΎΠΏΡ€ΠΎΡ‚ΠΈΠ²Π»Π΅Π½ΠΈΡŽ ΠΏΡ€ΠΎΠ²ΠΎΠ΄Π½ΠΈΠΊΠ° R.

  • Π•Π΄ΠΈΠ½ΠΈΡ†Π° сопротивлСния β€” Ом ([R] = 1 Ом). Ом β€” это сопротивлСниС ΠΏΡ€ΠΎΠ²ΠΎΠ΄Π½ΠΈΠΊΠ°, ΠΊΠΎΡ‚ΠΎΡ€ΠΎΠ΅ позволяСт Ρ‚ΠΎΠΊΡƒ 1 А ΠΏΡ€ΠΎΡ‚Π΅ΠΊΠ°Ρ‚ΡŒ ΠΏΡ€ΠΈ напряТСнии 1 Π’.

Π‘ΠΎΠΏΡ€ΠΎΡ‚ΠΈΠ²Π»Π΅Π½ΠΈΠ΅ зависит ΠΎΡ‚ свойств, Ρ„ΠΎΡ€ΠΌΡ‹ ΠΈ гСомСтричСских Ρ€Π°Π·ΠΌΠ΅Ρ€ΠΎΠ² ΠΏΡ€ΠΎΠ²ΠΎΠ΄Π½ΠΈΠΊΠ°. Для Ρ€Π°Π²Π½ΠΎΠΌΠ΅Ρ€Π½ΠΎΠ³ΠΎ цилиндричСского ΠΏΡ€ΠΎΠ²ΠΎΠ΄Π½ΠΈΠΊΠ°

Π”Π»ΠΈΠ½Π° ΠΏΡ€ΠΎΠ²ΠΎΠ΄Π½ΠΈΠΊΠ°, ΠΏΠ»ΠΎΡ‰Π°Π΄ΡŒ ΠΏΠΎΠΏΠ΅Ρ€Π΅Ρ‡Π½ΠΎΠ³ΠΎ сСчСния.

УдСльноС сопротивлСниС ΠΏΡ€ΠΎΠ²ΠΎΠ΄Π½ΠΈΠΊΠ° с Π΄Π»ΠΈΠ½ΠΎΠΉ r 1 ΠΌ ΠΈ ΠΏΠΎΠΏΠ΅Ρ€Π΅Ρ‡Π½Ρ‹ΠΌ сСчСниСм зависит ΠΎΡ‚ ΠΏΡ€ΠΈΡ€ΠΎΠ΄Ρ‹ ΠΈ Ρ‚Π΅ΠΌΠΏΠ΅Ρ€Π°Ρ‚ΡƒΡ€Ρ‹ ΠΏΡ€ΠΎΠ²ΠΎΠ΄Π½ΠΈΠΊΠ° ([r] = Ом.ΠΌ).

  • ΠŸΠΎΠ»ΡƒΡ‡ΠΈΡ‚Π΅ Π·Π°ΠΊΠΎΠ½ Ома для ΠΎΠ΄Π½ΠΎΡ€ΠΎΠ΄Π½Ρ‹Ρ… сСчСний Π΄ΠΈΡ„Ρ„Π΅Ρ€Π΅Π½Ρ†ΠΈΠ°Π»ΡŒΠ½ΠΎΠΉ Ρ†Π΅ΠΏΠΎΡ‡ΠΊΠΈ Ρ„ΠΎΡ€ΠΌ. Для этого Π²Ρ‹Π±Π΅Ρ€ΠΈΡ‚Π΅ основной цилиндричСский объСм с Π³Π΅Π½Π΅Ρ€Π°Ρ‚ΠΎΡ€ΠΎΠΌ рядом с Ρ‚ΠΎΡ‡ΠΊΠΎΠΉ Π²Π½ΡƒΡ‚Ρ€ΠΈ ΠΏΡ€ΠΎΠ²ΠΎΠ΄Π½ΠΈΠΊΠ° ΠΈ Π² этой Ρ‚ΠΎΡ‡ΠΊΠ΅, ΠΏΠ°Ρ€Π°Π»Π»Π΅Π»ΡŒΠ½ΠΎΠΉ Π²Π΅ΠΊΡ‚ΠΎΡ€Ρƒ плотности Ρ‚ΠΎΠΊΠ° j.

Π’ слоТных цСпях Π΅ΡΡ‚ΡŒ соСдинСния, ΠΊΠΎΡ‚ΠΎΡ€Ρ‹Π΅ Π½Π΅ происходят Π½ΠΈ ΠΎΡ‚ ΠΏΠΎΡΠ»Π΅Π΄ΠΎΠ²Π°Ρ‚Π΅Π»ΡŒΠ½Ρ‹Ρ…, Π½ΠΈ ΠΎΡ‚ ΠΏΠ°Ρ€Π°Π»Π»Π΅Π»ΡŒΠ½Ρ‹Ρ…. К Ρ‚Π°ΠΊΠΈΠΌ соСдинСниям относятся Ρ‚Ρ€Π΅Ρ…Π»ΡƒΡ‡Π΅Π²Ρ‹Π΅ Π·Π²Π΅Π·Π΄Ρ‹ ΠΈ Ρ‚Ρ€Π΅ΡƒΠ³ΠΎΠ»ΡŒΠ½ΠΈΠΊΠΈ сопротивлСния. Π’ΠΎ ΠΌΠ½ΠΎΠ³ΠΈΡ… случаях ΠΈΡ… Π²Π·Π°ΠΈΠΌΠ½Ρ‹Π΅ эквивалСнтныС прСобразования ΠΌΠΎΠ³ΡƒΡ‚ ΡƒΠΏΡ€ΠΎΡΡ‚ΠΈΡ‚ΡŒ схСму ΠΈ привСсти Π΅Π΅ ΠΊ цСпям со ΡΠΌΠ΅ΡˆΠ°Π½Π½Ρ‹ΠΌ рСзистором (ΠΏΠ°Ρ€Π°Π»Π»Π΅Π»ΡŒΠ½Ρ‹ΠΌ ΠΈ ΠΏΠΎΡΠ»Π΅Π΄ΠΎΠ²Π°Ρ‚Π΅Π»ΡŒΠ½Ρ‹ΠΌ). Π’ этом случаС сопротивлСниС Π·Π²Π΅Π·Π΄Ρ‹ ΠΈΠ»ΠΈ Ρ‚Ρ€Π΅ΡƒΠ³ΠΎΠ»ΡŒΠ½ΠΈΠΊΠ° Π½Π΅ΠΎΠ±Ρ…ΠΎΠ΄ΠΈΠΌΠΎ ΠΏΠ΅Ρ€Π΅ΡΡ‡ΠΈΡ‚Π°Ρ‚ΡŒ ΠΎΠΏΡ€Π΅Π΄Π΅Π»Π΅Π½Π½Ρ‹ΠΌ ΠΎΠ±Ρ€Π°Π·ΠΎΠΌ.

Π‘ΠΌΠΎΡ‚Ρ€ΠΈΡ‚Π΅ Ρ‚Π°ΠΊΠΆΠ΅:

Π—Π°ΠΊΠΎΠ½Ρ‹ расчСта Ρ†Π΅ΠΏΠ΅ΠΉ Π·Π°ΠΊΠΎΠ½ ΠΎΠΌΠ° β€” КоллСгия Π°Π΄Π²ΠΎΠΊΠ°Ρ‚ΠΎΠ²

ΠžΡΠ½ΠΎΠ²Π½Ρ‹ΠΌ Π·Π°ΠΊΠΎΠ½ΠΎΠΌ элСктротСхники, ΠΏΡ€ΠΈ ΠΏΠΎΠΌΠΎΡ‰ΠΈ ΠΊΠΎΡ‚ΠΎΡ€ΠΎΠ³ΠΎ ΠΌΠΎΠΆΠ½ΠΎ ΠΈΠ·ΡƒΡ‡Π°Ρ‚ΡŒ ΠΈ Ρ€Π°ΡΡΡ‡ΠΈΡ‚Ρ‹Π²Π°Ρ‚ΡŒ элСктричСскиС Ρ†Π΅ΠΏΠΈ, являСтся Π·Π°ΠΊΠΎΠ½ Ома, ΡƒΡΡ‚Π°Π½Π°Π²Π»ΠΈΠ²Π°ΡŽΡ‰ΠΈΠΉ ΡΠΎΠΎΡ‚Π½ΠΎΡˆΠ΅Π½ΠΈΠ΅ ΠΌΠ΅ΠΆΠ΄Ρƒ Ρ‚ΠΎΠΊΠΎΠΌ, напряТСниСм ΠΈ сопротивлСниСм. НСобходимо ΠΎΡ‚Ρ‡Π΅Ρ‚Π»ΠΈΠ²ΠΎ ΠΏΠΎΠ½ΠΈΠΌΠ°Ρ‚ΡŒ Π΅Π³ΠΎ ΡΡƒΡ‰Π½ΠΎΡΡ‚ΡŒ ΠΈ ΡƒΠΌΠ΅Ρ‚ΡŒ ΠΏΡ€Π°Π²ΠΈΠ»ΡŒΠ½ΠΎ ΠΏΠΎΠ»ΡŒΠ·ΠΎΠ²Π°Ρ‚ΡŒΡΡ ΠΈΠΌ ΠΏΡ€ΠΈ Ρ€Π΅ΡˆΠ΅Π½ΠΈΠΈ практичСских Π·Π°Π΄Π°Ρ‡. Часто Π² элСктротСхникС Π΄ΠΎΠΏΡƒΡΠΊΠ°ΡŽΡ‚ΡΡ ошибки ΠΈΠ·-Π·Π° нСумСния ΠΏΡ€Π°Π²ΠΈΠ»ΡŒΠ½ΠΎ ΠΏΡ€ΠΈΠΌΠ΅Π½ΠΈΡ‚ΡŒ Π·Π°ΠΊΠΎΠ½ Ома.

Π—Π°ΠΊΠΎΠ½ Ома для участка Ρ†Π΅ΠΏΠΈ гласит: Ρ‚ΠΎΠΊ прямо ΠΏΡ€ΠΎΠΏΠΎΡ€Ρ†ΠΈΠΎΠ½Π°Π»Π΅Π½ Π½Π°ΠΏΡ€ΡΠΆΠ΅Π½ΠΈΡŽ ΠΈ ΠΎΠ±Ρ€Π°Ρ‚Π½ΠΎ ΠΏΡ€ΠΎΠΏΠΎΡ€Ρ†ΠΈΠΎΠ½Π°Π»Π΅Π½ ΡΠΎΠΏΡ€ΠΎΡ‚ΠΈΠ²Π»Π΅Π½ΠΈΡŽ.

Если ΡƒΠ²Π΅Π»ΠΈΡ‡ΠΈΡ‚ΡŒ Π² нСсколько Ρ€Π°Π· напряТСниС, Π΄Π΅ΠΉΡΡ‚Π²ΡƒΡŽΡ‰Π΅Π΅ Π² элСктричСской Ρ†Π΅ΠΏΠΈ, Ρ‚ΠΎ Ρ‚ΠΎΠΊ Π² этой Ρ†Π΅ΠΏΠΈ увСличится Π²ΠΎ ΡΡ‚ΠΎΠ»ΡŒΠΊΠΎ ΠΆΠ΅ Ρ€Π°Π·. А Ссли ΡƒΠ²Π΅Π»ΠΈΡ‡ΠΈΡ‚ΡŒ Π² нСсколько Ρ€Π°Π· сопротивлСниС Ρ†Π΅ΠΏΠΈ, Ρ‚ΠΎ Ρ‚ΠΎΠΊ Π²ΠΎ ΡΡ‚ΠΎΠ»ΡŒΠΊΠΎ ΠΆΠ΅ Ρ€Π°Π· ΡƒΠΌΠ΅Π½ΡŒΡˆΠΈΡ‚ΡΡ. Подобно этому водяной ΠΏΠΎΡ‚ΠΎΠΊ Π² Ρ‚Ρ€ΡƒΠ±Π΅ Ρ‚Π΅ΠΌ большС, Ρ‡Π΅ΠΌ сильнСС Π΄Π°Π²Π»Π΅Π½ΠΈΠ΅ ΠΈ Ρ‡Π΅ΠΌ мСньшС сопротивлСниС, ΠΊΠΎΡ‚ΠΎΡ€ΠΎΠ΅ ΠΎΠΊΠ°Π·Ρ‹Π²Π°Π΅Ρ‚ Ρ‚Ρ€ΡƒΠ±Π° двиТСнию Π²ΠΎΠ΄Ρ‹.

Π§Ρ‚ΠΎΠ±Ρ‹ Π²Ρ‹Ρ€Π°Π·ΠΈΡ‚ΡŒ Π·Π°ΠΊΠΎΠ½ Ома матСматичСски Π½Π°ΠΈΠ±ΠΎΠ»Π΅Π΅ просто, ΡΡ‡ΠΈΡ‚Π°ΡŽΡ‚, Ρ‡Ρ‚ΠΎ сопротивлСниС ΠΏΡ€ΠΎΠ²ΠΎΠ΄Π½ΠΈΠΊΠ°, Π² ΠΊΠΎΡ‚ΠΎΡ€ΠΎΠΌ ΠΏΡ€ΠΈ напряТСнии 1 Π’ ΠΏΡ€ΠΎΡ…ΠΎΠ΄ΠΈΡ‚ Ρ‚ΠΎΠΊ 1 А, Ρ€Π°Π²Π½ΠΎ 1 Ом.

Π’ΠΎΠΊ Π² Π°ΠΌΠΏΠ΅Ρ€Π°Ρ… ΠΌΠΎΠΆΠ½ΠΎ всСгда ΠΎΠΏΡ€Π΅Π΄Π΅Π»ΠΈΡ‚ΡŒ, Ссли Ρ€Π°Π·Π΄Π΅Π»ΠΈΡ‚ΡŒ напряТСниС Π² Π²ΠΎΠ»ΡŒΡ‚Π°Ρ… Π½Π° сопротивлСниС Π² ΠΎΠΌΠ°Ρ…. ΠŸΠΎΡΡ‚ΠΎΠΌΡƒ Π·Π°ΠΊΠΎΠ½ Ома для участка Ρ†Π΅ΠΏΠΈ записываСтся ΡΠ»Π΅Π΄ΡƒΡŽΡ‰Π΅ΠΉ Ρ„ΠΎΡ€ΠΌΡƒΠ»ΠΎΠΉ:

Π›ΡŽΠ±ΠΎΠΉ участок ΠΈΠ»ΠΈ элСмСнт элСктричСской Ρ†Π΅ΠΏΠΈ ΠΌΠΎΠΆΠ½ΠΎ ΠΎΡ…Π°Ρ€Π°ΠΊΡ‚Π΅Ρ€ΠΈΠ·ΠΎΠ²Π°Ρ‚ΡŒ ΠΏΡ€ΠΈ ΠΏΠΎΠΌΠΎΡ‰ΠΈ Ρ‚Ρ€Ρ‘Ρ… характСристик: Ρ‚ΠΎΠΊΠ°, напряТСния ΠΈ сопротивлСния.

Как ΠΈΡΠΏΠΎΠ»ΡŒΠ·ΠΎΠ²Π°Ρ‚ΡŒ Ρ‚Ρ€Π΅ΡƒΠ³ΠΎΠ»ΡŒΠ½ΠΈΠΊ Ома: Π·Π°ΠΊΡ€Ρ‹Π²Π°Π΅ΠΌ ΠΈΡΠΊΠΎΠΌΡƒΡŽ Π²Π΅Π»ΠΈΡ‡ΠΈΠ½Ρƒ β€” Π΄Π²Π° Π΄Ρ€ΡƒΠ³ΠΈΡ… символа Π΄Π°Π΄ΡƒΡ‚ Ρ„ΠΎΡ€ΠΌΡƒΠ»Ρƒ для Π΅Ρ‘ вычислСния. ΠšΡΡ‚Π°Ρ‚ΠΈ, Π·Π°ΠΊΠΎΠ½ΠΎΠΌ Ома называСтся Ρ‚ΠΎΠ»ΡŒΠΊΠΎ ΠΎΠ΄Π½Π° Ρ„ΠΎΡ€ΠΌΡƒΠ»Π° ΠΈΠ· Ρ‚Ρ€Π΅ΡƒΠ³ΠΎΠ»ΡŒΠ½ΠΈΠΊΠ° – Ρ‚Π°, которая ΠΎΡ‚Ρ€Π°ΠΆΠ°Π΅Ρ‚ Π·Π°Π²ΠΈΡΠΈΠΌΠΎΡΡ‚ΡŒ Ρ‚ΠΎΠΊΠ° ΠΎΡ‚ напряТСния ΠΈ сопротивлСния. Π”Π²Π΅ Π΄Ρ€ΡƒΠ³ΠΈΠ΅ Ρ„ΠΎΡ€ΠΌΡƒΠ»Ρ‹, хотя ΠΈ ΡΠ²Π»ΡΡŽΡ‚ΡΡ Π΅Ρ‘ слСдствиСм, физичСского смысла Π½Π΅ ΠΈΠΌΠ΅ΡŽΡ‚.

РасчСты, выполняСмыС с ΠΏΠΎΠΌΠΎΡ‰ΡŒΡŽ Π·Π°ΠΊΠΎΠ½Π° Ома для участка Ρ†Π΅ΠΏΠΈ, Π±ΡƒΠ΄ΡƒΡ‚ ΠΏΡ€Π°Π²ΠΈΠ»ΡŒΠ½Ρ‹ Π² Ρ‚ΠΎΠΌ случаС, ΠΊΠΎΠ³Π΄Π° напряТСниС Π²Ρ‹Ρ€Π°ΠΆΠ΅Π½ΠΎ Π² Π²ΠΎΠ»ΡŒΡ‚Π°Ρ…, сопротивлСниС Π² ΠΎΠΌΠ°Ρ… ΠΈ Ρ‚ΠΎΠΊ Π² Π°ΠΌΠΏΠ΅Ρ€Π°Ρ…. Если ΠΈΡΠΏΠΎΠ»ΡŒΠ·ΡƒΡŽΡ‚ΡΡ ΠΊΡ€Π°Ρ‚Π½Ρ‹Π΅ Π΅Π΄ΠΈΠ½ΠΈΡ†Ρ‹ ΠΈΠ·ΠΌΠ΅Ρ€Π΅Π½ΠΈΠΉ этих Π²Π΅Π»ΠΈΡ‡ΠΈΠ½ (Π½Π°ΠΏΡ€ΠΈΠΌΠ΅Ρ€, ΠΌΠΈΠ»Π»ΠΈΠ°ΠΌΠΏΠ΅Ρ€, ΠΌΠΈΠ»Π»ΠΈΠ²ΠΎΠ»ΡŒΡ‚, ΠΌΠ΅Π³Π°ΠΎΠΌ ΠΈ Ρ‚. Π΄.), Ρ‚ΠΎ ΠΈΡ… слСдуСт пСрСвСсти соотвСтствСнно Π² Π°ΠΌΠΏΠ΅Ρ€Ρ‹, Π²ΠΎΠ»ΡŒΡ‚Ρ‹ ΠΈ ΠΎΠΌΡ‹. Π§Ρ‚ΠΎΠ±Ρ‹ ΠΏΠΎΠ΄Ρ‡Π΅Ρ€ΠΊΠ½ΡƒΡ‚ΡŒ это, ΠΈΠ½ΠΎΠ³Π΄Π° Ρ„ΠΎΡ€ΠΌΡƒΠ»Ρƒ Π·Π°ΠΊΠΎΠ½Π° Ома для участка Ρ†Π΅ΠΏΠΈ ΠΏΠΈΡˆΡƒΡ‚ Ρ‚Π°ΠΊ:

МоТно Ρ‚Π°ΠΊΠΆΠ΅ Ρ€Π°ΡΡΡ‡ΠΈΡ‚Ρ‹Π²Π°Ρ‚ΡŒ Ρ‚ΠΎΠΊ Π² ΠΌΠΈΠ»Π»ΠΈΠ°ΠΌΠΏΠ΅Ρ€Π°Ρ… ΠΈ ΠΌΠΈΠΊΡ€ΠΎΠ°ΠΌΠΏΠ΅Ρ€Π°Ρ…, ΠΏΡ€ΠΈ этом напряТСниС Π΄ΠΎΠ»ΠΆΠ½ΠΎ Π±Ρ‹Ρ‚ΡŒ Π²Ρ‹Ρ€Π°ΠΆΠ΅Π½ΠΎ Π² Π²ΠΎΠ»ΡŒΡ‚Π°Ρ…, Π° сопротивлСниС β€” Π² ΠΊΠΈΠ»ΠΎΠΎΠΌΠ°Ρ… ΠΈ ΠΌΠ΅Π³Π°ΠΎΠΌΠ°Ρ… соотвСтствСнно.

Π”Ρ€ΡƒΠ³ΠΈΠ΅ ΡΡ‚Π°Ρ‚ΡŒΠΈ ΠΏΡ€ΠΎ элСктричСство Π² простом ΠΈ доступном ΠΈΠ·Π»ΠΎΠΆΠ΅Π½ΠΈΠΈ:

Π—Π°ΠΊΠΎΠ½ Ома справСдлив для любого участка Ρ†Π΅ΠΏΠΈ. Если трСбуСтся ΠΎΠΏΡ€Π΅Π΄Π΅Π»ΠΈΡ‚ΡŒ Ρ‚ΠΎΠΊ Π² Π΄Π°Π½Π½ΠΎΠΌ участкС Ρ†Π΅ΠΏΠΈ, Ρ‚ΠΎ Π½Π΅ΠΎΠ±Ρ…ΠΎΠ΄ΠΈΠΌΠΎ напряТСниС, Π΄Π΅ΠΉΡΡ‚Π²ΡƒΡŽΡ‰Π΅Π΅ Π½Π° этом участкС (рис. 1), Ρ€Π°Π·Π΄Π΅Π»ΠΈΡ‚ΡŒ Π½Π° сопротивлСниС ΠΈΠΌΠ΅Π½Π½ΠΎ этого участка.

Рис 1. ΠŸΡ€ΠΈΠΌΠ΅Π½Π΅Π½ΠΈΠ΅ Π·Π°ΠΊΠΎΠ½Π° Ома для участка Ρ†Π΅ΠΏΠΈ

ΠŸΡ€ΠΈΠ²Π΅Π΄Π΅ΠΌ ΠΏΡ€ΠΈΠΌΠ΅Ρ€ расчСта Ρ‚ΠΎΠΊΠ° ΠΏΠΎ Π·Π°ΠΊΠΎΠ½Ρƒ Ома . ΠŸΡƒΡΡ‚ΡŒ трСбуСтся ΠΎΠΏΡ€Π΅Π΄Π΅Π»ΠΈΡ‚ΡŒ Ρ‚ΠΎΠΊ Π² Π»Π°ΠΌΠΏΠ΅, ΠΈΠΌΠ΅ΡŽΡ‰Π΅ΠΉ сопротивлСниС 2,5 Ом, Ссли напряТСниС, ΠΏΡ€ΠΈΠ»ΠΎΠΆΠ΅Π½Π½ΠΎΠ΅ ΠΊ Π»Π°ΠΌΠΏΠ΅, составляСт 5 Π’. Π Π°Π·Π΄Π΅Π»ΠΈΠ² 5 Π’ Π½Π° 2,5 Ом, ΠΏΠΎΠ»ΡƒΡ‡ΠΈΠΌ Π·Π½Π°Ρ‡Π΅Π½ΠΈΠ΅ Ρ‚ΠΎΠΊΠ°, Ρ€Π°Π²Π½ΠΎΠ΅ 2 А. Π’ΠΎ Π²Ρ‚ΠΎΡ€ΠΎΠΌ ΠΏΡ€ΠΈΠΌΠ΅Ρ€Π΅ ΠΎΠΏΡ€Π΅Π΄Π΅Π»ΠΈΠΌ Ρ‚ΠΎΠΊ, ΠΊΠΎΡ‚ΠΎΡ€Ρ‹ΠΉ Π±ΡƒΠ΄Π΅Ρ‚ ΠΏΡ€ΠΎΡ‚Π΅ΠΊΠ°Ρ‚ΡŒ ΠΏΠΎΠ΄ дСйствиСм напряТСния 500 Π’ Π² Ρ†Π΅ΠΏΠΈ, сопротивлСниС ΠΊΠΎΡ‚ΠΎΡ€ΠΎΠΉ Ρ€Π°Π²Π½ΠΎ 0,5 МОм. Для этого Π²Ρ‹Ρ€Π°Π·ΠΈΠΌ сопротивлСниС Π² ΠΎΠΌΠ°Ρ…. Π Π°Π·Π΄Π΅Π»ΠΈΠ² 500 Π’ Π½Π° 500 000 Ом, Π½Π°ΠΉΠ΄Π΅ΠΌ Π·Π½Π°Ρ‡Π΅Π½ΠΈΠ΅ Ρ‚ΠΎΠΊΠ° Π² Ρ†Π΅ΠΏΠΈ, ΠΊΠΎΡ‚ΠΎΡ€ΠΎΠ΅ Ρ€Π°Π²Π½ΠΎ 0,001 А ΠΈΠ»ΠΈ 1 мА.

Часто, зная Ρ‚ΠΎΠΊ ΠΈ сопротивлСниС, ΠΎΠΏΡ€Π΅Π΄Π΅Π»ΡΡŽΡ‚ с ΠΏΠΎΠΌΠΎΡ‰ΡŒΡŽ Π·Π°ΠΊΠΎΠ½Π° Ома напряТСниС. Π—Π°ΠΏΠΈΡˆΠ΅ΠΌ Ρ„ΠΎΡ€ΠΌΡƒΠ»Ρƒ для опрСдСлСния напряТСния

Из этой Ρ„ΠΎΡ€ΠΌΡƒΠ»Ρ‹ Π²ΠΈΠ΄Π½ΠΎ, Ρ‡Ρ‚ΠΎ напряТСниС Π½Π° ΠΊΠΎΠ½Ρ†Π°Ρ… Π΄Π°Π½Π½ΠΎΠ³ΠΎ участка Ρ†Π΅ΠΏΠΈ прямо ΠΏΡ€ΠΎΠΏΠΎΡ€Ρ†ΠΈΠΎΠ½Π°Π»ΡŒΠ½ΠΎ Ρ‚ΠΎΠΊΡƒ ΠΈ ΡΠΎΠΏΡ€ΠΎΡ‚ΠΈΠ²Π»Π΅Π½ΠΈΡŽ . Бмысл этой зависимости ΠΏΠΎΠ½ΡΡ‚ΡŒ Π½Π΅Ρ‚Ρ€ΡƒΠ΄Π½ΠΎ. Если Π½Π΅ ΠΈΠ·ΠΌΠ΅Π½ΡΡ‚ΡŒ сопротивлСниС участка Ρ†Π΅ΠΏΠΈ, Ρ‚ΠΎ ΡƒΠ²Π΅Π»ΠΈΡ‡ΠΈΡ‚ΡŒ Ρ‚ΠΎΠΊ ΠΌΠΎΠΆΠ½ΠΎ Ρ‚ΠΎΠ»ΡŒΠΊΠΎ ΠΏΡƒΡ‚Π΅ΠΌ увСличСния напряТСния. Π—Π½Π°Ρ‡ΠΈΡ‚ ΠΏΡ€ΠΈ постоянном сопротивлСнии Π±ΠΎΠ»ΡŒΡˆΠ΅ΠΌΡƒ Ρ‚ΠΎΠΊΡƒ соотвСтствуСт большСС напряТСниС. Если ΠΆΠ΅ Π½Π°Π΄ΠΎ ΠΏΠΎΠ»ΡƒΡ‡ΠΈΡ‚ΡŒ ΠΎΠ΄ΠΈΠ½ ΠΈ Ρ‚ΠΎΡ‚ ΠΆΠ΅ Ρ‚ΠΎΠΊ ΠΏΡ€ΠΈ Ρ€Π°Π·Π»ΠΈΡ‡Π½Ρ‹Ρ… сопротивлСниях, Ρ‚ΠΎ ΠΏΡ€ΠΈ большСм сопротивлСнии Π΄ΠΎΠ»ΠΆΠ½ΠΎ Π±Ρ‹Ρ‚ΡŒ соотвСтствСнно большСС напряТСниС.

НапряТСниС Π½Π° участкС Ρ†Π΅ΠΏΠΈ часто Π½Π°Π·Ρ‹Π²Π°ΡŽΡ‚ ΠΏΠ°Π΄Π΅Π½ΠΈΠ΅ΠΌ напряТСния . Π­Ρ‚ΠΎ Π½Π΅Ρ€Π΅Π΄ΠΊΠΎ ΠΏΡ€ΠΈΠ²ΠΎΠ΄ΠΈΡ‚ ΠΊ Π½Π΅Π΄ΠΎΡ€Π°Π·ΡƒΠΌΠ΅Π½ΠΈΡŽ. МногиС Π΄ΡƒΠΌΠ°ΡŽΡ‚, Ρ‡Ρ‚ΠΎ ΠΏΠ°Π΄Π΅Π½ΠΈΠ΅ напряТСния Π΅ΡΡ‚ΡŒ ΠΊΠ°ΠΊΠΎΠ΅-Ρ‚ΠΎ потСрянноС Π½Π΅Π½ΡƒΠΆΠ½ΠΎΠ΅ напряТСниС. Π’ Π΄Π΅ΠΉΡΡ‚Π²ΠΈΡ‚Π΅Π»ΡŒΠ½ΠΎΡΡ‚ΠΈ ΠΆΠ΅ понятия напряТСниС ΠΈ ΠΏΠ°Π΄Π΅Π½ΠΈΠ΅ напряТСния Ρ€Π°Π²Π½ΠΎΠ·Π½Π°Ρ‡Π½Ρ‹. ΠŸΠΎΡ‚Π΅Ρ€ΠΈ ΠΈ ΠΏΠ°Π΄Π΅Π½ΠΈΠ΅ напряТСния β€” Π² Ρ‡Π΅ΠΌ Ρ€Π°Π·Π»ΠΈΡ‡ΠΈΠ΅?

РасчСт напряТСния с ΠΏΠΎΠΌΠΎΡ‰ΡŒΡŽ Π·Π°ΠΊΠΎΠ½Π° Ома ΠΌΠΎΠΆΠ½ΠΎ ΠΏΠΎΠΊΠ°Π·Π°Ρ‚ΡŒ Π½Π° ΡΠ»Π΅Π΄ΡƒΡŽΡ‰Π΅ΠΌ ΠΏΡ€ΠΈΠΌΠ΅Ρ€Π΅. ΠŸΡƒΡΡ‚ΡŒ Ρ‡Π΅Ρ€Π΅Π· участок Ρ†Π΅ΠΏΠΈ с сопротивлСниСм 10 кОм ΠΏΡ€ΠΎΡ…ΠΎΠ΄ΠΈΡ‚ Ρ‚ΠΎΠΊ 5 мА ΠΈ трСбуСтся ΠΎΠΏΡ€Π΅Π΄Π΅Π»ΠΈΡ‚ΡŒ напряТСниС Π½Π° этом участкС.

Π£ΠΌΠ½ΠΎΠΆΠΈΠ² I = 0,005 А Π½Π° R β€”10 000 Ом, ΠΏΠΎΠ»ΡƒΡ‡ΠΈΠΌ напряТСниС,Ρ€Π°Π²Π½ΠΎΠ΅ 5 0 Π’. МоТно Π±Ρ‹Π»ΠΎ Π±Ρ‹ ΠΏΠΎΠ»ΡƒΡ‡ΠΈΡ‚ΡŒ Ρ‚ΠΎΡ‚ ΠΆΠ΅ Ρ€Π΅Π·ΡƒΠ»ΡŒΡ‚Π°Ρ‚, ΡƒΠΌΠ½ΠΎΠΆΠΈΠ² 5 мА Π½Π° 10 кОм: U = 50 Π’

Π’ элСктронных устройствах Ρ‚ΠΎΠΊ ΠΎΠ±Ρ‹Ρ‡Π½ΠΎ выраТаСтся Π² ΠΌΠΈΠ»Π»ΠΈΠ°ΠΌΠΏΠ΅Ρ€Π°Ρ…, Π° сопротивлСниС β€” Π² ΠΊΠΈΠ»ΠΎΠΎΠΌΠ°Ρ…. ΠŸΠΎΡΡ‚ΠΎΠΌΡƒ ΡƒΠ΄ΠΎΠ±Π½ΠΎ Π² расчСтах ΠΏΠΎ Π·Π°ΠΊΠΎΠ½Ρƒ Ома ΠΏΡ€ΠΈΠΌΠ΅Π½ΡΡ‚ΡŒ ΠΈΠΌΠ΅Π½Π½ΠΎ эти Π΅Π΄ΠΈΠ½ΠΈΡ†Ρ‹ ΠΈΠ·ΠΌΠ΅Ρ€Π΅Π½ΠΈΠΉ.

По Π·Π°ΠΊΠΎΠ½Ρƒ Ома рассчитываСтся Ρ‚Π°ΠΊΠΆΠ΅ сопротивлСниС, Ссли извСстно напряТСниС ΠΈ Ρ‚ΠΎΠΊ. Π€ΠΎΡ€ΠΌΡƒΠ»Π° для этого случая ΠΏΠΈΡˆΠ΅Ρ‚ΡΡ ΡΠ»Π΅Π΄ΡƒΡŽΡ‰ΠΈΠΌ ΠΎΠ±Ρ€Π°Π·ΠΎΠΌ: R = U/I.

Π‘ΠΎΠΏΡ€ΠΎΡ‚ΠΈΠ²Π»Π΅Π½ΠΈΠ΅ всСгда прСдставляСт собой ΠΎΡ‚Π½ΠΎΡˆΠ΅Π½ΠΈΠ΅ напряТСния ΠΊ Ρ‚ΠΎΠΊΡƒ. Если напряТСниС ΡƒΠ²Π΅Π»ΠΈΡ‡ΠΈΡ‚ΡŒ ΠΈΠ»ΠΈ ΡƒΠΌΠ΅Π½ΡŒΡˆΠΈΡ‚ΡŒ Π² нСсколько Ρ€Π°Π·, Ρ‚ΠΎ Ρ‚ΠΎΠΊ увСличится ΠΈΠ»ΠΈ ΡƒΠΌΠ΅Π½ΡŒΡˆΠΈΡ‚ΡΡ Π² Ρ‚Π°ΠΊΠΎΠ΅ ΠΆΠ΅ число Ρ€Π°Π·. ΠžΡ‚Π½ΠΎΡˆΠ΅Π½ΠΈΠ΅ напряТСния ΠΊ Ρ‚ΠΎΠΊΡƒ, Ρ€Π°Π²Π½ΠΎΠ΅ ΡΠΎΠΏΡ€ΠΎΡ‚ΠΈΠ²Π»Π΅Π½ΠΈΡŽ, остаСтся Π½Π΅ΠΈΠ·ΠΌΠ΅Π½Π½Ρ‹ΠΌ.

НС слСдуСт ΠΏΠΎΠ½ΠΈΠΌΠ°Ρ‚ΡŒ Ρ„ΠΎΡ€ΠΌΡƒΠ»Ρƒ для опрСдСлСния сопротивлСния Π² Ρ‚ΠΎΠΌ смыслС, Ρ‡Ρ‚ΠΎ сопротивлСниС Π΄Π°Π½Π½ΠΎΠ³ΠΎ ΠΏΡ€ΠΎΠ²ΠΎΠ΄Π½ΠΈΠΊΠ° зависит ΠΎΡ‚Ρ‚ΠΎΠΊΠ° ΠΈ напряТСния. Π˜Π·Π²Π΅ΡΡ‚Π½ΠΎ, Ρ‡Ρ‚ΠΎ ΠΎΠ½ΠΎ зависит ΠΎΡ‚ Π΄Π»ΠΈΠ½Ρ‹, ΠΏΠ»ΠΎΡ‰Π°Π΄ΠΈ сСчСния ΠΈ ΠΌΠ°Ρ‚Π΅Ρ€ΠΈΠ°Π»Π° ΠΏΡ€ΠΎΠ²ΠΎΠ΄Π½ΠΈΠΊΠ°. По Π²Π½Π΅ΡˆΠ½Π΅ΠΌΡƒ Π²ΠΈΠ΄Ρƒ Ρ„ΠΎΡ€ΠΌΡƒΠ»Π° для опрСдСлСния сопротивлСния Π½Π°ΠΏΠΎΠΌΠΈΠ½Π°Π΅Ρ‚ Ρ„ΠΎΡ€ΠΌΡƒΠ»Ρƒ для расчСта Ρ‚ΠΎΠΊΠ°, Π½ΠΎ ΠΌΠ΅ΠΆΠ΄Ρƒ Π½ΠΈΠΌΠΈ имССтся ΠΏΡ€ΠΈΠ½Ρ†ΠΈΠΏΠΈΠ°Π»ΡŒΠ½Π°Ρ Ρ€Π°Π·Π½ΠΈΡ†Π°.

Π’ΠΎΠΊ Π² Π΄Π°Π½Π½ΠΎΠΌ участкС Ρ†Π΅ΠΏΠΈ Π΄Π΅ΠΉΡΡ‚Π²ΠΈΡ‚Π΅Π»ΡŒΠ½ΠΎ зависит ΠΎΡ‚ напряТСния ΠΈ сопротивлСния ΠΈ измСняСтся ΠΏΡ€ΠΈ ΠΈΡ… ΠΈΠ·ΠΌΠ΅Π½Π΅Π½ΠΈΠΈ. А сопротивлСниС Π΄Π°Π½Π½ΠΎΠ³ΠΎ участка Ρ†Π΅ΠΏΠΈ являСтся Π²Π΅Π»ΠΈΡ‡ΠΈΠ½ΠΎΠΉ постоянной, Π½Π΅ зависящСй ΠΎΡ‚ измСнСния напряТСния ΠΈ Ρ‚ΠΎΠΊΠ°, Π½ΠΎ Ρ€Π°Π²Π½ΠΎΠΉ ΠΎΡ‚Π½ΠΎΡˆΠ΅Π½ΠΈΡŽ этих Π²Π΅Π»ΠΈΡ‡ΠΈΠ½.

Когда ΠΎΠ΄ΠΈΠ½ ΠΈ Ρ‚ΠΎΡ‚ ΠΆΠ΅ Ρ‚ΠΎΠΊ ΠΏΡ€ΠΎΡ…ΠΎΠ΄ΠΈΡ‚ Π² Π΄Π²ΡƒΡ… участках Ρ†Π΅ΠΏΠΈ, Π° напряТСния, ΠΏΡ€ΠΈΠ»ΠΎΠΆΠ΅Π½Π½Ρ‹Π΅ ΠΊ Π½ΠΈΠΌ, Ρ€Π°Π·Π»ΠΈΡ‡Π½Ρ‹, Ρ‚ΠΎ ясно, Ρ‡Ρ‚ΠΎ участок, ΠΊ ΠΊΠΎΡ‚ΠΎΡ€ΠΎΠΌΡƒ ΠΏΡ€ΠΈΠ»ΠΎΠΆΠ΅Π½ΠΎ большСС напряТСниС, ΠΈΠΌΠ΅Π΅Ρ‚ соотвСтствСнно большСС сопротивлСниС.

А Ссли ΠΏΠΎΠ΄ дСйствиСм ΠΎΠ΄Π½ΠΎΠ³ΠΎ ΠΈ Ρ‚ΠΎΠ³ΠΎ ΠΆΠ΅ напряТСния Π² Π΄Π²ΡƒΡ… Ρ€Π°Π·Π½Ρ‹Ρ… участках Ρ†Π΅ΠΏΠΈ ΠΏΡ€ΠΎΡ…ΠΎΠ΄ΠΈΡ‚ Ρ€Π°Π·Π»ΠΈΡ‡Π½Ρ‹ΠΉ Ρ‚ΠΎΠΊ, Ρ‚ΠΎ мСньший Ρ‚ΠΎΠΊ всСгда Π±ΡƒΠ΄Π΅Ρ‚ Π½Π° Ρ‚ΠΎΠΌ участкС, ΠΊΠΎΡ‚ΠΎΡ€Ρ‹ΠΉ ΠΈΠΌΠ΅Π΅Ρ‚ большСС сопротивлСниС. ВсС это Π²Ρ‹Ρ‚Π΅ΠΊΠ°Π΅Ρ‚ ΠΈΠ· основной Ρ„ΠΎΡ€ΠΌΡƒΠ»ΠΈΡ€ΠΎΠ²ΠΊΠΈ Π·Π°ΠΊΠΎΠ½Π° Ома для участка Ρ†Π΅ΠΏΠΈ, Ρ‚. Π΅. ΠΈΠ· Ρ‚ΠΎΠ³ΠΎ, Ρ‡Ρ‚ΠΎ Ρ‚ΠΎΠΊ Ρ‚Π΅ΠΌ большС, Ρ‡Π΅ΠΌ большС напряТСниС ΠΈ Ρ‡Π΅ΠΌ мСньшС сопротивлСниС.

РасчСт сопротивлСния с ΠΏΠΎΠΌΠΎΡ‰ΡŒΡŽ Π·Π°ΠΊΠΎΠ½Π° Ома для участка Ρ†Π΅ΠΏΠΈ ΠΏΠΎΠΊΠ°ΠΆΠ΅ΠΌ Π½Π° ΡΠ»Π΅Π΄ΡƒΡŽΡ‰Π΅ΠΌ ΠΏΡ€ΠΈΠΌΠ΅Ρ€Π΅. ΠŸΡƒΡΡ‚ΡŒ трСбуСтся Π½Π°ΠΉΡ‚ΠΈ сопротивлСниС участка, Ρ‡Π΅Ρ€Π΅Π· ΠΊΠΎΡ‚ΠΎΡ€Ρ‹ΠΉ ΠΏΡ€ΠΈ напряТСнии 40 Π’ ΠΏΡ€ΠΎΡ…ΠΎΠ΄ΠΈΡ‚ Ρ‚ΠΎΠΊ 50 мА. Π’Ρ‹Ρ€Π°Π·ΠΈΠ² Ρ‚ΠΎΠΊ Π² Π°ΠΌΠΏΠ΅Ρ€Π°Ρ…, ΠΏΠΎΠ»ΡƒΡ‡ΠΈΠΌ I = 0,05 А. Π Π°Π·Π΄Π΅Π»ΠΈΠΌ 40 Π½Π° 0,05 ΠΈ Π½Π°ΠΉΠ΄Π΅ΠΌ, Ρ‡Ρ‚ΠΎ сопротивлСниС составляСт 800 Ом.

Π—Π°ΠΊΠΎΠ½ Ома ΠΌΠΎΠΆΠ½ΠΎ наглядно ΠΏΡ€Π΅Π΄ΡΡ‚Π°Π²ΠΈΡ‚ΡŒ Π² Π²ΠΈΠ΄Π΅ Ρ‚Π°ΠΊ Π½Π°Π·Ρ‹Π²Π°Π΅ΠΌΠΎΠΉ Π²ΠΎΠ»ΡŒΡ‚-Π°ΠΌΠΏΠ΅Ρ€Π½ΠΎΠΉ характСристики . Как извСстно, прямая ΠΏΡ€ΠΎΠΏΠΎΡ€Ρ†ΠΈΠΎΠ½Π°Π»ΡŒΠ½Π°Ρ Π·Π°Π²ΠΈΡΠΈΠΌΠΎΡΡ‚ΡŒ ΠΌΠ΅ΠΆΠ΄Ρƒ двумя Π²Π΅Π»ΠΈΡ‡ΠΈΠ½Π°ΠΌΠΈ прСдставляСт собой ΠΏΡ€ΡΠΌΡƒΡŽ линию, ΠΏΡ€ΠΎΡ…ΠΎΠ΄ΡΡ‰ΡƒΡŽ Ρ‡Π΅Ρ€Π΅Π· Π½Π°Ρ‡Π°Π»ΠΎ ΠΊΠΎΠΎΡ€Π΄ΠΈΠ½Π°Ρ‚. Π’Π°ΠΊΡƒΡŽ Π·Π°Π²ΠΈΡΠΈΠΌΠΎΡΡ‚ΡŒ принято Π½Π°Π·Ρ‹Π²Π°Ρ‚ΡŒ Π»ΠΈΠ½Π΅ΠΉΠ½ΠΎΠΉ .

На рис. 2 ΠΏΠΎΠΊΠ°Π·Π°Π½ Π² качСствС ΠΏΡ€ΠΈΠΌΠ΅Ρ€Π° Π³Ρ€Π°Ρ„ΠΈΠΊ Π·Π°ΠΊΠΎΠ½Π° Ома для участка Ρ†Π΅ΠΏΠΈ с сопротивлСниСм 100 Ом. По Π³ΠΎΡ€ΠΈΠ·ΠΎΠ½Ρ‚Π°Π»ΡŒΠ½ΠΎΠΉ оси ΠΎΡ‚Π»ΠΎΠΆΠ΅Π½ΠΎ напряТСниС Π² Π²ΠΎΠ»ΡŒΡ‚Π°Ρ…, Π° ΠΏΠΎ Π²Π΅Ρ€Ρ‚ΠΈΠΊΠ°Π»ΡŒΠ½ΠΎΠΉ оси β€” Ρ‚ΠΎΠΊ Π² Π°ΠΌΠΏΠ΅Ρ€Π°Ρ…. ΠœΠ°ΡΡˆΡ‚Π°Π± Ρ‚ΠΎΠΊΠ° ΠΈ напряТСния ΠΌΠΎΠΆΠ΅Ρ‚ Π±Ρ‹Ρ‚ΡŒ Π²Ρ‹Π±Ρ€Π°Π½ ΠΊΠ°ΠΊΠΈΠΌ ΡƒΠ³ΠΎΠ΄Π½ΠΎ. ΠŸΡ€ΡΠΌΠ°Ρ линия ΠΏΡ€ΠΎΠ²Π΅Π΄Π΅Π½Π° Ρ‚Π°ΠΊ, Ρ‡Ρ‚ΠΎ для любой Π΅Π΅ Ρ‚ΠΎΡ‡ΠΊΠΈ ΠΎΡ‚Π½ΠΎΡˆΠ΅Π½ΠΈΠ΅ напряТСния ΠΊ Ρ‚ΠΎΠΊΡƒ Ρ€Π°Π²Π½ΠΎ 100 Ом. НапримСр, Ссли U = 50 Π’, Ρ‚ΠΎ I = 0,5 А ΠΈ R = 50 : 0,5 = 100 Ом.

Рис. 2 . Π—Π°ΠΊΠΎΠ½ Ома (Π²ΠΎΠ»ΡŒΡ‚-ампСрная характСристика)

Π“Ρ€Π°Ρ„ΠΈΠΊ Π·Π°ΠΊΠΎΠ½Π° Ома для ΠΎΡ‚Ρ€ΠΈΡ†Π°Ρ‚Π΅Π»ΡŒΠ½Ρ‹Ρ… Π·Π½Π°Ρ‡Π΅Π½ΠΈΠΉ Ρ‚ΠΎΠΊΠ° ΠΈ напряТСния ΠΈΠΌΠ΅Π΅Ρ‚ Ρ‚Π°ΠΊΠΎΠΉ ΠΆΠ΅ Π²ΠΈΠ΄. Π­Ρ‚ΠΎ Π³ΠΎΠ²ΠΎΡ€ΠΈΡ‚ ΠΎ Ρ‚ΠΎΠΌ, Ρ‡Ρ‚ΠΎ Ρ‚ΠΎΠΊ Π² Ρ†Π΅ΠΏΠΈ ΠΏΡ€ΠΎΡ…ΠΎΠ΄ΠΈΡ‚ ΠΎΠ΄ΠΈΠ½Π°ΠΊΠΎΠ²ΠΎ Π² ΠΎΠ±ΠΎΠΈΡ… направлСниях. Π§Π΅ΠΌ большС сопротивлСниС, Ρ‚Π΅ΠΌ мСньшС получаСтся Ρ‚ΠΎΠΊ ΠΏΡ€ΠΈ Π΄Π°Π½Π½ΠΎΠΌ напряТСнии ΠΈ Ρ‚Π΅ΠΌ Π±ΠΎΠ»Π΅Π΅ ΠΏΠΎΠ»ΠΎΠ³ΠΎ ΠΈΠ΄Π΅Ρ‚ прямая.

ΠŸΡ€ΠΈΠ±ΠΎΡ€Ρ‹, Ρƒ ΠΊΠΎΡ‚ΠΎΡ€Ρ‹Ρ… Π²ΠΎΠ»ΡŒΡ‚-ампСрная характСристика являСтся прямой Π»ΠΈΠ½ΠΈΠ΅ΠΉ, проходящСй Ρ‡Π΅Ρ€Π΅Π· Π½Π°Ρ‡Π°Π»ΠΎ ΠΊΠΎΠΎΡ€Π΄ΠΈΠ½Π°Ρ‚, Ρ‚. Π΅. сопротивлСниС остаСтся постоянным ΠΏΡ€ΠΈ ΠΈΠ·ΠΌΠ΅Π½Π΅Π½ΠΈΠΈ напряТСния ΠΈΠ»ΠΈ Ρ‚ΠΎΠΊΠ°, Π½Π°Π·Ρ‹Π²Π°ΡŽΡ‚ΡΡ Π»ΠΈΠ½Π΅ΠΉΠ½Ρ‹ΠΌΠΈ ΠΏΡ€ΠΈΠ±ΠΎΡ€Π°ΠΌΠΈ . ΠŸΡ€ΠΈΠΌΠ΅Π½ΡΡŽΡ‚ Ρ‚Π°ΠΊΠΆΠ΅ Ρ‚Π΅Ρ€ΠΌΠΈΠ½Ρ‹ Π»ΠΈΠ½Π΅ΠΉΠ½Ρ‹Π΅ Ρ†Π΅ΠΏΠΈ, Π»ΠΈΠ½Π΅ΠΉΠ½Ρ‹Π΅ сопротивлСния.

Π‘ΡƒΡ‰Π΅ΡΡ‚Π²ΡƒΡŽΡ‚ Ρ‚Π°ΠΊΠΆΠ΅ ΠΏΡ€ΠΈΠ±ΠΎΡ€Ρ‹, Ρƒ ΠΊΠΎΡ‚ΠΎΡ€Ρ‹Ρ… сопротивлСниС измСняСтся ΠΏΡ€ΠΈ ΠΈΠ·ΠΌΠ΅Π½Π΅Π½ΠΈΠΈ напряТСния ΠΈΠ»ΠΈ Ρ‚ΠΎΠΊΠ°. Π’ΠΎΠ³Π΄Π° Π·Π°Π²ΠΈΡΠΈΠΌΠΎΡΡ‚ΡŒ ΠΌΠ΅ΠΆΠ΄Ρƒ Ρ‚ΠΎΠΊΠΎΠΌ ΠΈ напряТСниСм выраТаСтся Π½Π΅ ΠΏΠΎ Π·Π°ΠΊΠΎΠ½Ρƒ Ома, Π° Π±ΠΎΠ»Π΅Π΅ слоТно. Для Ρ‚Π°ΠΊΠΈΡ… ΠΏΡ€ΠΈΠ±ΠΎΡ€ΠΎΠ² Π²ΠΎΠ»ΡŒΡ‚-ампСрная характСристика Π½Π΅ Π±ΡƒΠ΄Π΅Ρ‚ прямой Π»ΠΈΠ½ΠΈΠ΅ΠΉ, проходящСй Ρ‡Π΅Ρ€Π΅Π· Π½Π°Ρ‡Π°Π»ΠΎ ΠΊΠΎΠΎΡ€Π΄ΠΈΠ½Π°Ρ‚, Π° являСтся Π»ΠΈΠ±ΠΎ ΠΊΡ€ΠΈΠ²ΠΎΠΉ, Π»ΠΈΠ±ΠΎ Π»ΠΎΠΌΠ°Π½ΠΎΠΉ Π»ΠΈΠ½ΠΈΠ΅ΠΉ. Π­Ρ‚ΠΈ ΠΏΡ€ΠΈΠ±ΠΎΡ€Ρ‹ Π½Π°Π·Ρ‹Π²Π°ΡŽΡ‚ΡΡ Π½Π΅Π»ΠΈΠ½Π΅ΠΉΠ½Ρ‹ΠΌΠΈ .

Π—Π°ΠΊΠΎΠ½ Ома для участка Ρ†Π΅ΠΏΠΈ, бСзусловно, ΠΌΠΎΠΆΠ½ΠΎ ΠΎΠΏΠΈΡΠ°Ρ‚ΡŒ извСстной ΠΈΠ· школьного курса Ρ„ΠΈΠ·ΠΈΠΊΠΈ Ρ„ΠΎΡ€ΠΌΡƒΠ»ΠΎΠΉ: I=U/R , Π½ΠΎ Π½Π΅ΠΊΠΎΡ‚ΠΎΡ€Ρ‹Π΅ измСнСния ΠΈ уточнСния внСсти, Π΄ΡƒΠΌΠ°ΡŽ, стоит.

Π’ΠΎΠ·ΡŒΠΌΠ΅ΠΌ Π·Π°ΠΌΠΊΠ½ΡƒΡ‚ΡƒΡŽ ΡΠ»Π΅ΠΊΡ‚Ρ€ΠΈΡ‡Π΅ΡΠΊΡƒΡŽ Ρ†Π΅ΠΏΡŒ (рисунок 1) ΠΈ рассмотрим Π΅Π΅ участок ΠΌΠ΅ΠΆΠ΄Ρƒ Ρ‚ΠΎΡ‡ΠΊΠ°ΠΌΠΈ 1-2. Для простоты я взял участок элСктричСской Ρ†Π΅ΠΏΠΈ, Π½Π΅ содСрТащий источников Π­Π”Π‘ ( Π• ).

Π˜Ρ‚Π°ΠΊ, Π·Π°ΠΊΠΎΠ½ Ома для рассматриваСмого участка Ρ†Π΅ΠΏΠΈ ΠΈΠΌΠ΅Π΅Ρ‚ Π²ΠΈΠ΄:

  • I β€” Ρ‚ΠΎΠΊ, ΠΏΡ€ΠΎΡ‚Π΅ΠΊΠ°ΡŽΡ‰ΠΈΠΉ ΠΏΠΎ участку Ρ†Π΅ΠΏΠΈ.
  • R β€” сопротивлСниС этого участка.
  • Ο†1-Ο†2 β€” Ρ€Π°Π·Π½ΠΎΡΡ‚ΡŒ ΠΏΠΎΡ‚Π΅Π½Ρ†ΠΈΠ°Π»ΠΎΠ² ΠΌΠ΅ΠΆΠ΄Ρƒ Ρ‚ΠΎΡ‡ΠΊΠ°ΠΌΠΈ 1-2.
  • Если ΡƒΡ‡Π΅ΡΡ‚ΡŒ, Ρ‡Ρ‚ΠΎ Ρ€Π°Π·Π½ΠΎΡΡ‚ΡŒ ΠΏΠΎΡ‚Π΅Π½Ρ†ΠΈΠ°Π»ΠΎΠ² это напряТСниС, Ρ‚ΠΎ ΠΏΡ€ΠΈΡ…ΠΎΠ΄ΠΈΠΌ ΠΊ ΠΏΡ€ΠΎΠΈΠ·Π²ΠΎΠ΄Π½ΠΎΠΉ Ρ„ΠΎΡ€ΠΌΡƒΠ»Ρ‹ Π·Π°ΠΊΠΎΠ½Π° Ома, которая ΠΏΡ€ΠΈΠ²Π΅Π΄Π΅Π½Π° Π² Π½Π°Ρ‡Π°Π»Π΅ страницы: U=I*R

    Π­Ρ‚ΠΎ Ρ„ΠΎΡ€ΠΌΡƒΠ»Π° Π·Π°ΠΊΠΎΠ½Π° Ома для пассивного участка Ρ†Π΅ΠΏΠΈ (Π½Π΅ содСрТащСго источников элСктроэнСргии).

    Π’ Π½Π΅Ρ€Π°Π·Π²Π΅Ρ‚Π²Π»Π΅Π½Π½ΠΎΠΉ элСктричСской Ρ†Π΅ΠΏΠΈ (рис.2) сила Ρ‚ΠΎΠΊΠ° Π²ΠΎ всСх участках ΠΎΠ΄ΠΈΠ½Π°ΠΊΠΎΠ²Π°, Π° напряТСниС Π½Π° любом участкС опрСдСляСтся Π΅Π³ΠΎ сопротивлСниСм:

    • U1=I*R1
    • U2=I*R2
    • Un=I*Rn
    • U=I*(R1+R2+. +Rn

    ΠžΡ‚ΡΡŽΠ΄Π° ΠΌΠΎΠΆΠ½ΠΎ ΠΏΠΎΠ»ΡƒΡ‡ΠΈΡ‚ΡŒ Ρ„ΠΎΡ€ΠΌΡƒΠ»Ρ‹, ΠΊΠΎΡ‚ΠΎΡ€Ρ‹Π΅ пригодятся ΠΏΡ€ΠΈ практичСских вычислСниях. НапримСр:

    РасчСт слоТных (Ρ€Π°Π·Π²Π΅Ρ‚Π²Π»Π΅Π½Π½Ρ‹Ρ…) Ρ†Π΅ΠΏΠ΅ΠΉ осущСствляСтся с ΠΏΠΎΠΌΠΎΡ‰ΡŒΡŽ Π·Π°ΠΊΠΎΠ½ΠΎΠ² ΠšΠΈΡ€Ρ…Π³ΠΎΡ„Π°.

    ΠŸΠ ΠΠ’Π˜Π›Πž Π—ΠΠΠšΠžΠ’ Π”Π›Π― Π­Π”Π‘

    ΠŸΠ΅Ρ€Π΅Π΄ Ρ‚Π΅ΠΌ ΠΊΠ°ΠΊ Ρ€Π°ΡΡΠΌΠΎΡ‚Ρ€Π΅Ρ‚ΡŒ Π·Π°ΠΊΠΎΠ½ Ома для ΠΏΠΎΠ»Π½ΠΎΠΉ (Π·Π°ΠΌΠΊΠ½ΡƒΡ‚ΠΎΠΉ) Ρ†Π΅ΠΏΠΈ ΠΏΡ€ΠΈΠ²Π΅Π΄Ρƒ ΠΏΡ€Π°Π²ΠΈΠ»ΠΎ Π·Π½Π°ΠΊΠΎΠ² для Π­Π”Π‘, ΠΊΠΎΡ‚ΠΎΡ€ΠΎΠ΅ гласит:

    Если Π²Π½ΡƒΡ‚Ρ€ΠΈ источника Π­Π”Π‘ Ρ‚ΠΎΠΊ ΠΈΠ΄Π΅Ρ‚ ΠΎΡ‚ ΠΊΠ°Ρ‚ΠΎΠ΄Π° (-) ΠΊ Π°Π½ΠΎΠ΄Ρƒ (+) (Π½Π°ΠΏΡ€Π°Π²Π»Π΅Π½ΠΈΠ΅ напряТСнности поля сторонних сил совпадаСт с Π½Π°ΠΏΡ€Π°Π²Π»Π΅Π½ΠΈΠ΅ΠΌ Ρ‚ΠΎΠΊΠ° Π² Ρ†Π΅ΠΏΠΈ, Ρ‚ΠΎ Π­Π”Π‘ Ρ‚Π°ΠΊΠΎΠ³ΠΎ источника считаСтся ΠΏΠΎΠ»ΠΎΠΆΠΈΡ‚Π΅Π»ΡŒΠ½ΠΎΠΉ (рис.3.1). Π’ ΠΏΡ€ΠΎΡ‚ΠΈΠ²Π½ΠΎΠΌ случаС β€” Π­Π”Π‘ считаСтся ΠΎΡ‚Ρ€ΠΈΡ†Π°Ρ‚Π΅Π»ΡŒΠ½ΠΎΠΉ (рис.3.2).

    ΠŸΡ€Π°ΠΊΡ‚ΠΈΡ‡Π΅ΡΠΊΠΈΠΌ ΠΏΡ€ΠΈΠΌΠ΅Π½Π΅Π½ΠΈΠ΅ΠΌ этого ΠΏΡ€Π°Π²ΠΈΠ»Π° являСтся Π²ΠΎΠ·ΠΌΠΎΠΆΠ½ΠΎΡΡ‚ΡŒ привСдСния Π½Π΅ΡΠΊΠΎΠ»ΡŒΠΊΠΈΡ… источников Π­Π”Π‘ Π² Ρ†Π΅ΠΏΠΈ ΠΊ ΠΎΠ΄Π½ΠΎΠΌΡƒ с Π²Π΅Π»ΠΈΡ‡ΠΈΠ½ΠΎΠΉ E=E1+E2+. +En , СстСствСнно, с ΡƒΡ‡Π΅Ρ‚ΠΎΠΌ Π·Π½Π°ΠΊΠΎΠ², опрСдСляСмых ΠΏΠΎ Π²Ρ‹ΡˆΠ΅ΠΏΡ€ΠΈΠ²Π΅Π΄Π΅Π½Π½ΠΎΠΌΡƒ ΠΏΡ€Π°Π²ΠΈΠ»Ρƒ. НапримСр (рис.3.3) E=E1+E2-E3 .

    ΠŸΡ€ΠΈ отсутствии встрСчно Π²ΠΊΠ»ΡŽΡ‡Π΅Π½Π½ΠΎΠ³ΠΎ источника E3 (Π½Π° ΠΏΡ€Π°ΠΊΡ‚ΠΈΠΊΠ΅ Ρ‚Π°ΠΊ ΠΏΠΎΡ‡Ρ‚ΠΈ Π½ΠΈΠΊΠΎΠ³Π΄Π° Π½Π΅ Π±Ρ‹Π²Π°Π΅Ρ‚) ΠΈΠΌΠ΅Π΅ΠΌ ΡˆΠΈΡ€ΠΎΠΊΠΎ распространСнноС ΠΏΠΎΡΠ»Π΅Π΄ΠΎΠ²Π°Ρ‚Π΅Π»ΡŒΠ½ΠΎΠ΅ Π²ΠΊΠ»ΡŽΡ‡Π΅Π½ΠΈΠ΅ элСмСнтов питания, ΠΏΡ€ΠΈ ΠΊΠΎΡ‚ΠΎΡ€ΠΎΠΌ ΠΈΡ… напряТСния ΡΡƒΠΌΠΌΠΈΡ€ΡƒΡŽΡ‚ΡΡ.

    Π—ΠΠšΠžΠ ОМА Π”Π›Π― ΠŸΠžΠ›ΠΠžΠ™ Π¦Π•ΠŸΠ˜

    Π—Π°ΠΊΠΎΠ½ Ома для ΠΏΠΎΠ»Π½ΠΎΠΉ Ρ†Π΅ΠΏΠΈ β€” Π΅Π³ΠΎ Π΅Ρ‰Π΅ ΠΌΠΎΠΆΠ½ΠΎ Π½Π°Π·Π²Π°Ρ‚ΡŒ Π·Π°ΠΊΠΎΠ½ ΠΎΠΌΠ° для Π·Π°ΠΌΠΊΠ½ΡƒΡ‚ΠΎΠΉ Ρ†Π΅ΠΏΠΈ, ΠΈΠΌΠ΅Π΅Ρ‚ Π²ΠΈΠ΄ I=E/(R+r) .

    ΠŸΡ€ΠΈΠ²Π΅Π΄Π΅Π½Π½Π°Ρ Ρ„ΠΎΡ€ΠΌΡƒΠ»Π° Π·Π°ΠΊΠΎΠ½Π° Ома содСрТит ΠΎΠ±ΠΎΠ·Π½Π°Ρ‡Π΅Π½ΠΈΠ΅ r , ΠΊΠΎΡ‚ΠΎΡ€ΠΎΠ΅ Π΅Ρ‰Π΅ Π½Π΅ ΡƒΠΏΠΎΠΌΠΈΠ½Π°Π»ΠΎΡΡŒ. Π­Ρ‚ΠΎ Π²Π½ΡƒΡ‚Ρ€Π΅Π½Π½Π΅Π΅ сопротивлСниС источника Π­Π”Π‘. Оно достаточно ΠΌΠ°Π»ΠΎ, Π² Π±ΠΎΠ»ΡŒΡˆΠΈΠ½ΡΡ‚Π²Π΅ случаСв ΠΏΡ€ΠΈ практичСских расчСтах ΠΈΠΌ ΠΌΠΎΠΆΠ½ΠΎ ΠΏΡ€Π΅Π½Π΅Π±Ρ€Π΅Ρ‡ΡŒ (ΠΏΡ€ΠΈ условии, Ρ‡Ρ‚ΠΎ R>>r β€” сопротивлСниС Ρ†Π΅ΠΏΠΈ ΠΌΠ½ΠΎΠ³ΠΎ большС Π²Π½ΡƒΡ‚Ρ€Π΅Π½Π½Π΅Π³ΠΎ сопротивлСния источника). Однако, ΠΊΠΎΠ³Π΄Π° ΠΎΠ½ΠΈ соизмСримы, ΠΏΡ€Π΅Π½Π΅Π±Ρ€Π΅Π³Π°Ρ‚ΡŒ Π²Π΅Π»ΠΈΡ‡ΠΈΠ½ΠΎΠΉ r нСльзя.

    Как Π²Π°Ρ€ΠΈΠ°Π½Ρ‚ ΠΌΠΎΠΆΠ½ΠΎ Ρ€Π°ΡΡΠΌΠΎΡ‚Ρ€Π΅Ρ‚ΡŒ случай, ΠΏΡ€ΠΈ ΠΊΠΎΡ‚ΠΎΡ€ΠΎΠΌ R=0 (ΠΊΠΎΡ€ΠΎΡ‚ΠΊΠΎΠ΅ Π·Π°ΠΌΡ‹ΠΊΠ°Π½ΠΈΠ΅). Π’ΠΎΠ³Π΄Π° привСдСнная Ρ„ΠΎΡ€ΠΌΡƒΠ»Π° Π·Π°ΠΊΠΎΠ½Π° Ома для ΠΏΠΎΠ»Π½ΠΎΠΉ Ρ†Π΅ΠΏΠΈ ΠΏΡ€ΠΈΠΌΠ΅Ρ‚ Π²ΠΈΠ΄: I=E/r , Ρ‚ΠΎ Π΅ΡΡ‚ΡŒ Π²Π΅Π»ΠΈΡ‡ΠΈΠ½Π° Π²Π½ΡƒΡ‚Ρ€Π΅Π½Π½Π΅Π³ΠΎ сопротивлСния Π±ΡƒΠ΄Π΅Ρ‚ ΠΎΠΏΡ€Π΅Π΄Π΅Π»ΡΡ‚ΡŒ Ρ‚ΠΎΠΊ ΠΊΠΎΡ€ΠΎΡ‚ΠΊΠΎΠ³ΠΎ замыкания. Вакая ситуация Π²ΠΏΠΎΠ»Π½Π΅ ΠΌΠΎΠΆΠ΅Ρ‚ Π±Ρ‹Ρ‚ΡŒ Ρ€Π΅Π°Π»ΡŒΠ½ΠΎΠΉ.

    Π—Π°ΠΊΠΎΠ½ Ома рассмотрСн здСсь достоточно Π±Π΅Π³Π»ΠΎ, Π½ΠΎ ΠΏΡ€ΠΈΠ²Π΅Π΄Π΅Π½Π½Ρ‹Ρ… Ρ„ΠΎΡ€ΠΌΡƒΠ» достаточно для провСдСния Π±ΠΎΠ»ΡŒΡˆΠΈΠ½ΡΡ‚Π²Π° расчСтов, ΠΏΡ€ΠΈΠΌΠ΅Ρ€Ρ‹ ΠΊΠΎΡ‚ΠΎΡ€Ρ‹Ρ…, ΠΏΠΎ ΠΌΠ΅Ρ€Π΅ размСщСния Π΄Ρ€ΡƒΠ³ΠΈΡ… ΠΌΠ°Ρ‚Π΅Ρ€ΠΈΠ°Π»ΠΎΠ² я Π±ΡƒΠ΄Ρƒ ΠΏΡ€ΠΈΠ²ΠΎΠ΄ΠΈΡ‚ΡŒ.

    Β© 2012-2018 Π³. ВсС ΠΏΡ€Π°Π²Π° Π·Π°Ρ‰ΠΈΡ‰Π΅Π½Ρ‹.

    ВсС прСдставлСнныС Π½Π° этом сайтС ΠΌΠ°Ρ‚Π΅Ρ€ΠΈΠ°Π»Ρ‹ ΠΈΠΌΠ΅ΡŽΡ‚ ΠΈΡΠΊΠ»ΡŽΡ‡ΠΈΡ‚Π΅Π»ΡŒΠ½ΠΎ ΠΈΠ½Ρ„ΠΎΡ€ΠΌΠ°Ρ†ΠΈΠΎΠ½Π½Ρ‹ΠΉ Ρ…Π°Ρ€Π°ΠΊΡ‚Π΅Ρ€ ΠΈ Π½Π΅ ΠΌΠΎΠ³ΡƒΡ‚ Π±Ρ‹Ρ‚ΡŒ ΠΈΡΠΏΠΎΠ»ΡŒΠ·ΠΎΠ²Π°Π½Ρ‹ Π² качСствС руководящих ΠΈ Π½ΠΎΡ€ΠΌΠ°Ρ‚ΠΈΠ²Π½Ρ‹Ρ… Π΄ΠΎΠΊΡƒΠΌΠ΅Π½Ρ‚ΠΎΠ²

    eltechbook.ru

    Π—Π°ΠΊΠΎΠ½Ρ‹ Ома ΠΈ ΠšΠΈΡ€Ρ…Π³ΠΎΡ„Π°

    Π—Π°ΠΊΠΎΠ½ Ома являСтся основным Π·Π°ΠΊΠΎΠ½ΠΎΠΌ, ΠΊΠΎΡ‚ΠΎΡ€Ρ‹ΠΉ ΠΈΡΠΏΠΎΠ»ΡŒΠ·ΡƒΡŽΡ‚ ΠΏΡ€ΠΈ расчСтах Ρ†Π΅ΠΏΠ΅ΠΉ постоянного Ρ‚ΠΎΠΊΠ°. Он являСтся Ρ„ΡƒΠ½Π΄Π°ΠΌΠ΅Π½Ρ‚Π°Π»ΡŒΠ½Ρ‹ΠΌ ΠΈ ΠΌΠΎΠΆΠ΅Ρ‚ ΠΏΡ€ΠΈΠΌΠ΅Π½ΡΡ‚ΡŒΡΡ для Π»ΡŽΠ±Ρ‹Ρ… физичСских систСм, Π³Π΄Π΅ Π΅ΡΡ‚ΡŒ ΠΏΠΎΡ‚ΠΎΠΊΠΈ частиц ΠΈ поля, прСодолСваСтся сопротивлСниС.

    Π—Π°ΠΊΠΎΠ½Ρ‹ ΠΈΠ»ΠΈ ΠΏΡ€Π°Π²ΠΈΠ»Π° ΠšΠΈΡ€Ρ…Π³ΠΎΡ„Π° ΡΠ²Π»ΡΡŽΡ‚ΡΡ ΠΏΡ€ΠΈΠ»ΠΎΠΆΠ΅Π½ΠΈΠ΅ΠΌ ΠΊ Π·Π°ΠΊΠΎΠ½Ρƒ Ома, ΠΈΡΠΏΠΎΠ»ΡŒΠ·ΡƒΠ΅ΠΌΡ‹ΠΌ для расчСта слоТных элСктричСских Ρ†Π΅ΠΏΠ΅ΠΉ постоянного Ρ‚ΠΎΠΊΠ°.

    ΠžΠ±ΠΎΠ±Ρ‰Π΅Π½Π½Ρ‹ΠΉ Π·Π°ΠΊΠΎΠ½ Ома для Π½Π΅ΠΎΠ΄Π½ΠΎΡ€ΠΎΠ΄Π½ΠΎΠ³ΠΎ участка Ρ†Π΅ΠΏΠΈ (участка Ρ†Π΅ΠΏΠΈ, содСрТащСго источник Π­Π”Π‘) ΠΈΠΌΠ΅Π΅Ρ‚ Π²ΠΈΠ΄:

    – Ρ€Π°Π·Π½ΠΎΡΡ‚ΡŒ ΠΏΠΎΡ‚Π΅Π½Ρ†ΠΈΠ°Π»ΠΎΠ² Π½Π° ΠΊΠΎΠ½Ρ†Π°Ρ… участка Ρ†Π΅ΠΏΠΈ; – Π­Π”Π‘ источника Π½Π° рассматриваСмом участкС Ρ†Π΅ΠΏΠΈ; R – внСшнСС сопротивлСниС Ρ†Π΅ΠΏΠΈ; r – Π²Π½ΡƒΡ‚Ρ€Π΅Π½Π½Π΅Π΅ сопротивлСниС источника Π­Π”Π‘. Если Ρ†Π΅ΠΏΡŒ Ρ€Π°Π·ΠΎΠΌΠΊΠ½ΡƒΡ‚Π°, Π·Π½Π°Ρ‡ΠΈΡ‚, Ρ‚ΠΎΠΊΠ° Π² Π½Π΅ΠΉ Π½Π΅Ρ‚ (), Ρ‚ΠΎ ΠΈΠ· (2) ΠΏΠΎΠ»ΡƒΡ‡ΠΈΠΌ:

    Π­Π”Π‘, Π΄Π΅ΠΉΡΡ‚Π²ΡƒΡŽΡ‰Π°Ρ Π² Π½Π΅Π·Π°ΠΌΠΊΠ½ΡƒΡ‚ΠΎΠΉ Ρ†Π΅ΠΏΠΈ, Ρ€Π°Π²Π½Π° разности ΠΏΠΎΡ‚Π΅Π½Ρ†ΠΈΠ°Π»ΠΎΠ² Π½Π° Π΅Π΅ ΠΊΠΎΠ½Ρ†Π°Ρ…. ΠŸΠΎΠ»ΡƒΡ‡Π°Π΅Ρ‚ΡΡ, для нахоТдСния Π­Π”Π‘ источника слСдуСт ΠΈΠ·ΠΌΠ΅Ρ€ΠΈΡ‚ΡŒ Ρ€Π°Π·Π½ΠΎΡΡ‚ΡŒ ΠΏΠΎΡ‚Π΅Π½Ρ†ΠΈΠ°Π»ΠΎΠ² Π½Π° Π΅Π³ΠΎ ΠΊΠ»Π΅ΠΌΠΌΠ°Ρ… ΠΏΡ€ΠΈ Π½Π΅Π·Π°ΠΌΠΊΠ½ΡƒΡ‚ΠΎΠΉ Ρ†Π΅ΠΏΠΈ.

    Π—Π°ΠΊΠΎΠ½ Ома для Π·Π°ΠΌΠΊΠ½ΡƒΡ‚ΠΎΠΉ Ρ†Π΅ΠΏΠΈ Π·Π°ΠΏΠΈΡΡ‹Π²Π°ΡŽΡ‚ ΠΊΠ°ΠΊ:

    Π’Π΅Π»ΠΈΡ‡ΠΈΠ½Ρƒ ΠΈΠ½ΠΎΠ³Π΄Π° Π½Π°Π·Ρ‹Π²Π°ΡŽΡ‚ ΠΏΠΎΠ»Π½Ρ‹ΠΌ сопротивлСниСм Ρ†Π΅ΠΏΠΈ. Π€ΠΎΡ€ΠΌΡƒΠ»Π° (2) ΠΏΠΎΠΊΠ°Π·Ρ‹Π²Π°Π΅Ρ‚, Ρ‡Ρ‚ΠΎ элСктродвиТущая сила источника Ρ‚ΠΎΠΊΠ°, дСлСнная Π½Π° ΠΏΠΎΠ»Π½ΠΎΠ΅ сопротивлСниС Ρ€Π°Π²Π½Π° силС Ρ‚ΠΎΠΊΠ° Π² Ρ†Π΅ΠΏΠΈ.

    Π—Π°ΠΊΠΎΠ½ ΠšΠΈΡ€Ρ…Π³ΠΎΡ„Π°

    ΠŸΡƒΡΡ‚ΡŒ имССтся ΠΏΡ€ΠΎΠΈΠ·Π²ΠΎΠ»ΡŒΠ½Π°Ρ развСтвлСнная ΡΠ΅Ρ‚ΡŒ ΠΏΡ€ΠΎΠ²ΠΎΠ΄Π½ΠΈΠΊΠΎΠ². Π’ ΠΎΡ‚Π΄Π΅Π»ΡŒΠ½Ρ‹Ρ… участках Π²ΠΊΠ»ΡŽΡ‡Π΅Π½Ρ‹ Ρ€Π°Π·Π½ΠΎΠΎΠ±Ρ€Π°Π·Π½Ρ‹Π΅ источники Ρ‚ΠΎΠΊΠ°. Π­Π”Π‘ источников постоянны ΠΈ Π±ΡƒΠ΄Π΅ΠΌ ΡΡ‡ΠΈΡ‚Π°Ρ‚ΡŒ извСстными. ΠŸΡ€ΠΈ этом Ρ‚ΠΎΠΊΠΈ Π²ΠΎ всСх участках Ρ†Π΅ΠΏΠΈ ΠΈ разности ΠΏΠΎΡ‚Π΅Π½Ρ†ΠΈΠ°Π»ΠΎΠ² Π½Π° Π½ΠΈΡ… ΠΌΠΎΠΆΠ½ΠΎ Π²Ρ‹Ρ‡ΠΈΡΠ»ΠΈΡ‚ΡŒ ΠΏΡ€ΠΈ ΠΏΠΎΠΌΠΎΡ‰ΠΈ Π·Π°ΠΊΠΎΠ½Π° Ома ΠΈ Π·Π°ΠΊΠΎΠ½Π° сохранСния заряда.

    Для упрощСния Ρ€Π΅ΡˆΠ΅Π½ΠΈΡ Π·Π°Π΄Π°Ρ‡ ΠΏΠΎ расчСтам Ρ€Π°Π·Π²Π΅Ρ‚Π²Π»Ρ‘Π½Π½Ρ‹Ρ… элСктричСских Ρ†Π΅ΠΏΠ΅ΠΉ, ΠΈΠΌΠ΅ΡŽΡ‰ΠΈΡ… нСсколько Π·Π°ΠΌΠΊΠ½ΡƒΡ‚Ρ‹Ρ… ΠΊΠΎΠ½Ρ‚ΡƒΡ€ΠΎΠ², нСсколько источников Π­Π”Π‘, ΠΈΡΠΏΠΎΠ»ΡŒΠ·ΡƒΡŽΡ‚ Π·Π°ΠΊΠΎΠ½Ρ‹ (ΠΈΠ»ΠΈ ΠΏΡ€Π°Π²ΠΈΠ»Π°) ΠšΠΈΡ€Ρ…Π³ΠΎΡ„Π°. ΠŸΡ€Π°Π²ΠΈΠ»Π° ΠšΠΈΡ€Ρ…Π³ΠΎΡ„Π° слуТат для Ρ‚ΠΎΠ³ΠΎ, Ρ‡Ρ‚ΠΎΠ±Ρ‹ ΡΠΎΡΡ‚Π°Π²ΠΈΡ‚ΡŒ систСму ΡƒΡ€Π°Π²Π½Π΅Π½ΠΈΠΉ, ΠΈΠ· ΠΊΠΎΡ‚ΠΎΡ€ΠΎΠΉ находят силы Ρ‚ΠΎΠΊΠ° Π² элСмСнтах слоТной Ρ€Π°Π·Π²Π΅Ρ‚Π²Π»Π΅Π½Π½ΠΎΠΉ Ρ†Π΅ΠΏΠΈ.

    Π‘ΡƒΠΌΠΌΠ° Ρ‚ΠΎΠΊΠΎΠ² Π² ΡƒΠ·Π»Π΅ Ρ†Π΅ΠΏΠΈ с ΡƒΡ‡Π΅Ρ‚ΠΎΠΌ ΠΈΡ… Π·Π½Π°ΠΊΠΎΠ² Ρ€Π°Π²Π½Π° Π½ΡƒΠ»ΡŽ:

    ΠŸΠ΅Ρ€Π²ΠΎΠ΅ ΠΏΡ€Π°Π²ΠΈΠ»ΠΎ ΠšΠΈΡ€Ρ…Π³ΠΎΡ„Π° являСтся слСдствиСм Π·Π°ΠΊΠΎΠ½Π° сохранСния элСктричСского заряда. АлгСбраичСская сумма Ρ‚ΠΎΠΊΠΎΠ², сходящихся Π² любом ΡƒΠ·Π»Π΅ Ρ†Π΅ΠΏΠΈ – это заряд, ΠΊΠΎΡ‚ΠΎΡ€Ρ‹ΠΉ ΠΏΡ€ΠΈΡ…ΠΎΠ΄ΠΈΡ‚ Π² ΡƒΠ·Π΅Π» Π·Π° Π΅Π΄ΠΈΠ½ΠΈΡ†Ρƒ Π²Ρ€Π΅ΠΌΠ΅Π½ΠΈ.

    ΠŸΡ€ΠΈ составлСнии ΡƒΡ€Π°Π²Π½Π΅Π½ΠΈΠ΅ ΠΈΡΠΏΠΎΠ»ΡŒΠ·ΡƒΡ Π·Π°ΠΊΠΎΠ½Ρ‹ ΠšΠΈΡ€Ρ…Π³ΠΎΡ„Π° Π²Π°ΠΆΠ½ΠΎ ΡƒΡ‡ΠΈΡ‚Ρ‹Π²Π°Ρ‚ΡŒ Π·Π½Π°ΠΊΠΈ с ΠΊΠΎΡ‚ΠΎΡ€Ρ‹ΠΌΠΈ силы Ρ‚ΠΎΠΊΠΎΠ² входят Π² эти уравнСния. Π‘Π»Π΅Π΄ΡƒΠ΅Ρ‚ ΡΡ‡ΠΈΡ‚Π°Ρ‚ΡŒ, Ρ‡Ρ‚ΠΎ Ρ‚ΠΎΠΊΠΈ, ΠΈΠ΄ΡƒΡ‰ΠΈΠ΅ ΠΊ Ρ‚ΠΎΡ‡ΠΊΠ΅ развСтвлСния, ΠΈ исходящиС ΠΎΡ‚ развСтвлСния ΠΈΠΌΠ΅ΡŽΡ‚ ΠΏΡ€ΠΎΡ‚ΠΈΠ²ΠΎΠΏΠΎΠ»ΠΎΠΆΠ½Ρ‹Π΅ Π·Π½Π°ΠΊΠΈ. ΠŸΡ€ΠΈ этом Π½ΡƒΠΆΠ½ΠΎ для сСбя ΠΎΠΏΡ€Π΅Π΄Π΅Π»ΠΈΡ‚ΡŒ ΠΊΠ°ΠΊΠΎΠ΅ Π½Π°ΠΏΡ€Π°Π²Π»Π΅Π½ΠΈΠ΅ (ΠΊ ΡƒΠ·Π»Ρƒ ΠΈΠ»ΠΈ ΠΎΡ‚ ΡƒΠ·Π»Π°) ΡΡ‡ΠΈΡ‚Π°Ρ‚ΡŒ ΠΏΠΎΠ»ΠΎΠΆΠΈΡ‚Π΅Π»ΡŒΠ½Ρ‹ΠΌ.

    ΠŸΡ€ΠΎΠΈΠ·Π²Π΅Π΄Π΅Π½ΠΈΠ΅ алгСбраичСской Π²Π΅Π»ΠΈΡ‡ΠΈΠ½Ρ‹ силы Ρ‚ΠΎΠΊΠ° (I) Π½Π° сумму Π²Π΅ΡˆΠ½ΠΈΡ… ΠΈ Π²Π½ΡƒΡ‚Ρ€Π΅Π½Π½ΠΈΡ… сопротивлСний всСх участков Π·Π°ΠΌΠΊΠ½ΡƒΡ‚ΠΎΠ³ΠΎ ΠΊΠΎΠ½Ρ‚ΡƒΡ€Π° Ρ€Π°Π²Π½ΠΎ суммС алгСбраичСских Π·Π½Π°Ρ‡Π΅Π½ΠΈΠΉ сторонних Π­Π”Π‘ () рассматриваСмого ΠΊΠΎΠ½Ρ‚ΡƒΡ€Π°:

    КаТдоС ΠΏΡ€ΠΎΠΈΠ·Π²Π΅Π΄Π΅Π½ΠΈΠ΅ опрСдСляСт Ρ€Π°Π·Π½ΠΎΡΡ‚ΡŒ ΠΏΠΎΡ‚Π΅Π½Ρ†ΠΈΠ°Π»ΠΎΠ², которая сущСствовала Π±Ρ‹ ΠΌΠ΅ΠΆΠ΄Ρƒ ΠΊΠΎΠ½Ρ†Π°ΠΌΠΈ ΡΠΎΠΎΡ‚Π²Π΅Ρ‚ΡΡ‚Π²ΡƒΡŽΡ‰Π΅Π³ΠΎ участка, Ссли Π±Ρ‹ Π­Π”Π‘ Π² Π½Π΅ΠΌ Π±Ρ‹Π»Π° Ρ€Π°Π²Π½ΠΎ Π½ΡƒΠ»ΡŽ. Π’Π΅Π»ΠΈΡ‡ΠΈΠ½Ρƒ Π½Π°Π·Ρ‹Π²Π°ΡŽΡ‚ ΠΏΠ°Π΄Π΅Π½ΠΈΠ΅ΠΌ напряТСния, ΠΊΠΎΡ‚ΠΎΡ€ΠΎΠ΅ вызываСтся Ρ‚ΠΎΠΊΠΎΠΌ.

    Π’Ρ‚ΠΎΡ€ΠΎΠΉ Π·Π°ΠΊΠΎΠ½ ΠšΠΈΡ€Ρ…Π³ΠΎΡ„Π° ΠΈΠ½ΠΎΠ³Π΄Π° Ρ„ΠΎΡ€ΠΌΡƒΠ»ΠΈΡ€ΡƒΡŽΡ‚ ΡΠ»Π΅Π΄ΡƒΡŽΡ‰ΠΈΠΌ ΠΎΠ±Ρ€Π°Π·ΠΎΠΌ:

    Для Π·Π°ΠΌΠΊΠ½ΡƒΡ‚ΠΎΠ³ΠΎ ΠΊΠΎΠ½Ρ‚ΡƒΡ€Π° сумма ΠΏΠ°Π΄Π΅Π½ΠΈΠΉ напряТСния Π΅ΡΡ‚ΡŒ сума Π­Π”Π‘ Π² рассматриваСмом ΠΊΠΎΠ½Ρ‚ΡƒΡ€Π΅.

    Π’Ρ‚ΠΎΡ€ΠΎΠ΅ ΠΏΡ€Π°Π²ΠΈΠ»ΠΎ (Π·Π°ΠΊΠΎΠ½) ΠšΠΈΡ€Ρ…Π³ΠΎΡ„Π° являСтся слСдствиСм ΠΎΠ±ΠΎΠ±Ρ‰Π΅Π½Π½ΠΎΠ³ΠΎ Π·Π°ΠΊΠΎΠ½Π° Ома. Π’Π°ΠΊ, Ссли Π² ΠΈΠ·ΠΎΠ»ΠΈΡ€ΠΎΠ²Π°Π½Π½ΠΎΠΉ Π·Π°ΠΌΠΊΠ½ΡƒΡ‚ΠΎΠΉ Ρ†Π΅ΠΏΠΈ Π΅ΡΡ‚ΡŒ ΠΎΠ΄ΠΈΠ½ источник Π­Π”Π‘, Ρ‚ΠΎ сила Ρ‚ΠΎΠΊΠ° Π² Ρ†Π΅ΠΏΠΈ Π±ΡƒΠ΄Π΅Ρ‚ Ρ‚Π°ΠΊΠΎΠΉ, Ρ‡Ρ‚ΠΎ сумма падСния напряТСния Π½Π° внСшнСм сопротивлСнии ΠΈ Π²Π½ΡƒΡ‚Ρ€Π΅Π½Π½Π΅ΠΌ сопротивлСнии источника Π±ΡƒΠ΄Π΅Ρ‚ Ρ€Π°Π²Π½Π° стороннСй Π­Π”Π‘ источника. Если источников Π­Π”Π‘ нСсколько, Ρ‚ΠΎ Π±Π΅Ρ€ΡƒΡ‚ ΠΈΡ… Π°Π»Π³Π΅Π±Ρ€Π°ΠΈΡ‡Π΅ΡΠΊΡƒΡŽ сумму. Π—Π½Π°ΠΊ Π­Π”Π‘ выбираСтся ΠΏΠΎΠ»ΠΎΠΆΠΈΡ‚Π΅Π»ΡŒΠ½Ρ‹ΠΌ, Ссли ΠΏΡ€ΠΈ Π΄Π²ΠΈΠΆΠ΅Π½ΠΈΠΈ ΠΏΠΎ ΠΊΠΎΠ½Ρ‚ΡƒΡ€Ρƒ Π² ΠΏΠΎΠ»ΠΎΠΆΠΈΡ‚Π΅Π»ΡŒΠ½ΠΎΠΌ Π½Π°ΠΏΡ€Π°Π²Π»Π΅Π½ΠΈΠΈ ΠΏΠ΅Ρ€Π²Ρ‹ΠΌ встрСчаСтся ΠΎΡ‚Ρ€ΠΈΡ†Π°Ρ‚Π΅Π»ΡŒΠ½Ρ‹ΠΉ полюс источника. (Π—Π° ΠΏΠΎΠ»ΠΎΠΆΠΈΡ‚Π΅Π»ΡŒΠ½ΠΎΠ΅ Π½Π°ΠΏΡ€Π°Π²Π»Π΅Π½ΠΈΠ΅ ΠΎΠ±Ρ…ΠΎΠ΄Π° ΠΊΠΎΠ½Ρ‚ΡƒΡ€Π° ΠΏΡ€ΠΈΠ½ΠΈΠΌΠ°ΡŽΡ‚ Π½Π°ΠΏΡ€Π°Π²Π»Π΅Π½ΠΈΠ΅ ΠΎΠ±Ρ…ΠΎΠ΄Π° Ρ†Π΅ΠΏΠΈ Π»ΠΈΠ±ΠΎ ΠΏΠΎ часовой стрСлкС, Π»ΠΈΠ±ΠΎ ΠΏΡ€ΠΎΡ‚ΠΈΠ² Π½Π΅Π΅).

    ΠŸΡ€ΠΈΠΌΠ΅Ρ€Ρ‹ Ρ€Π΅ΡˆΠ΅Π½ΠΈΡ Π·Π°Π΄Π°Ρ‡

    Π’ΠΎ Π²Ρ‚ΠΎΡ€ΠΎΠΌ случаС:

    Π‘ΠΈΠ»Π° Ρ‚ΠΎΠΊΠ° Π² любом мСстС Ρ†Π΅ΠΏΠΈ рис.1(Π°) Ρ€Π°Π²Π½Π° , ΡΠ»Π΅Π΄ΠΎΠ²Π°Ρ‚Π΅Π»ΡŒΠ½ΠΎ, напряТСниС, ΠΊΠΎΡ‚ΠΎΡ€ΠΎΠ΅ ΠΏΠΎΠΊΠ°Π·Ρ‹Π²Π°Π΅Ρ‚ Π²ΠΎΠ»ΡŒΡ‚ΠΌΠ΅Ρ‚Ρ€ Π² ΠΏΠ΅Ρ€Π²ΠΎΠΌ случаС Ρ€Π°Π²Π½ΠΎ:

    Π’ΠΎ Π²Ρ‚ΠΎΡ€ΠΎΠΌ случаС, ΠΈΠΌΠ΅Π΅ΠΌ:

    ΠŸΡ€ΠΈΡ€Π°Π²Π½ΡΠ΅ΠΌ Π»Π΅Π²Ρ‹Π΅ части Π²Ρ‹Ρ€Π°ΠΆΠ΅Π½ΠΈΠΉ (1.4) ΠΈ (1.5):

    Из Ρ„ΠΎΡ€ΠΌΡƒΠ»Ρ‹ (1.6), Π²Ρ‹Ρ€Π°Π·ΠΈΠΌ искомоС сопротивлСниС:

    ΠŸΡ€ΠΈΠΌΠ΅ΠΌ Π·Π° Π½Π°ΠΏΡ€Π°Π²Π»Π΅Π½ΠΈΠ΅ ΠΎΠ±Ρ…ΠΎΠ΄Π° Π΄Π²ΠΈΠΆΠ΅Π½ΠΈΠ΅ ΠΏΠΎ часовой стрСлкС. Рассмотрим ΠΊΠΎΠ½Ρ‚ΡƒΡ€ . По Π²Ρ‚ΠΎΡ€ΠΎΠΌΡƒ ΠΏΡ€Π°Π²ΠΈΠ»Ρƒ ΠšΠΈΡ€Ρ…Π³ΠΎΡ„Π° запишСм:

    Рассмотрим ΠΊΠΎΠ½Ρ‚ΡƒΡ€ ADFK, ΠΈΠΌΠ΅Π΅ΠΌ:

    Π˜ΡΠΏΠΎΠ»ΡŒΠ·ΡƒΡ систСму ΠΈΠ· Ρ‚Ρ€Π΅Ρ… ΡƒΡ€Π°Π²Π½Π΅Π½ΠΈΠ΅(2.1 -2.3), Π²Ρ‹Ρ€Π°Π·ΠΈΠΌ силу Ρ‚ΠΎΠΊΠ° I, ΠΏΠΎΠ»ΡƒΡ‡Π°Π΅ΠΌ:

    ru.solverbook.com

    1.7 ΠŸΡ€ΠΈΠΌΠ΅Π½Π΅Π½ΠΈΠ΅ Π·Π°ΠΊΠΎΠ½Π° ΠΎΠΌΠ° ΠΈ Π·Π°ΠΊΠΎΠ½ΠΎΠ² ΠΊΠΈΡ€Ρ…Π³ΠΎΡ„Π° для расчСтов элСктричСских Ρ†Π΅ΠΏΠ΅ΠΉ

    ΠŸΡ€ΠΈ ΠΏΠΎΠΌΠΎΡ‰ΠΈ Π·Π°ΠΊΠΎΠ½Π° Ома ΠΈ Π΄Π²ΡƒΡ… Π·Π°ΠΊΠΎΠ½ΠΎΠ² ΠšΠΈΡ€Ρ…Π³ΠΎΡ„Π° ΠΌΠΎΠΆΠ½ΠΎ Ρ€Π°ΡΡΡ‡ΠΈΡ‚Π°Ρ‚ΡŒ Ρ€Π΅ΠΆΠΈΠΌ Ρ€Π°Π±ΠΎΡ‚Ρ‹ элСктричСской Ρ†Π΅ΠΏΠΈ любой слоТности.

    ΠžΠ±Ρ‰Π΅ΠΉ Π·Π°Π΄Π°Ρ‡Π΅ΠΉ расчСта являСтся ΠΎΠΏΡ€Π΅Π΄Π΅Π»Π΅Π½ΠΈΠ΅ Ρ‚ΠΎΠΊΠΎΠ² Π²ΠΎ всСх участ­ках Ρ†Π΅ΠΏΠΈ ΠΏΡ€ΠΈ Π·Π°Π΄Π°Π½Π½Ρ‹Ρ… ΠΏΠ°Ρ€Π°ΠΌΠ΅Ρ‚Ρ€Π°Ρ… элСмСнтов Ρ†Π΅ΠΏΠΈ ΠΈ извСстной ΠΊΠΎΠ½Β­Ρ„ΠΈΠ³ΡƒΡ€Π°Ρ†ΠΈΠΈ Ρ†Π΅ΠΏΠΈ.

    Для составлСния ΡƒΡ€Π°Π²Π½Π΅Π½ΠΈΠΉ ΠΏΠΎ Π·Π°ΠΊΠΎΠ½Ρƒ Ома ΠΈ Π΄Π²ΡƒΠΌ Π·Π°ΠΊΠΎΠ½Π°ΠΌ ΠšΠΈΡ€Β­Ρ…Π³ΠΎΡ„Π° слСдуСт ΠΏΡ€Π΅ΠΆΠ΄Π΅ всСго Π²Ρ‹Π±Ρ€Π°Ρ‚ΡŒ (ΠΏΡ€ΠΎΠΈΠ·Π²ΠΎΠ»ΡŒΠ½ΠΎ) ΠΏΠΎΠ»ΠΎΠΆΠΈΡ‚Π΅Π»ΡŒΠ½Ρ‹Π΅ направлСния Ρ‚ΠΎΠΊΠΎΠ² Π²ΠΎ всСх вСтвях рассчитываСмой элСктричСской Ρ†Π΅ΠΏΠΈ.

    ΠŸΡ€ΠΈ записи ΡƒΡ€Π°Π²Π½Π΅Π½ΠΈΠΉ для ΡƒΠ·Π»ΠΎΠ² Ρ†Π΅ΠΏΠΈ ΠΏΠΎ ΠΏΠ΅Ρ€Π²ΠΎΠΌΡƒ Π·Π°ΠΊΠΎΠ½Ρƒ ΠšΠΈΡ€Ρ…Π³ΠΎΡ„Π° Π½Π΅ΠΎΠ±Ρ…ΠΎΠ΄ΠΈΠΌΠΎ ΠΈΠΌΠ΅Ρ‚ΡŒ Π² Π²ΠΈΠ΄Ρƒ, Ρ‡Ρ‚ΠΎ число нСзависимых ΡƒΡ€Π°Π²Π½Π΅Π½ΠΈΠΉ Π½Π° Π΅Π΄ΠΈΠ½ΠΈΡ†Ρƒ мСньшС ΠΎΠ±Ρ‰Π΅Π³ΠΎ числа ΡƒΠ·Π»ΠΎΠ² Ρƒ, Ρ‚. Π΅. Π½ΡƒΠΆΠ½ΠΎ ΡΠΎΡΡ‚Π°Π²ΠΈΡ‚ΡŒ Ρƒ β€” 1 ΡƒΡ€Π°Π²Π½Π΅Π½ΠΈΠΉ. Π”Π΅ΠΉΡΡ‚Π²ΠΈΡ‚Π΅Π»ΡŒΠ½ΠΎ, Ссли ΡΠΎΡΡ‚Π°Π²ΠΈΡ‚ΡŒ уравнСния для всСх Ρƒ ΡƒΠ·Π»ΠΎΠ², Ρ‚ΠΎ Ρ‚ΠΎΠΊ ΠΊΠ°ΠΆΠ΄ΠΎΠΉ Π²Π΅Ρ‚Π²ΠΈ Π²ΠΎΠΉΠ΄Π΅Ρ‚ Π΄Π²Π°ΠΆΠ΄Ρ‹ Π² уравнСния для ΡƒΠ·Π»ΠΎΠ², Ρ‚Π°ΠΊ ΠΊΠ°ΠΏ Π²Π΅Ρ‚Π²ΡŒ соСдиняСт Π΄Π²Π° ΡƒΠ·Π»Π°, ΠΏΡ€ΠΈΡ‡Π΅ΠΌ с ΠΏΡ€ΠΎΡ‚ΠΈΠ²ΠΎΠΏΠΎΠ»ΠΎΠΆΠ½Ρ‹ΠΌΠΈ Π·Π½Π°Β­ΠΊΠ°ΠΌΠΈ (Ρ‚ΠΎΠΊ ΠΊΠ°ΠΆΠ΄ΠΎΠΉ Π²Π΅Ρ‚Π²ΠΈ Π½Π°ΠΏΡ€Π°Π²Π»Π΅Π½ ΠΊ ΠΎΠ΄Π½ΠΎΠΌΡƒ ΠΈΠ· ΡƒΠ·Π»ΠΎΠ², ΠΈ ΡΠ»Π΅Π΄ΠΎΠ²Π°Ρ‚Π΅Π»ΡŒΒ­Π½ΠΎ, Π½Π°ΠΏΡ€Π°Π²Π»Π΅Π½ ΠΎΡ‚ Π΄Ρ€ΡƒΠ³ΠΎΠ³ΠΎ ΡƒΠ·Π»Π°). ΠŸΡ€ΠΈ суммировании всСх Π»Π΅Π²Ρ‹Ρ… частСй

    ΡƒΡ€Π°Π²Π½Π΅Π½ΠΈΠΉ составлСнных ΠΏΠΎ ΠΏΠ΅Ρ€Π²ΠΎΠΌΡƒ Π·Π°ΠΊΠΎΠ½Ρƒ ΠšΠΈΡ€Ρ…Π³ΠΎΡ„Π° получаСтся тоТдСствСнно Π½ΡƒΠ»ΡŒ.

    ΠŸΡ€ΠΈ составлСнии ΡƒΡ€Π°Π²Π½Π΅Π½ΠΈΠΉ Π½Π° основании Π²Ρ‚ΠΎΡ€ΠΎΠ³ΠΎ Π·Π°ΠΊΠΎΠ½Π° ΠšΠΈΡ€Ρ…Π³ΠΎΡ„Π° Π½Π΅ΠΎΠ±Ρ…ΠΎΠ΄ΠΈΠΌΠΎ Ρ‚Π°ΠΊΠΆΠ΅ ΠΏΠΎΠ»ΡƒΡ‡ΠΈΡ‚ΡŒ Π½Π΅Π·Π°Π²ΠΈΡΠΈΠΌΡƒΡŽ систСму. Π’ частности, Π±ΡƒΠ΄Π΅Ρ‚ ΠΏΠΎΠ»ΡƒΡ‡Π΅Π½Π° нСзависимая систСма, Ссли Π²Ρ‹Π±Ρ€Π°Ρ‚ΡŒ ΠΊΠΎΠ½Ρ‚ΡƒΡ€Ρ‹ Ρ‚Π°ΠΊ, Ρ‡Ρ‚ΠΎΠ±Ρ‹ ΠΊΠ°ΠΆΠ΄Ρ‹ΠΉ ΡΠ»Π΅Π΄ΡƒΡŽΡ‰ΠΈΠΉ содСрТал хотя Π±Ρ‹ ΠΎΠ΄Π½Ρƒ Π²Π΅Ρ‚Π²ΡŒ, Π½Π΅ вошСд-

    ΡˆΡƒΡŽ Π² ΠΊΠΎΠ½Ρ‚ΡƒΡ€Ρ‹, для ΠΊΠΎΡ‚ΠΎΡ€Ρ‹Ρ… ΡƒΠΆΠ΅ составлСны уравнСния. Π’Π°ΠΊΠΈΠ΅ ΠΊΠΎΠ½Ρ‚ΡƒΡ€Ρ‹ Π½Π°Π·Ρ‹Π²Π°ΡŽΡ‚ΡΡ нСзависимыми ΠΊΠΎΠ½Ρ‚ΡƒΡ€Π°ΠΌΠΈ, Ρ‚Π°ΠΊ ΠΊΠ°ΠΊ ΠΈΡ… уравнСния Π²Π·Π°ΠΈΠΌΠ½ΠΎ нСзависимы.

    Число нСизвСстных Ρ‚ΠΎΠΊΠΎΠ² Ρ€Π°Π²Π½ΠΎ числу Π²Π΅Ρ‚Π²Π΅ΠΉ Π². Для опрСдСлСния этих Ρ‚ΠΎΠΊΠΎΠ² Π½Π΅ΠΎΠ±Ρ…ΠΎΠ΄ΠΈΠΌΠΎ ΡΠΎΡΡ‚Π°Π²ΠΈΡ‚ΡŒ Π² нСзависимых ΡƒΡ€Π°Π²Π½Π΅Π½ΠΈΠΉ. Π’Π°ΠΊ ΠΊΠ°ΠΊ ΠΏΠΎ ΠΏΠ΅Ρ€Π²ΠΎΠΌΡƒ Π·Π°ΠΊΠΎΠ½Ρƒ ΠšΠΈΡ€Ρ…Π³ΠΎΡ„Π° составляСтся Ρƒ β€” 1 нСзависимых ΡƒΡ€Π°Π²Π½Π΅Π½ΠΈΠΉ, Ρ‚ΠΎ Π½Π° основании Π²Ρ‚ΠΎΡ€ΠΎΠ³ΠΎ Π·Π°ΠΊΠΎΠ½Π° ΠšΠΈΡ€Ρ…Π³ΠΎΡ„Π° Π΄ΠΎΠ»ΠΆΠ½ΠΎ Π±Ρ‹Ρ‚ΡŒ составлСно Π² β€” (Ρƒ β€” 1) ΡƒΡ€Π°Π²Π½Π΅Π½ΠΈΠΉ.

    НапримСр, для Ρ†Π΅ΠΏΠΈ ΠΏΠΎ рис. 1.11 ΠΌΠΎΠΆΠ½ΠΎ Π±Ρ‹Π»ΠΎ Π±Ρ‹ ΡΠΎΡΡ‚Π°Π²ΠΈΡ‚ΡŒ Π΄Π²Π° уравнСния Π½Π° основании ΠΏΠ΅Ρ€Π²ΠΎΠ³ΠΎ Π·Π°ΠΊΠΎΠ½Π° ΠšΠΈΡ€Ρ…Π³ΠΎΡ„Π° для Π΄Π²ΡƒΡ… ΡƒΠ·Π»ΠΎΠ² (Π²Ρ‹Π±Ρ€Π°Π½Π½Ρ‹Π΅ ΠΏΠΎΠ»ΠΎΠΆΠΈΡ‚Π΅Π»ΡŒΠ½Ρ‹Π΅ направлСния Ρ‚ΠΎΠΊΠΎΠ² ΠΏΠΎΠΊΠ°Π·Π°Π½Ρ‹ стрСл­ками). Для ΡƒΠ·Π»Π° 1:

    На основании Π²Ρ‚ΠΎΡ€ΠΎΠ³ΠΎ Π·Π°ΠΊΠΎΠ½Π° ΠšΠΈΡ€Ρ…Π³ΠΎΡ„Π° ΠΈ Π·Π°ΠΊΠΎΠ½Π° Ома ΠΌΠΎΠΆΠ½ΠΎ Π±Ρ‹Π»ΠΎ Π±Ρ‹ ΡΠΎΡΡ‚Π°Π²ΠΈΡ‚ΡŒ Ρ‚Ρ€ΠΈ уравнСния для Ρ‚Ρ€Π΅Ρ… ΠΊΠΎΠ½Ρ‚ΡƒΡ€ΠΎΠ². Для ΠΊΠΎΠ½Ρ‚ΡƒΡ€Π° 1:

    Но схСма Ρ†Π΅ΠΏΠΈ ΠΏΠΎ рис. 1.11 содСрТит Ρ‚ΠΎΠ»ΡŒΠΊΠΎ Ρ‚Ρ€ΠΈ Π²Π΅Ρ‚Π²ΠΈ, Ρ‚. Π΅. для Π΅Π΅ расчСта Π½Π΅ΠΎΠ±Ρ…ΠΎΠ΄ΠΈΠΌΡ‹ Ρ‚Ρ€ΠΈ нСзависимых уравнСния.

    Для этой схСмы число ΡƒΠ·Π»ΠΎΠ² Ρƒ = 2, Ρ‚. Π΅. ΠΏΠΎ ΠΏΠ΅Ρ€Π²ΠΎΠΌΡƒ Π·Π°ΠΊΠΎΠ½Ρƒ ΠšΠΈΡ€Ρ…Β­Π³ΠΎΡ„Π° Π΄ΠΎΠ»ΠΆΠ½ΠΎ Π±Ρ‹Ρ‚ΡŒ составлСно Ρ‚ΠΎΠ»ΡŒΠΊΠΎ ΠΎΠ΄Π½ΠΎ нСзависимоС ΡƒΡ€Π°Π²Π½Π΅Π½ΠΈΠ΅: (1 8) ΠΈΠ»ΠΈ (1.9). Π”Π΅ΠΉΡΡ‚Π²ΠΈΡ‚Π΅Π»ΡŒΠ½ΠΎ, ΠΎΠ΄Π½ΠΎ ΠΈΠ· этих ΡƒΡ€Π°Π²Π½Π΅Π½ΠΈΠΉ получаСтся ΠΈΠ· Π΄Ρ€ΡƒΠ³ΠΎΠ³ΠΎ ΡƒΠΌΠ½ΠΎΠΆΠ΅Π½ΠΈΠ΅ΠΌ Π½Π° β€”I.

    Из Ρ‚Ρ€Π΅Ρ… ΠΊΠΎΠ½Ρ‚ΡƒΡ€ΠΎΠ² Π½Π° рис. 1.11 нСзависимыми ΠΌΠΎΠΆΠ½ΠΎ, Π½Π°ΠΏΡ€ΠΈΠΌΠ΅Ρ€, ΡΡ‡ΠΈΡ‚Π°Ρ‚ΡŒ 1-ΠΉ ΠΈ 2-ΠΉ (Π²ΠΎ 2-ΠΉ Π²Ρ…ΠΎΠ΄ΠΈΡ‚ Π²Π΅Ρ‚Π²ΡŒ 2, ΠΊΠΎΡ‚ΠΎΡ€ΡƒΡŽ Π½Π΅ содСрТит, 1-ΠΉ ΠΊΠΎΠ½Ρ‚ΡƒΡ€). Π”Π΅ΠΉΡΡ‚Π²ΠΈΡ‚Π΅Π»ΡŒΠ½ΠΎ, ΡƒΡ€Π°Π²Π½Π΅Π½ΠΈΠ΅ (1.16) ΠΌΠΎΠΆΠ½ΠΎ ΠΏΠΎΠ»ΡƒΡ‡ΠΈΡ‚ΡŒ, составив соотвСтствСнно разности ΠΏΡ€Π°Π²Ρ‹Ρ… ΠΈ Π»Π΅Π²Ρ‹Ρ… частСй ΡƒΡ€Π°Π²Π½Π΅Π½ΠΈΠΉ (1.10 ΠΈ (1.11). Π’ качСствС нСзависимых для схСмы Ρ†Π΅ΠΏΠΈ ΠΏΠΎ рис. 1.40 ΠΌΠΎΠΆΠ½ΠΎ Π²Π·ΡΡ‚ΡŒ ΠΈ Π»ΡŽΠ±ΡƒΡŽ Π΄Ρ€ΡƒΠ³ΡƒΡŽ ΠΏΠ°Ρ€Ρƒ ΠΊΠΎΠ½Ρ‚ΡƒΡ€ΠΎΠ².

    ПослС совмСстного Ρ€Π΅ΡˆΠ΅Π½ΠΈΡ систСмы нСзависимых ΡƒΡ€Π°Π²Π½Π΅Π½ΠΈΠΉ ΠΎΠΏΡ€Π΅Β­Π΄Π΅Π»ΡΡŽΡ‚ΡΡ Ρ‚ΠΎΠΊΠΈ Π² вСтвях Ρ†Π΅ΠΏΠΈ. Если для ΠΊΠ°ΠΊΠΎΠ³ΠΎ-Π»ΠΈΠ±ΠΎ Ρ‚ΠΎΠΊΠ° Π±ΡƒΠ΄Π΅Ρ‚ ΠΏΠΎΠ»ΡƒΒ­Ρ‡Π΅Π½ΠΎ ΠΎΡ‚Ρ€ΠΈΡ†Π°Ρ‚Π΅Π»ΡŒΠ½ΠΎΠ΅ Π·Π½Π°Ρ‡Π΅Π½ΠΈΠ΅, Ρ‚ΠΎ ΠΈΠ· этого слСдуСт, Ρ‡Ρ‚ΠΎ Π΅Π³ΠΎ Π΄Π΅ΠΉΡΡ‚Π²ΠΈΒ­Ρ‚Π΅Π»ΡŒΠ½ΠΎΠ΅ Π½Π°ΠΏΡ€Π°Π²Π»Π΅Π½ΠΈΠ΅ ΠΏΡ€ΠΎΡ‚ΠΈΠ²ΠΎΠΏΠΎΠ»ΠΎΠΆΠ½ΠΎ Π²Ρ‹Π±Ρ€Π°Π½Π½ΠΎΠΌΡƒ ΠΏΠΎΠ»ΠΎΠΆΠΈΡ‚Π΅Π»ΡŒΠ½ΠΎΠΌΡƒ Π½Π°ΠΏΡ€Π°Π²Π»Π΅Π½ΠΈΡŽ.

    БовмСстноС Ρ€Π΅ΡˆΠ΅Π½ΠΈΠ΅ систСмы ΡƒΡ€Π°Π²Π½Π΅Π½ΠΈΠΉ, составлСнных Π½Π° основа­нии Π΄Π²ΡƒΡ… Π·Π°ΠΊΠΎΠ½ΠΎΠ² ΠšΠΈΡ€Ρ…Π³ΠΎΡ„Π° для слоТной Ρ†Π΅ΠΏΠΈ, часто Ρ‚Ρ€Π΅Π±ΡƒΠ΅Ρ‚ Π·Π½Π°Β­Ρ‡ΠΈΡ‚Π΅Π»ΡŒΠ½ΠΎΠΉ Π·Π°Ρ‚Ρ€Π°Ρ‚Ρ‹ Π²Ρ€Π΅ΠΌΠ΅Π½ΠΈ. ΠŸΠΎΡΡ‚ΠΎΠΌΡƒ ΠΆΠ΅Π»Π°Ρ‚Π΅Π»ΡŒΠ½ΠΎ, ΠΊΠΎΠ³Π΄Π° это Π²ΠΎΠ·ΠΌΠΎΠΆΠ½ΠΎ, вСсти расчСты Π±ΠΎΠ»Π΅Π΅ простыми ΠΌΠ΅Ρ‚ΠΎΠ΄Π°ΠΌΠΈ, ΠΊΠΎΡ‚ΠΎΡ€Ρ‹Π΅ Ρ€Π°ΡΡΠΌΠ°Ρ‚Ρ€ΠΈΠ²Π°ΡŽΡ‚ΡΡ Π² ΡΠ»Π΅Π΄ΡƒΡŽΡ‰ΠΈΡ… ΠΏΠ°Ρ€Π°Π³Ρ€Π°Ρ„Π°Ρ…. ВсС эти ΠΌΠ΅Ρ‚ΠΎΠ΄Ρ‹ ΠΎΡΠ½ΠΎΠ²Ρ‹Π²Π°ΡŽΡ‚ΡΡ Π½Π° Π·Π°ΠΊΠΎΠ½Π°Ρ… Ома ΠΈ ΠšΠΈΡ€Ρ…Π³ΠΎΡ„Π°.

    ВрядС случаСв расчСт слоТной элСктричСской Ρ†Π΅ΠΏΠΈ сущСствСнно упрощаСтся, Ссли Π² этой Ρ†Π΅ΠΏΠΈ Π·Π°ΠΌΠ΅Π½ΠΈΡ‚ΡŒ Π³Ρ€ΡƒΠΏΠΏΡƒ рСзистивиых элСмСнтов Π΄Ρ€ΡƒΠ³ΠΎΠΉ эквивалСнтной Π³Ρ€ΡƒΠΏΠΏΠΎΠΉ, Π² ΠΊΠΎΡ‚ΠΎΡ€ΠΎΠΉ рСзистивныС элСмСнты соСдинСны ΠΈΠ½Π°Ρ‡Π΅, Ρ‡Π΅ΠΌ Π² Π·Π°Π΄Π°Π½Π½ΠΎΠΉ Ρ†Π΅ΠΏΠΈ. Взаимная ΡΠΊΠ²ΠΈΠ²Π°Π»Π΅Π½Ρ‚Π½ΠΎΡΡ‚ΡŒ Π΄Π²ΡƒΡ… Π³Ρ€ΡƒΠΏΠΏ рСзистивных элСмСнтов опрСдСляСтся Ρ‚Π΅ΠΌ, Ρ‡Ρ‚ΠΎ послС Π·Π°Β­ΠΌΠ΅Π½Ρ‹ Ρ€Π΅ΠΆΠΈΠΌ Ρ€Π°Π±ΠΎΡ‚Ρ‹ ΠΎΡΡ‚Π°Π»ΡŒΠ½ΠΎΠΉ части элСктричСской Ρ†Π΅ΠΏΠΈ Π½Π΅ измСнится.

    Π’ ΠΎΠ±Ρ‰Π΅ΠΌ случаС Ρ†Π΅ΠΏΡŒ ΠΈΠ· рСзис­тивных элСмСнтов ΠΏΠΎ схСмС n-лучСвая Π·Π²Π΅Π·Π΄Π° ΠΌΠΎΠΆΠ΅Ρ‚ Π±Ρ‹Ρ‚ΡŒ Π·Π°ΠΌΠ΅Ρ‰Π΅Π½Π° эквивалСнтной Ρ†Π΅ΠΏΡŒΡŽ ΠΏΠΎ схСмС n -сторонний ΠΌΠ½ΠΎΠ³ΠΎΡƒΠ³ΠΎΠ»ΡŒΠ½ΠΈΠΊ. Π’ ΠΎΠ±Ρ€Π°Ρ‚Β­Π½ΠΎΠΌ Π½Π°ΠΏΡ€Π°Π²Π»Π΅Π½ΠΈΠΈ ΠΏΡ€Π΅ΠΎΠ±Ρ€Π°Π·ΠΎΠ²Π°Π½ΠΈΠ΅ Π²ΠΎΠ·ΠΌΠΎΠΆΠ½ΠΎ Π² ΠΎΠ³Ρ€Π°Π½ΠΈΡ‡Π΅Π½Π½ΠΎΠΌ числС случаСв. Π’ частности, прСобразования Π² ΠΎΠ±ΠΎΠΈΡ… направлСниях Π²ΠΎΠ·ΠΌΠΎΠΆΒ­Π½Ρ‹ для Ρ†Π΅ΠΏΠ΅ΠΉ рСзистивных элСмСнтов ΠΏΠΎ схСмам Ρ‚Ρ€Π΅ΡƒΠ³ΠΎΠ»ΡŒΠ½ΠΈΠΊ ΠΈ трСхлучСвая Π·Π²Π΅Π·Π΄Π°. Π’Π°ΠΊΠΎΠ΅ ΠΏΡ€Π΅ΠΎΠ±Ρ€Π°Π·ΠΎΠ²Π°Π½ΠΈΠ΅ часто примСняСтся для слоТ­ных Ρ†Π΅ΠΏΠ΅ΠΉ постоянного Ρ‚ΠΎΠΊΠ°, Π½ΠΎ особСнно Π²Π°ΠΆΠ½ΠΎ ΠΏΡ€ΠΈ расчСтах слоТ­ных Ρ†Π΅ΠΏΠ΅ΠΉ Ρ‚Ρ€Π΅Ρ…Ρ„Π°Π·Π½ΠΎΠ³ΠΎ Ρ‚ΠΎΠΊΠ° (Π³Π». 3).

    Условия эквивалСнтности Ρ†Π΅ΠΏΠ΅ΠΉ ΠΏΠΎ схСмам Ρ‚Ρ€Π΅ΡƒΠ³ΠΎΠ»ΡŒΠ½ΠΈΠΊ ΠΈ Π·Π²Π΅Π·Π΄Π° (рис. 1.12) ΠΏΡ€ΠΎΡ‰Π΅ всСго ΠΎΠΏΡ€Π΅Π΄Π΅Π»ΡΡŽΡ‚ΡΡ ΠΏΡ€ΠΈΡ€Π°Π²Π½ΠΈΠ²Π°Π½ΠΈΠ΅ΠΌ Π·Π½Π°Ρ‡Π΅Π½ΠΈΠΉ сопротивлСний ΠΈ проводимостСй ΠΌΠ΅ΠΆΠ΄Ρƒ ΠΎΠ΄Π½ΠΎΠΈΠΌΠ΅Π½Π½Ρ‹ΠΌΠΈ ΡƒΠ·Π»Π°ΠΌΠΈ этих.

    Π§Ρ‚ΠΎΠ±Ρ‹ ΠΎΠΏΡ€Π΅Π΄Π΅Π»ΠΈΡ‚ΡŒ сопротивлСниС Π²Π΅Ρ‚Π²ΠΈ ΠΏΠΎ схСмС Π·Π²Π΅Π·Π΄Π° rΠ° Π½Π°ΠΉΠ΄Π΅ΠΌ:

    Π”Π²Π° Π΄Ρ€ΡƒΠ³ΠΈΠ΅ выраТСния для опрСдСлСния сопротивлСний Π²Π΅Ρ‚Π²Π΅ΠΉ Ρ†Π΅ΠΏΠΈ ΠΏΠΎ схСмС Π·Π²Π΅Π·Π΄Π° ΠΏΠΎΠ»ΡƒΡ‡ΠΈΠΌ ΠΏΡƒΡ‚Π΅ΠΌ простой цикличСской пСрСстановки индСксов:

    ΠžΠΏΡ€Π΅Π΄Π΅Π»ΠΈΠΌ сопротив­лСниС Π²Π΅Ρ‚Π²ΠΈ эквивалСнтной Ρ†Π΅Π½ΠΈ ΠΏΠΎ схСмС Ρ‚Ρ€Π΅ΡƒΠ³ΠΎΠ»ΡŒΠ½ΠΈΠΊ:

    ΠŸΠΎΡΡ€Π΅Π΄ΡΡ‚Π²ΠΎΠΌ цикличСской пСрСстановки индСксов Π² (1.24) Π½Π°ΠΉΠ΄Π΅ΠΌ вы­раТСния для сопротивлСний Π΄Π²ΡƒΡ… Π΄Ρ€ΡƒΠ³ΠΈΡ… Π²Π΅Ρ‚Π²Π΅ΠΉ

    studfiles.net

    ЭлСктричСскиС Ρ†Π΅ΠΏΠΈ постоянного Ρ‚ΠΎΠΊΠ° ΠΈ ΠΌΠ΅Ρ‚ΠΎΠ΄Ρ‹ ΠΈΡ… расчСта

    1.1. ЭлСктричСская Ρ†Π΅ΠΏΡŒ ΠΈ Π΅Π΅ элСмСнты

    Π’ элСктротСхникС рассматриваСтся устройство ΠΈ ΠΏΡ€ΠΈΠ½Ρ†ΠΈΠΏ дСйствия основных элСктротСхничСских устройств, ΠΈΡΠΏΠΎΠ»ΡŒΠ·ΡƒΠ΅ΠΌΡ‹Ρ… Π² Π±Ρ‹Ρ‚Ρƒ ΠΈ ΠΏΡ€ΠΎΠΌΡ‹ΡˆΠ»Π΅Π½Π½ΠΎΡΡ‚ΠΈ. Π§Ρ‚ΠΎΠ±Ρ‹ элСктротСхничСскоС устройство Ρ€Π°Π±ΠΎΡ‚Π°Π»ΠΎ, Π΄ΠΎΠ»ΠΆΠ½Π° Π±Ρ‹Ρ‚ΡŒ создана элСктричСская Ρ†Π΅ΠΏΡŒ, Π·Π°Π΄Π°Ρ‡Π° ΠΊΠΎΡ‚ΠΎΡ€ΠΎΠΉ ΠΏΠ΅Ρ€Π΅Π΄Π°Ρ‚ΡŒ ΡΠ»Π΅ΠΊΡ‚Ρ€ΠΈΡ‡Π΅ΡΠΊΡƒΡŽ ΡΠ½Π΅Ρ€Π³ΠΈΡŽ этому устройству ΠΈ ΠΎΠ±Π΅ΡΠΏΠ΅Ρ‡ΠΈΡ‚ΡŒ Π΅ΠΌΡƒ Ρ‚Ρ€Π΅Π±ΡƒΠ΅ΠΌΡ‹ΠΉ Ρ€Π΅ΠΆΠΈΠΌ Ρ€Π°Π±ΠΎΡ‚Ρ‹.

    ЭлСктричСской Ρ†Π΅ΠΏΡŒΡŽ называСтся ΡΠΎΠ²ΠΎΠΊΡƒΠΏΠ½ΠΎΡΡ‚ΡŒ устройств ΠΈ ΠΎΠ±ΡŠΠ΅ΠΊΡ‚ΠΎΠ², ΠΎΠ±Ρ€Π°Π·ΡƒΡŽΡ‰ΠΈΡ… ΠΏΡƒΡ‚ΡŒ для элСктричСского Ρ‚ΠΎΠΊΠ°, элСктромагнитныС процСссы Π² ΠΊΠΎΡ‚ΠΎΡ€Ρ‹Ρ… ΠΌΠΎΠ³ΡƒΡ‚ Π±Ρ‹Ρ‚ΡŒ описаны с ΠΏΠΎΠΌΠΎΡ‰ΡŒΡŽ понятий ΠΎΠ± элСктричСском Ρ‚ΠΎΠΊΠ΅, Π­Π”Π‘ (элСктродвиТущая сила) ΠΈ элСктричСском напряТСнии.

    Для Π°Π½Π°Π»ΠΈΠ·Π° ΠΈ расчСта элСктричСская Ρ†Π΅ΠΏΡŒ графичСски прСдставляСтся Π² Π²ΠΈΠ΄Π΅ элСктричСской схСмы, содСрТащСй условныС обозначСния Π΅Π΅ элСмСнтов ΠΈ способы ΠΈΡ… соСдинСния. ЭлСктричСская схСма ΠΏΡ€ΠΎΡΡ‚Π΅ΠΉΡˆΠ΅ΠΉ элСктричСской Ρ†Π΅ΠΏΠΈ, ΠΎΠ±Π΅ΡΠΏΠ΅Ρ‡ΠΈΠ²Π°ΡŽΡ‰Π΅ΠΉ Ρ€Π°Π±ΠΎΡ‚Ρƒ ΠΎΡΠ²Π΅Ρ‚ΠΈΡ‚Π΅Π»ΡŒΠ½ΠΎΠΉ Π°ΠΏΠΏΠ°Ρ€Π°Ρ‚ΡƒΡ€Ρ‹, прСдставлСна Π½Π° рис. 1.1.

    ВсС устройства ΠΈ ΠΎΠ±ΡŠΠ΅ΠΊΡ‚Ρ‹, входящиС Π² состав элСктричСской Ρ†Π΅ΠΏΠΈ, ΠΌΠΎΠ³ΡƒΡ‚ Π±Ρ‹Ρ‚ΡŒ Ρ€Π°Π·Π΄Π΅Π»Π΅Π½Ρ‹ Π½Π° Ρ‚Ρ€ΠΈ Π³Ρ€ΡƒΠΏΠΏΡ‹:

    1) Π˜ΡΡ‚ΠΎΡ‡Π½ΠΈΠΊΠΈ элСктричСской энСргии (питания).

    ΠžΠ±Ρ‰ΠΈΠΌ свойством всСх источников питания являСтся ΠΏΡ€Π΅ΠΎΠ±Ρ€Π°Π·ΠΎΠ²Π°Π½ΠΈΠ΅ ΠΊΠ°ΠΊΠΎΠ³ΠΎ-Π»ΠΈΠ±ΠΎ Π²ΠΈΠ΄Π° энСргии Π² ΡΠ»Π΅ΠΊΡ‚Ρ€ΠΈΡ‡Π΅ΡΠΊΡƒΡŽ. Π˜ΡΡ‚ΠΎΡ‡Π½ΠΈΠΊΠΈ, Π² ΠΊΠΎΡ‚ΠΎΡ€Ρ‹Ρ… происходит ΠΏΡ€Π΅ΠΎΠ±Ρ€Π°Π·ΠΎΠ²Π°Π½ΠΈΠ΅ нСэлСктричСской энСргии Π² ΡΠ»Π΅ΠΊΡ‚Ρ€ΠΈΡ‡Π΅ΡΠΊΡƒΡŽ, Π½Π°Π·Ρ‹Π²Π°ΡŽΡ‚ΡΡ ΠΏΠ΅Ρ€Π²ΠΈΡ‡Π½Ρ‹ΠΌΠΈ источниками. Π’Ρ‚ΠΎΡ€ΠΈΡ‡Π½Ρ‹Π΅ источники – это Ρ‚Π°ΠΊΠΈΠ΅ источники, Ρƒ ΠΊΠΎΡ‚ΠΎΡ€Ρ‹Ρ… ΠΈ Π½Π° Π²Ρ…ΠΎΠ΄Π΅, ΠΈ Π½Π° Π²Ρ‹Ρ…ΠΎΠ΄Π΅ – элСктричСская энСргия (Π½Π°ΠΏΡ€ΠΈΠΌΠ΅Ρ€, Π²Ρ‹ΠΏΡ€ΡΠΌΠΈΡ‚Π΅Π»ΡŒΠ½Ρ‹Π΅ устройства).

    2) ΠŸΠΎΡ‚Ρ€Π΅Π±ΠΈΡ‚Π΅Π»ΠΈ элСктричСской энСргии.

    ΠžΠ±Ρ‰ΠΈΠΌ свойством всСх ΠΏΠΎΡ‚Ρ€Π΅Π±ΠΈΡ‚Π΅Π»Π΅ΠΉ являСтся ΠΏΡ€Π΅ΠΎΠ±Ρ€Π°Π·ΠΎΠ²Π°Π½ΠΈΠ΅ элСктроэнСргии Π² Π΄Ρ€ΡƒΠ³ΠΈΠ΅ Π²ΠΈΠ΄Ρ‹ энСргии (Π½Π°ΠΏΡ€ΠΈΠΌΠ΅Ρ€, Π½Π°Π³Ρ€Π΅Π²Π°Ρ‚Π΅Π»ΡŒΠ½Ρ‹ΠΉ ΠΏΡ€ΠΈΠ±ΠΎΡ€). Иногда ΠΏΠΎΡ‚Ρ€Π΅Π±ΠΈΡ‚Π΅Π»ΠΈ Π½Π°Π·Ρ‹Π²Π°ΡŽΡ‚ Π½Π°Π³Ρ€ΡƒΠ·ΠΊΠΎΠΉ.

    3) Π’ΡΠΏΠΎΠΌΠΎΠ³Π°Ρ‚Π΅Π»ΡŒΠ½Ρ‹Π΅ элСмСнты Ρ†Π΅ΠΏΠΈ: ΡΠΎΠ΅Π΄ΠΈΠ½ΠΈΡ‚Π΅Π»ΡŒΠ½Ρ‹Π΅ ΠΏΡ€ΠΎΠ²ΠΎΠ΄Π°, коммутационная Π°ΠΏΠΏΠ°Ρ€Π°Ρ‚ΡƒΡ€Π°, Π°ΠΏΠΏΠ°Ρ€Π°Ρ‚ΡƒΡ€Π° Π·Π°Ρ‰ΠΈΡ‚Ρ‹, ΠΈΠ·ΠΌΠ΅Ρ€ΠΈΡ‚Π΅Π»ΡŒΠ½Ρ‹Π΅ ΠΏΡ€ΠΈΠ±ΠΎΡ€Ρ‹ ΠΈ Ρ‚.Π΄., Π±Π΅Π· ΠΊΠΎΡ‚ΠΎΡ€Ρ‹Ρ… Ρ€Π΅Π°Π»ΡŒΠ½Π°Ρ Ρ†Π΅ΠΏΡŒ Π½Π΅ Ρ€Π°Π±ΠΎΡ‚Π°Π΅Ρ‚.

    ВсС элСмСнты Ρ†Π΅ΠΏΠΈ ΠΎΡ…Π²Π°Ρ‡Π΅Π½Ρ‹ ΠΎΠ΄Π½ΠΈΠΌ элСктромагнитным процСссом.

    Π’ элСктричСской схСмС Π½Π° рис. 1.1 элСктричСская энСргия ΠΎΡ‚ источника Π­Π”Π‘ $E$, ΠΎΠ±Π»Π°Π΄Π°ΡŽΡ‰Π΅Π³ΠΎ Π²Π½ΡƒΡ‚Ρ€Π΅Π½Π½ΠΈΠΌ сопротивлСниСм $r_0$, с ΠΏΠΎΠΌΠΎΡ‰ΡŒΡŽ Π²ΡΠΏΠΎΠΌΠΎΠ³Π°Ρ‚Π΅Π»ΡŒΠ½Ρ‹Ρ… элСмСнтов Ρ†Π΅ΠΏΠΈ ΠΏΠ΅Ρ€Π΅Π΄Π°ΡŽΡ‚ΡΡ Ρ‡Π΅Ρ€Π΅Π· Ρ€Π΅Π³ΡƒΠ»ΠΈΡ€ΠΎΠ²ΠΎΡ‡Π½Ρ‹ΠΉ рСостат $R$ ΠΊ потрСбитСлям (Π½Π°Π³Ρ€ΡƒΠ·ΠΊΠ΅): элСктричСским Π»Π°ΠΌΠΏΠΎΡ‡ΠΊΠ°ΠΌ $EL_1$ ΠΈ $EL_2$.

    1.2. ΠžΡΠ½ΠΎΠ²Π½Ρ‹Π΅ понятия ΠΈ опрСдСлСния для элСктричСской Ρ†Π΅ΠΏΠΈ

    Для расчСта ΠΈ Π°Π½Π°Π»ΠΈΠ·Π° Ρ€Π΅Π°Π»ΡŒΠ½Π°Ρ элСктричСская Ρ†Π΅ΠΏΡŒ прСдставляСтся графичСски Π² Π²ΠΈΠ΄Π΅ расчСтной элСктричСской схСмы (схСмы замСщСния). Π’ этой схСмС Ρ€Π΅Π°Π»ΡŒΠ½Ρ‹Π΅ элСмСнты Ρ†Π΅ΠΏΠΈ ΠΈΠ·ΠΎΠ±Ρ€Π°ΠΆΠ°ΡŽΡ‚ΡΡ условными обозначСниями, ΠΏΡ€ΠΈΡ‡Π΅ΠΌ Π²ΡΠΏΠΎΠΌΠΎΠ³Π°Ρ‚Π΅Π»ΡŒΠ½Ρ‹Π΅ элСмСнты Ρ†Π΅ΠΏΠΈ ΠΎΠ±Ρ‹Ρ‡Π½ΠΎ Π½Π΅ ΠΈΠ·ΠΎΠ±Ρ€Π°ΠΆΠ°ΡŽΡ‚ΡΡ, Π° Ссли сопротивлСниС ΡΠΎΠ΅Π΄ΠΈΠ½ΠΈΡ‚Π΅Π»ΡŒΠ½Ρ‹Ρ… ΠΏΡ€ΠΎΠ²ΠΎΠ΄ΠΎΠ² Π½Π°ΠΌΠ½ΠΎΠ³ΠΎ мСньшС сопротивлСния Π΄Ρ€ΡƒΠ³ΠΈΡ… элСмСнтов Ρ†Π΅ΠΏΠΈ, Π΅Π³ΠΎ Π½Π΅ ΡƒΡ‡ΠΈΡ‚Ρ‹Π²Π°ΡŽΡ‚. Π˜ΡΡ‚ΠΎΡ‡Π½ΠΈΠΊ питания показываСтся ΠΊΠ°ΠΊ источник Π­Π”Π‘ $E$ с Π²Π½ΡƒΡ‚Ρ€Π΅Π½Π½ΠΈΠΌ сопротивлСниСм $r_0$, Ρ€Π΅Π°Π»ΡŒΠ½Ρ‹Π΅ ΠΏΠΎΡ‚Ρ€Π΅Π±ΠΈΡ‚Π΅Π»ΠΈ элСктричСской энСргии постоянного Ρ‚ΠΎΠΊΠ° Π·Π°ΠΌΠ΅Π½ΡΡŽΡ‚ΡΡ ΠΈΡ… элСктричСскими ΠΏΠ°Ρ€Π°ΠΌΠ΅Ρ‚Ρ€Π°ΠΌΠΈ: Π°ΠΊΡ‚ΠΈΠ²Π½Ρ‹ΠΌΠΈ сопротивлСниями $R_1$, $R_2,

    R_n$. Π‘ ΠΏΠΎΠΌΠΎΡ‰ΡŒΡŽ сопротивлСния $R$ ΡƒΡ‡ΠΈΡ‚Ρ‹Π²Π°ΡŽΡ‚ ΡΠΏΠΎΡΠΎΠ±Π½ΠΎΡΡ‚ΡŒ Ρ€Π΅Π°Π»ΡŒΠ½ΠΎΠ³ΠΎ элСмСнта Ρ†Π΅ΠΏΠΈ Π½Π΅ΠΎΠ±Ρ€Π°Ρ‚ΠΈΠΌΠΎ ΠΏΡ€Π΅ΠΎΠ±Ρ€Π°Π·ΠΎΠ²Ρ‹Π²Π°Ρ‚ΡŒ ΡΠ»Π΅ΠΊΡ‚Ρ€ΠΎΡΠ½Π΅Ρ€Π³ΠΈΡŽ Π² Π΄Ρ€ΡƒΠ³ΠΈΠ΅ Π²ΠΈΠ΄Ρ‹, Π½Π°ΠΏΡ€ΠΈΠΌΠ΅Ρ€, Ρ‚Π΅ΠΏΠ»ΠΎΠ²ΡƒΡŽ ΠΈΠ»ΠΈ Π»ΡƒΡ‡ΠΈΡΡ‚ΡƒΡŽ.

    ΠŸΡ€ΠΈ этих условиях схСма Π½Π° рис. 1.1 ΠΌΠΎΠΆΠ΅Ρ‚ Π±Ρ‹Ρ‚ΡŒ прСдставлСна Π² Π²ΠΈΠ΄Π΅ расчСтной элСктричСской схСмы (рис. 1.2), Π² ΠΊΠΎΡ‚ΠΎΡ€ΠΎΠΉ Π΅ΡΡ‚ΡŒ источник питания с Π­Π”Π‘ $E$ ΠΈ Π²Π½ΡƒΡ‚Ρ€Π΅Π½Π½ΠΈΠΌ сопротивлСниСм $r_0$, Π° ΠΏΠΎΡ‚Ρ€Π΅Π±ΠΈΡ‚Π΅Π»ΠΈ элСктричСской энСргии: Ρ€Π΅Π³ΡƒΠ»ΠΈΡ€ΠΎΠ²ΠΎΡ‡Π½Ρ‹ΠΉ рСостат $R$, элСктричСскиС Π»Π°ΠΌΠΏΠΎΡ‡ΠΊΠΈ $EL_1$ ΠΈ $EL_2$ Π·Π°ΠΌΠ΅Π½Π΅Π½Ρ‹ Π°ΠΊΡ‚ΠΈΠ²Π½Ρ‹ΠΌΠΈ сопротивлСниями $R,

    Π˜ΡΡ‚ΠΎΡ‡Π½ΠΈΠΊ Π­Π”Π‘ Π½Π° элСктричСской схСмС (рис. 1.2) ΠΌΠΎΠΆΠ΅Ρ‚ Π±Ρ‹Ρ‚ΡŒ Π·Π°ΠΌΠ΅Π½Π΅Π½ источником напряТСния $U$, ΠΏΡ€ΠΈΡ‡Π΅ΠΌ условноС ΠΏΠΎΠ»ΠΎΠΆΠΈΡ‚Π΅Π»ΡŒΠ½ΠΎΠ΅ Π½Π°ΠΏΡ€Π°Π²Π»Π΅Π½ΠΈΠ΅ напряТСния $U$ источника задаСтся ΠΏΡ€ΠΎΡ‚ΠΈΠ²ΠΎΠΏΠΎΠ»ΠΎΠΆΠ½Ρ‹ΠΌ Π½Π°ΠΏΡ€Π°Π²Π»Π΅Π½ΠΈΡŽ Π­Π”Π‘.

    ΠŸΡ€ΠΈ расчСтС Π² схСмС элСктричСской Ρ†Π΅ΠΏΠΈ Π²Ρ‹Π΄Π΅Π»ΡΡŽΡ‚ нСсколько основных элСмСнтов.

    Π’Π΅Ρ‚Π²ΡŒ элСктричСской Ρ†Π΅ΠΏΠΈ (схСмы) – участок Ρ†Π΅ΠΏΠΈ с ΠΎΠ΄Π½ΠΈΠΌ ΠΈ Ρ‚Π΅ΠΌ ΠΆΠ΅ Ρ‚ΠΎΠΊΠΎΠΌ. Π’Π΅Ρ‚Π²ΡŒ ΠΌΠΎΠΆΠ΅Ρ‚ ΡΠΎΡΡ‚ΠΎΡΡ‚ΡŒ ΠΈΠ· ΠΎΠ΄Π½ΠΎΠ³ΠΎ ΠΈΠ»ΠΈ Π½Π΅ΡΠΊΠΎΠ»ΡŒΠΊΠΈΡ… ΠΏΠΎΡΠ»Π΅Π΄ΠΎΠ²Π°Ρ‚Π΅Π»ΡŒΠ½ΠΎ соСдинСнных элСмСнтов. Π‘Ρ…Π΅ΠΌΠ° Π½Π° рис. 1.2 ΠΈΠΌΠ΅Π΅Ρ‚ Ρ‚Ρ€ΠΈ Π²Π΅Ρ‚Π²ΠΈ: Π²Π΅Ρ‚Π²ΡŒ bma, Π² ΠΊΠΎΡ‚ΠΎΡ€ΡƒΡŽ Π²ΠΊΠ»ΡŽΡ‡Π΅Π½Ρ‹ элСмСнты $r_0,

    R$ ΠΈ Π² ΠΊΠΎΡ‚ΠΎΡ€ΠΎΠΉ Π²ΠΎΠ·Π½ΠΈΠΊΠ°Π΅Ρ‚ Ρ‚ΠΎΠΊ $I$; Π²Π΅Ρ‚Π²ΡŒ ab с элСмСнтом $R_1$ ΠΈ Ρ‚ΠΎΠΊΠΎΠΌ $I_1$; Π²Π΅Ρ‚Π²ΡŒ anb с элСмСнтом $R_2$ ΠΈ Ρ‚ΠΎΠΊΠΎΠΌ $I_2$.

    Π£Π·Π΅Π» элСктричСской Ρ†Π΅ΠΏΠΈ (схСмы) – мСсто соСдинСния Ρ‚Ρ€Π΅Ρ… ΠΈ Π±ΠΎΠ»Π΅Π΅ Π²Π΅Ρ‚Π²Π΅ΠΉ. Π’ схСмС Π½Π° рис. 1.2 – Π΄Π²Π° ΡƒΠ·Π»Π° a ΠΈ b. Π’Π΅Ρ‚Π²ΠΈ, присоСдинСнныС ΠΊ ΠΎΠ΄Π½ΠΎΠΉ ΠΏΠ°Ρ€Π΅ ΡƒΠ·Π»ΠΎΠ², Π½Π°Π·Ρ‹Π²Π°ΡŽΡ‚ ΠΏΠ°Ρ€Π°Π»Π»Π΅Π»ΡŒΠ½Ρ‹ΠΌΠΈ. БопротивлСния $R_1$ ΠΈ $R_2$ (рис. 1.2) находятся Π² ΠΏΠ°Ρ€Π°Π»Π»Π΅Π»ΡŒΠ½Ρ‹Ρ… вСтвях.

    ΠšΠΎΠ½Ρ‚ΡƒΡ€ – любой Π·Π°ΠΌΠΊΠ½ΡƒΡ‚Ρ‹ΠΉ ΠΏΡƒΡ‚ΡŒ, проходящий ΠΏΠΎ нСскольким вСтвям. Π’ схСмС Π½Π° рис. 1.2 ΠΌΠΎΠΆΠ½ΠΎ Π²Ρ‹Π΄Π΅Π»ΠΈΡ‚ΡŒ Ρ‚Ρ€ΠΈ ΠΊΠΎΠ½Ρ‚ΡƒΡ€Π°: I – bmab; II – anba; III – manbm, Π½Π° схСмС стрСлкой ΠΏΠΎΠΊΠ°Π·Ρ‹Π²Π°ΡŽΡ‚ Π½Π°ΠΏΡ€Π°Π²Π»Π΅Π½ΠΈΠ΅ ΠΎΠ±Ρ…ΠΎΠ΄Π° ΠΊΠΎΠ½Ρ‚ΡƒΡ€Π°.

    УсловныС ΠΏΠΎΠ»ΠΎΠΆΠΈΡ‚Π΅Π»ΡŒΠ½Ρ‹Π΅ направлСния Π­Π”Π‘ источников питания, Ρ‚ΠΎΠΊΠΎΠ² Π²ΠΎ всСх вСтвях, напряТСний ΠΌΠ΅ΠΆΠ΄Ρƒ ΡƒΠ·Π»Π°ΠΌΠΈ ΠΈ Π½Π° Π·Π°ΠΆΠΈΠΌΠ°Ρ… элСмСнтов Ρ†Π΅ΠΏΠΈ Π½Π΅ΠΎΠ±Ρ…ΠΎΠ΄ΠΈΠΌΠΎ Π·Π°Π΄Π°Ρ‚ΡŒ для ΠΏΡ€Π°Π²ΠΈΠ»ΡŒΠ½ΠΎΠΉ записи ΡƒΡ€Π°Π²Π½Π΅Π½ΠΈΠΉ, ΠΎΠΏΠΈΡΡ‹Π²Π°ΡŽΡ‰ΠΈΡ… процСссы Π² элСктричСской Ρ†Π΅ΠΏΠΈ ΠΈΠ»ΠΈ Π΅Π΅ элСмСнтах. На схСмС (рис. 1.2) стрСлками ΡƒΠΊΠ°ΠΆΠ΅ΠΌ ΠΏΠΎΠ»ΠΎΠΆΠΈΡ‚Π΅Π»ΡŒΠ½Ρ‹Π΅ направлСния Π­Π”Π‘, напряТСний ΠΈ Ρ‚ΠΎΠΊΠΎΠ²:

    Π°) для Π­Π”Π‘ источников – ΠΏΡ€ΠΎΠΈΠ·Π²ΠΎΠ»ΡŒΠ½ΠΎ, Π½ΠΎ ΠΏΡ€ΠΈ этом слСдуСт ΡƒΡ‡ΠΈΡ‚Ρ‹Π²Π°Ρ‚ΡŒ, Ρ‡Ρ‚ΠΎ полюс (Π·Π°ΠΆΠΈΠΌ источника), ΠΊ ΠΊΠΎΡ‚ΠΎΡ€ΠΎΠΌΡƒ Π½Π°ΠΏΡ€Π°Π²Π»Π΅Π½Π° стрСлка, ΠΈΠΌΠ΅Π΅Ρ‚ Π±ΠΎΠ»Π΅Π΅ высокий ΠΏΠΎΡ‚Π΅Π½Ρ†ΠΈΠ°Π» ΠΏΠΎ ΠΎΡ‚Π½ΠΎΡˆΠ΅Π½ΠΈΡŽ ΠΊ Π΄Ρ€ΡƒΠ³ΠΎΠΌΡƒ ΠΏΠΎΠ»ΡŽΡΡƒ;

    Π±) для Ρ‚ΠΎΠΊΠΎΠ² Π² вСтвях, содСрТащих источники Π­Π”Π‘ – ΡΠΎΠ²ΠΏΠ°Π΄Π°ΡŽΡ‰ΠΈΠΌΠΈ с Π½Π°ΠΏΡ€Π°Π²Π»Π΅Π½ΠΈΠ΅ΠΌ Π­Π”Π‘; Π²ΠΎ всСх Π΄Ρ€ΡƒΠ³ΠΈΡ… вСтвях ΠΏΡ€ΠΎΠΈΠ·Π²ΠΎΠ»ΡŒΠ½ΠΎ;

    Π²) для напряТСний – ΡΠΎΠ²ΠΏΠ°Π΄Π°ΡŽΡ‰ΠΈΠΌΠΈ с Π½Π°ΠΏΡ€Π°Π²Π»Π΅Π½ΠΈΠ΅ΠΌ Ρ‚ΠΎΠΊΠ° Π² Π²Π΅Ρ‚Π²ΠΈ ΠΈΠ»ΠΈ элСмСнта Ρ†Π΅ΠΏΠΈ.

    ВсС элСктричСскиС Ρ†Π΅ΠΏΠΈ дСлятся Π½Π° Π»ΠΈΠ½Π΅ΠΉΠ½Ρ‹Π΅ ΠΈ Π½Π΅Π»ΠΈΠ½Π΅ΠΉΠ½Ρ‹Π΅.

    Π­Π»Π΅ΠΌΠ΅Π½Ρ‚ элСктричСской Ρ†Π΅ΠΏΠΈ, ΠΏΠ°Ρ€Π°ΠΌΠ΅Ρ‚Ρ€Ρ‹ ΠΊΠΎΡ‚ΠΎΡ€ΠΎΠ³ΠΎ (сопротивлСниС ΠΈ Π΄Ρ€.) Π½Π΅ зависят ΠΎΡ‚ Ρ‚ΠΎΠΊΠ° Π² Π½Π΅ΠΌ, Π½Π°Π·Ρ‹Π²Π°ΡŽΡ‚ Π»ΠΈΠ½Π΅ΠΉΠ½Ρ‹ΠΌ, Π½Π°ΠΏΡ€ΠΈΠΌΠ΅Ρ€ ΡΠ»Π΅ΠΊΡ‚Ρ€ΠΎΠΏΠ΅Ρ‡ΡŒ.

    НСлинСйный элСмСнт, Π½Π°ΠΏΡ€ΠΈΠΌΠ΅Ρ€ Π»Π°ΠΌΠΏΠ° накаливания, ΠΈΠΌΠ΅Π΅Ρ‚ сопротивлСниС, Π²Π΅Π»ΠΈΡ‡ΠΈΠ½Π° ΠΊΠΎΡ‚ΠΎΡ€ΠΎΠ³ΠΎ увСличиваСтся ΠΏΡ€ΠΈ ΠΏΠΎΠ²Ρ‹ΡˆΠ΅Π½ΠΈΠΈ напряТСния, Π° ΡΠ»Π΅Π΄ΠΎΠ²Π°Ρ‚Π΅Π»ΡŒΠ½ΠΎ ΠΈ Ρ‚ΠΎΠΊΠ°, ΠΏΠΎΠ΄Π²ΠΎΠ΄ΠΈΠΌΠΎΠ³ΠΎ ΠΊ Π»Π°ΠΌΠΏΠΎΡ‡ΠΊΠ΅.

    Π‘Π»Π΅Π΄ΠΎΠ²Π°Ρ‚Π΅Π»ΡŒΠ½ΠΎ, Π² Π»ΠΈΠ½Π΅ΠΉΠ½ΠΎΠΉ элСктричСской Ρ†Π΅ΠΏΠΈ всС элСмСнты – Π»ΠΈΠ½Π΅ΠΉΠ½Ρ‹Π΅, Π° Π½Π΅Π»ΠΈΠ½Π΅ΠΉΠ½ΠΎΠΉ Π½Π°Π·Ρ‹Π²Π°ΡŽΡ‚ ΡΠ»Π΅ΠΊΡ‚Ρ€ΠΈΡ‡Π΅ΡΠΊΡƒΡŽ Ρ†Π΅ΠΏΡŒ, ΡΠΎΠ΄Π΅Ρ€ΠΆΠ°Ρ‰ΡƒΡŽ хотя Π±Ρ‹ ΠΎΠ΄ΠΈΠ½ Π½Π΅Π»ΠΈΠ½Π΅ΠΉΠ½Ρ‹ΠΉ элСмСнт.

    1.3. ΠžΡΠ½ΠΎΠ²Π½Ρ‹Π΅ Π·Π°ΠΊΠΎΠ½Ρ‹ Ρ†Π΅ΠΏΠ΅ΠΉ постоянного Ρ‚ΠΎΠΊΠ°

    РасчСт ΠΈ Π°Π½Π°Π»ΠΈΠ· элСктричСских Ρ†Π΅ΠΏΠ΅ΠΉ производится с использованиСм Π·Π°ΠΊΠΎΠ½Π° Ома, ΠΏΠ΅Ρ€Π²ΠΎΠ³ΠΎ ΠΈ Π²Ρ‚ΠΎΡ€ΠΎΠ³ΠΎ Π·Π°ΠΊΠΎΠ½ΠΎΠ² ΠšΠΈΡ€Ρ…Π³ΠΎΡ„Π°. На основС этих Π·Π°ΠΊΠΎΠ½ΠΎΠ² устанавливаСтся взаимосвязь ΠΌΠ΅ΠΆΠ΄Ρƒ значСниями Ρ‚ΠΎΠΊΠΎΠ², напряТСний, Π­Π”Π‘ всСй элСктричСской Ρ†Π΅ΠΏΠΈ ΠΈ ΠΎΡ‚Π΄Π΅Π»ΡŒΠ½Ρ‹Ρ… Π΅Π΅ участков ΠΈ ΠΏΠ°Ρ€Π°ΠΌΠ΅Ρ‚Ρ€Π°ΠΌΠΈ элСмСнтов, входящих Π² состав этой Ρ†Π΅ΠΏΠΈ.

    Π—Π°ΠΊΠΎΠ½ Ома для участка Ρ†Π΅ΠΏΠΈ

    Π‘ΠΎΠΎΡ‚Π½ΠΎΡˆΠ΅Π½ΠΈΠ΅ ΠΌΠ΅ΠΆΠ΄Ρƒ Ρ‚ΠΎΠΊΠΎΠΌ $I$, напряТСниСм $UR$ ΠΈ сопротивлСниСм $R$ участка Π°b элСктричСской Ρ†Π΅ΠΏΠΈ (рис. 1.3) выраТаСтся Π·Π°ΠΊΠΎΠ½ΠΎΠΌ Ома


    Рис. 1.3

    ΠΈΠ»ΠΈ $U_R = RI$.

    Π’ этом случаС $U_R=RI$ – Π½Π°Π·Ρ‹Π²Π°ΡŽΡ‚ напряТСниСм ΠΈΠ»ΠΈ ΠΏΠ°Π΄Π΅Π½ΠΈΠ΅ΠΌ напряТСния Π½Π° рСзисторС $R$, Π° – Ρ‚ΠΎΠΊΠΎΠΌ Π² рСзисторС $R$.

    ΠŸΡ€ΠΈ расчСтС элСктричСских Ρ†Π΅ΠΏΠ΅ΠΉ ΠΈΠ½ΠΎΠ³Π΄Π° ΡƒΠ΄ΠΎΠ±Π½Π΅Π΅ ΠΏΠΎΠ»ΡŒΠ·ΠΎΠ²Π°Ρ‚ΡŒΡΡ Π½Π΅ сопротивлСниСм $R$, Π° Π²Π΅Π»ΠΈΡ‡ΠΈΠ½ΠΎΠΉ ΠΎΠ±Ρ€Π°Ρ‚Π½ΠΎΠΉ ΡΠΎΠΏΡ€ΠΎΡ‚ΠΈΠ²Π»Π΅Π½ΠΈΡŽ, Ρ‚.Π΅. элСктричСской ΠΏΡ€ΠΎΠ²ΠΎΠ΄ΠΈΠΌΠΎΡΡ‚ΡŒΡŽ:

    .

    Π’ этом случаС Π·Π°ΠΊΠΎΠ½ Ома для участка Ρ†Π΅ΠΏΠΈ Π·Π°ΠΏΠΈΡˆΠ΅Ρ‚ΡΡ Π² Π²ΠΈΠ΄Π΅:

    Π—Π°ΠΊΠΎΠ½ Ома для всСй Ρ†Π΅ΠΏΠΈ

    Π­Ρ‚ΠΎΡ‚ Π·Π°ΠΊΠΎΠ½ опрСдСляСт Π·Π°Π²ΠΈΡΠΈΠΌΠΎΡΡ‚ΡŒ ΠΌΠ΅ΠΆΠ΄Ρƒ Π­Π”Π‘ $E$ источника питания с Π²Π½ΡƒΡ‚Ρ€Π΅Π½Π½ΠΈΠΌ сопротивлСниСм $r_0$ (рис. 1.3), Ρ‚ΠΎΠΊΠΎΠΌ $I$ элСктричСской Ρ†Π΅ΠΏΠΈ ΠΈ ΠΎΠ±Ρ‰ΠΈΠΌ эквивалСнтным сопротивлСниСм $R_Π­=r_0+R$ всСй Ρ†Π΅ΠΏΠΈ:

    .

    БлоТная элСктричСская Ρ†Π΅ΠΏΡŒ содСрТит, ΠΊΠ°ΠΊ ΠΏΡ€Π°Π²ΠΈΠ»ΠΎ, нСсколько Π²Π΅Ρ‚Π²Π΅ΠΉ, Π² ΠΊΠΎΡ‚ΠΎΡ€Ρ‹Π΅ ΠΌΠΎΠ³ΡƒΡ‚ Π±Ρ‹Ρ‚ΡŒ Π²ΠΊΠ»ΡŽΡ‡Π΅Π½Ρ‹ свои источники питания ΠΈ Ρ€Π΅ΠΆΠΈΠΌ Π΅Π΅ Ρ€Π°Π±ΠΎΡ‚Ρ‹ Π½Π΅ ΠΌΠΎΠΆΠ΅Ρ‚ Π±Ρ‹Ρ‚ΡŒ описан Ρ‚ΠΎΠ»ΡŒΠΊΠΎ Π·Π°ΠΊΠΎΠ½ΠΎΠΌ Ома. Но это ΠΌΠΎΠΆΠ½ΠΎ Π²Ρ‹ΠΏΠΎΠ»Π½ΠΈΡ‚ΡŒ Π½Π° основании ΠΏΠ΅Ρ€Π²ΠΎΠ³ΠΎ ΠΈ Π²Ρ‚ΠΎΡ€ΠΎΠ³ΠΎ Π·Π°ΠΊΠΎΠ½ΠΎΠ² ΠšΠΈΡ€Ρ…Π³ΠΎΡ„Π°, ΡΠ²Π»ΡΡŽΡ‰ΠΈΡ…ΡΡ слСдствиСм Π·Π°ΠΊΠΎΠ½Π° сохранСния энСргии.

    ΠŸΠ΅Ρ€Π²Ρ‹ΠΉ Π·Π°ΠΊΠΎΠ½ ΠšΠΈΡ€Ρ…Π³ΠΎΡ„Π°

    Π’ любом ΡƒΠ·Π»Π΅ элСктричСской Ρ†Π΅ΠΏΠΈ алгСбраичСская сумма Ρ‚ΠΎΠΊΠΎΠ² Ρ€Π°Π²Π½Π° Π½ΡƒΠ»ΡŽ

    ,

    Π³Π΄Π΅ $m$ – число Π²Π΅Ρ‚Π²Π΅ΠΉ ΠΏΠΎΠ΄ΠΊΠ»ΡŽΡ‡Π΅Π½Π½Ρ‹Ρ… ΠΊ ΡƒΠ·Π»Ρƒ.

    ΠŸΡ€ΠΈ записи ΡƒΡ€Π°Π²Π½Π΅Π½ΠΈΠΉ ΠΏΠΎ ΠΏΠ΅Ρ€Π²ΠΎΠΌΡƒ Π·Π°ΠΊΠΎΠ½Ρƒ ΠšΠΈΡ€Ρ…Π³ΠΎΡ„Π° Ρ‚ΠΎΠΊΠΈ, Π½Π°ΠΏΡ€Π°Π²Π»Π΅Π½Π½Ρ‹Π΅ ΠΊ ΡƒΠ·Π»Ρƒ, Π±Π΅Ρ€ΡƒΡ‚ со Π·Π½Π°ΠΊΠΎΠΌ «плюс», Π° Ρ‚ΠΎΠΊΠΈ, Π½Π°ΠΏΡ€Π°Π²Π»Π΅Π½Π½Ρ‹Π΅ ΠΎΡ‚ ΡƒΠ·Π»Π° – со Π·Π½Π°ΠΊΠΎΠΌ «минус». НапримСр, для ΡƒΠ·Π»Π° Π° (см. рис. 1.2) $I-I_1-I_2=0$.

    Π’Ρ‚ΠΎΡ€ΠΎΠΉ Π·Π°ΠΊΠΎΠ½ ΠšΠΈΡ€Ρ…Π³ΠΎΡ„Π°

    Π’ любом Π·Π°ΠΌΠΊΠ½ΡƒΡ‚ΠΎΠΌ ΠΊΠΎΠ½Ρ‚ΡƒΡ€Π΅ элСктричСской Ρ†Π΅ΠΏΠΈ алгСбраичСская сумма Π­Π”Π‘ Ρ€Π°Π²Π½Π° алгСбраичСской суммС ΠΏΠ°Π΄Π΅Π½ΠΈΠΉ напряТСний Π½Π° всСх Π΅Π³ΠΎ участках

    ,

    Π³Π΄Π΅ $n$ – число источников Π­Π”Π‘ Π² ΠΊΠΎΠ½Ρ‚ΡƒΡ€Π΅;
    $m$ – число элСмСнтов с сопротивлСниСм $R_k$ Π² ΠΊΠΎΠ½Ρ‚ΡƒΡ€Π΅;
    $U_k=R_kI_k$ – напряТСниС ΠΈΠ»ΠΈ ΠΏΠ°Π΄Π΅Π½ΠΈΠ΅ напряТСния Π½Π° $k$-ΠΌ элСмСнтС ΠΊΠΎΠ½Ρ‚ΡƒΡ€Π°.

    Для схСмы (рис. 1.2) запишСм ΡƒΡ€Π°Π²Π½Π΅Π½ΠΈΠ΅ ΠΏΠΎ Π²Ρ‚ΠΎΡ€ΠΎΠΌΡƒ Π·Π°ΠΊΠΎΠ½Ρƒ ΠšΠΈΡ€Ρ…Π³ΠΎΡ„Π°:

    Если Π² элСктричСской Ρ†Π΅ΠΏΠΈ Π²ΠΊΠ»ΡŽΡ‡Π΅Π½Ρ‹ источники напряТСний, Ρ‚ΠΎ Π²Ρ‚ΠΎΡ€ΠΎΠΉ Π·Π°ΠΊΠΎΠ½ ΠšΠΈΡ€Ρ…Π³ΠΎΡ„Π° формулируСтся Π² ΡΠ»Π΅Π΄ΡƒΡŽΡ‰Π΅ΠΌ Π²ΠΈΠ΄Π΅: алгСбраичСская сумма напряТСний Π½Π° всСх элСмСнтах ΠΊΠΎΠ½Ρ‚Ρ€Ρƒ, Π²ΠΊΠ»ΡŽΡ‡Π°Ρ источники Π­Π”Π‘ Ρ€Π°Π²Π½Π° Π½ΡƒΠ»ΡŽ

    .

    ΠŸΡ€ΠΈ записи ΡƒΡ€Π°Π²Π½Π΅Π½ΠΈΠΉ ΠΏΠΎ Π²Ρ‚ΠΎΡ€ΠΎΠΌΡƒ Π·Π°ΠΊΠΎΠ½Ρƒ ΠšΠΈΡ€Ρ…Π³ΠΎΡ„Π° Π½Π΅ΠΎΠ±Ρ…ΠΎΠ΄ΠΈΠΌΠΎ:

    1) Π·Π°Π΄Π°Ρ‚ΡŒ условныС ΠΏΠΎΠ»ΠΎΠΆΠΈΡ‚Π΅Π»ΡŒΠ½Ρ‹Π΅ направлСния Π­Π”Π‘, Ρ‚ΠΎΠΊΠΎΠ² ΠΈ напряТСний;

    2) Π²Ρ‹Π±Ρ€Π°Ρ‚ΡŒ Π½Π°ΠΏΡ€Π°Π²Π»Π΅Π½ΠΈΠ΅ ΠΎΠ±Ρ…ΠΎΠ΄Π° ΠΊΠΎΠ½Ρ‚ΡƒΡ€Π°, для ΠΊΠΎΡ‚ΠΎΡ€ΠΎΠ³ΠΎ записываСтся ΡƒΡ€Π°Π²Π½Π΅Π½ΠΈΠ΅;

    3) Π·Π°ΠΏΠΈΡΠ°Ρ‚ΡŒ ΡƒΡ€Π°Π²Π½Π΅Π½ΠΈΠ΅, ΠΏΠΎΠ»ΡŒΠ·ΡƒΡΡΡŒ ΠΎΠ΄Π½ΠΎΠΉ ΠΈΠ· Ρ„ΠΎΡ€ΠΌΡƒΠ»ΠΈΡ€ΠΎΠ²ΠΎΠΊ Π²Ρ‚ΠΎΡ€ΠΎΠ³ΠΎ Π·Π°ΠΊΠΎΠ½Π° ΠšΠΈΡ€Ρ…Π³ΠΎΡ„Π°, ΠΏΡ€ΠΈΡ‡Π΅ΠΌ слагаСмыС, входящиС Π² ΡƒΡ€Π°Π²Π½Π΅Π½ΠΈΠ΅, Π±Π΅Ρ€ΡƒΡ‚ со Π·Π½Π°ΠΊΠΎΠΌ «плюс», Ссли ΠΈΡ… условныС ΠΏΠΎΠ»ΠΎΠΆΠΈΡ‚Π΅Π»ΡŒΠ½Ρ‹Π΅ направлСния ΡΠΎΠ²ΠΏΠ°Π΄Π°ΡŽΡ‚ с ΠΎΠ±Ρ…ΠΎΠ΄ΠΎΠΌ ΠΊΠΎΠ½Ρ‚ΡƒΡ€Π°, ΠΈ со Π·Π½Π°ΠΊΠΎΠΌ «минус», Ссли ΠΎΠ½ΠΈ ΠΏΡ€ΠΎΡ‚ΠΈΠ²ΠΎΠΏΠΎΠ»ΠΎΠΆΠ½Ρ‹.

    Π—Π°ΠΏΠΈΡˆΠ΅ΠΌ уравнСния ΠΏΠΎ II Π·Π°ΠΊΠΎΠ½Ρƒ ΠšΠΈΡ€Ρ…Π³ΠΎΡ„Π° для ΠΊΠΎΠ½Ρ‚ΡƒΡ€ΠΎΠ² элСктричСской схСмы (рис. 1.2):

    ΠΊΠΎΠ½Ρ‚ΡƒΡ€ I: $E = RI + R_1I_1 + r_0I$,

    ΠΊΠΎΠ½Ρ‚ΡƒΡ€ II: $R_1I_1 + R_2I_2 = 0$,

    ΠΊΠΎΠ½Ρ‚ΡƒΡ€ III: $E = RI + R_2I_2 + r_0I$.

    Π’ Π΄Π΅ΠΉΡΡ‚Π²ΡƒΡŽΡ‰Π΅ΠΉ Ρ†Π΅ΠΏΠΈ элСктричСская энСргия источника питания прСобразуСтся Π² Π΄Ρ€ΡƒΠ³ΠΈΠ΅ Π²ΠΈΠ΄Ρ‹ энСргии. На участкС Ρ†Π΅ΠΏΠΈ с сопротивлСниСм $R$ Π² Ρ‚Π΅Ρ‡Π΅Π½ΠΈΠ΅ Π²Ρ€Π΅ΠΌΠ΅Π½ΠΈ $t$ ΠΏΡ€ΠΈ Ρ‚ΠΎΠΊΠ΅ $I$ расходуСтся элСктричСская энСргия

    Π‘ΠΊΠΎΡ€ΠΎΡΡ‚ΡŒ прСобразования элСктричСской энСргии Π² Π΄Ρ€ΡƒΠ³ΠΈΠ΅ Π²ΠΈΠ΄Ρ‹ прСдставляСт ΡΠ»Π΅ΠΊΡ‚Ρ€ΠΈΡ‡Π΅ΡΠΊΡƒΡŽ ΠΌΠΎΡ‰Π½ΠΎΡΡ‚ΡŒ

    .

    Из Π·Π°ΠΊΠΎΠ½Π° сохранСния энСргии слСдуСт, Ρ‡Ρ‚ΠΎ ΠΌΠΎΡ‰Π½ΠΎΡΡ‚ΡŒ источников питания Π² любой ΠΌΠΎΠΌΠ΅Π½Ρ‚ Π²Ρ€Π΅ΠΌΠ΅Π½ΠΈ Ρ€Π°Π²Π½Π° суммС мощностСй, расходуСмой Π½Π° всСх участках Ρ†Π΅ΠΏΠΈ.

    .

    Π­Ρ‚ΠΎ ΡΠΎΠΎΡ‚Π½ΠΎΡˆΠ΅Π½ΠΈΠ΅ (1.8) Π½Π°Π·Ρ‹Π²Π°ΡŽΡ‚ ΡƒΡ€Π°Π²Π½Π΅Π½ΠΈΠ΅ΠΌ баланса мощностСй. ΠŸΡ€ΠΈ составлСнии уравнСния баланса мощностСй слСдуСт ΡƒΡ‡Π΅ΡΡ‚ΡŒ, Ρ‡Ρ‚ΠΎ Ссли Π΄Π΅ΠΉΡΡ‚Π²ΠΈΡ‚Π΅Π»ΡŒΠ½Ρ‹Π΅ направлСния Π­Π”Π‘ ΠΈ Ρ‚ΠΎΠΊΠ° источника ΡΠΎΠ²ΠΏΠ°Π΄Π°ΡŽΡ‚, Ρ‚ΠΎ источник Π­Π”Π‘ Ρ€Π°Π±ΠΎΡ‚Π°Π΅Ρ‚ Π² Ρ€Π΅ΠΆΠΈΠΌΠ΅ источника питания, ΠΈ ΠΏΡ€ΠΎΠΈΠ·Π²Π΅Π΄Π΅Π½ΠΈΠ΅ $EI$ ΠΏΠΎΠ΄ΡΡ‚Π°Π²Π»ΡΡŽΡ‚ Π² (1.8) со Π·Π½Π°ΠΊΠΎΠΌ плюс.2R_2$.

    ΠŸΡ€ΠΈ расчСтС элСктричСских Ρ†Π΅ΠΏΠ΅ΠΉ ΠΈΡΠΏΠΎΠ»ΡŒΠ·ΡƒΡŽΡ‚ΡΡ ΠΎΠΏΡ€Π΅Π΄Π΅Π»Π΅Π½Π½Ρ‹Π΅ Π΅Π΄ΠΈΠ½ΠΈΡ†Ρ‹ измСрСния. ЭлСктричСский Ρ‚ΠΎΠΊ измСряСтся Π² Π°ΠΌΠΏΠ΅Ρ€Π°Ρ… (А), напряТСниС – Π² Π²ΠΎΠ»ΡŒΡ‚Π°Ρ… (Π’), сопротивлСниС – Π² ΠΎΠΌΠ°Ρ… (Ом), ΠΌΠΎΡ‰Π½ΠΎΡΡ‚ΡŒ – Π² Π²Π°Ρ‚Ρ‚Π°Ρ… (Π’Ρ‚), элСктричСская энСргия – Π²Π°Ρ‚Ρ‚-час (Π’Ρ‚-час) ΠΈ ΠΏΡ€ΠΎΠ²ΠΎΠ΄ΠΈΠΌΠΎΡΡ‚ΡŒ – Π² симСнсах (Π‘ΠΌ)

    ΠšΡ€ΠΎΠΌΠ΅ основных Π΅Π΄ΠΈΠ½ΠΈΡ† ΠΈΡΠΏΠΎΠ»ΡŒΠ·ΡƒΡŽΡ‚ Π±ΠΎΠ»Π΅Π΅ ΠΌΠ΅Π»ΠΊΠΈΠ΅ ΠΈ Π±ΠΎΠ»Π΅Π΅ ΠΊΡ€ΡƒΠΏΠ½Ρ‹Π΅ Π΅Π΄ΠΈΠ½ΠΈΡ†Ρ‹ измСрСния: ΠΌΠΈΠ»Π»ΠΈΠ°ΠΌΠΏΠ΅Ρ€ (1 ΠΌA = 10 –3 А), ΠΊΠΈΠ»ΠΎΠ°ΠΌΠΏΠ΅Ρ€ (1 ΠΊA = 10 3 А), ΠΌΠΈΠ»Π»ΠΈΠ²ΠΎΠ»ΡŒΡ‚ (1 ΠΌΠ’ = 10 –3 Π’), ΠΊΠΈΠ»ΠΎΠ²ΠΎΠ»ΡŒΡ‚ (1 ΠΊΠ’ = 10 3 Π’), ΠΊΠΈΠ»ΠΎΠΎΠΌ (1 кОм = 10 3 Ом), ΠΌΠ΅Π³Π°ΠΎΠΌ (1 МОм = 10 6 Ом), ΠΊΠΈΠ»ΠΎΠ²Π°Ρ‚Ρ‚ (1 ΠΊΠ’Ρ‚ = 10 3 Π’Ρ‚), ΠΊΠΈΠ»ΠΎΠ²Π°Ρ‚Ρ‚-час (1 ΠΊΠ’Ρ‚-час = 10 3 Π²Π°Ρ‚Ρ‚-час).

    1.4. Бпособы соСдинСния сопротивлСний ΠΈ расчСт эквивалСнтного
    сопротивлСния элСктричСской Ρ†Π΅ΠΏΠΈ

    БопротивлСния Π² элСктричСских цСпях ΠΌΠΎΠ³ΡƒΡ‚ Π±Ρ‹Ρ‚ΡŒ соСдинСны ΠΏΠΎΡΠ»Π΅Π΄ΠΎΠ²Π°Ρ‚Π΅Π»ΡŒΠ½ΠΎ, ΠΏΠ°Ρ€Π°Π»Π»Π΅Π»ΡŒΠ½ΠΎ, ΠΏΠΎ смСшанной схСмС ΠΈ ΠΏΠΎ схСмам Β«Π·Π²Π΅Π·Π΄Π°Β», Β«Ρ‚Ρ€Π΅ΡƒΠ³ΠΎΠ»ΡŒΠ½ΠΈΠΊΒ». РасчСт слоТной схСмы упрощаСтся, Ссли сопротивлСния Π² этой схСмС Π·Π°ΠΌΠ΅Π½ΡΡŽΡ‚ΡΡ ΠΎΠ΄Π½ΠΈΠΌ эквивалСнтным сопротивлСниСм $R_$, ΠΈ вся схСма прСдставляСтся Π² Π²ΠΈΠ΄Π΅ схСмы Π½Π° рис. 1.3, Π³Π΄Π΅ $R=R_$, Π° расчСт Ρ‚ΠΎΠΊΠΎΠ² ΠΈ напряТСний производится с ΠΏΠΎΠΌΠΎΡ‰ΡŒΡŽ Π·Π°ΠΊΠΎΠ½ΠΎΠ² Ома ΠΈ ΠšΠΈΡ€Ρ…Π³ΠΎΡ„Π°.

    ЭлСктричСская Ρ†Π΅ΠΏΡŒ с ΠΏΠΎΡΠ»Π΅Π΄ΠΎΠ²Π°Ρ‚Π΅Π»ΡŒΠ½Ρ‹ΠΌ соСдинСниСм элСмСнтов

    ΠŸΠΎΡΠ»Π΅Π΄ΠΎΠ²Π°Ρ‚Π΅Π»ΡŒΠ½Ρ‹ΠΌ Π½Π°Π·Ρ‹Π²Π°ΡŽΡ‚ Ρ‚Π°ΠΊΠΎΠ΅ соСдинСниС элСмСнтов Ρ†Π΅ΠΏΠΈ, ΠΏΡ€ΠΈ ΠΊΠΎΡ‚ΠΎΡ€ΠΎΠΌ Π²ΠΎ всСх Π²ΠΊΠ»ΡŽΡ‡Π΅Π½Π½Ρ‹Ρ… Π² Ρ†Π΅ΠΏΡŒ элСмСнтах Π²ΠΎΠ·Π½ΠΈΠΊΠ°Π΅Ρ‚ ΠΎΠ΄ΠΈΠ½ ΠΈ Ρ‚ΠΎΡ‚ ΠΆΠ΅ Ρ‚ΠΎΠΊ $I$ (рис. 1.4).

    На основании Π²Ρ‚ΠΎΡ€ΠΎΠ³ΠΎ Π·Π°ΠΊΠΎΠ½Π° ΠšΠΈΡ€Ρ…Π³ΠΎΡ„Π° (1.5) ΠΎΠ±Ρ‰Π΅Π΅ напряТСниС $U$ всСй Ρ†Π΅ΠΏΠΈ Ρ€Π°Π²Π½ΠΎ суммС напряТСний Π½Π° ΠΎΡ‚Π΄Π΅Π»ΡŒΠ½Ρ‹Ρ… участках:

    $U = U_1 + U_2 + U_3$ ΠΈΠ»ΠΈ $IR_ = IR_1 + IR_2 + IR_3$,

    $R_ = R_1 + R_2 + R_3$.

    Π’Π°ΠΊΠΈΠΌ ΠΎΠ±Ρ€Π°Π·ΠΎΠΌ, ΠΏΡ€ΠΈ ΠΏΠΎΡΠ»Π΅Π΄ΠΎΠ²Π°Ρ‚Π΅Π»ΡŒΠ½ΠΎΠΌ соСдинСнии элСмСнтов Ρ†Π΅ΠΏΠΈ ΠΎΠ±Ρ‰Π΅Π΅ эквивалСнтноС сопротивлСниС Ρ†Π΅ΠΏΠΈ Ρ€Π°Π²Π½ΠΎ арифмСтичСской суммС сопротивлСний ΠΎΡ‚Π΄Π΅Π»ΡŒΠ½Ρ‹Ρ… участков. Π‘Π»Π΅Π΄ΠΎΠ²Π°Ρ‚Π΅Π»ΡŒΠ½ΠΎ, Ρ†Π΅ΠΏΡŒ с Π»ΡŽΠ±Ρ‹ΠΌ числом ΠΏΠΎΡΠ»Π΅Π΄ΠΎΠ²Π°Ρ‚Π΅Π»ΡŒΠ½ΠΎ Π²ΠΊΠ»ΡŽΡ‡Π΅Π½Π½Ρ‹Ρ… сопротивлСний ΠΌΠΎΠΆΠ½ΠΎ Π·Π°ΠΌΠ΅Π½ΠΈΡ‚ΡŒ простой Ρ†Π΅ΠΏΡŒΡŽ с ΠΎΠ΄Π½ΠΈΠΌ эквивалСнтным сопротивлСниСм $R_$ (рис. 1.5). ПослС этого расчСт Ρ†Π΅ΠΏΠΈ сводится ΠΊ ΠΎΠΏΡ€Π΅Π΄Π΅Π»Π΅Π½ΠΈΡŽ Ρ‚ΠΎΠΊΠ° $I$ всСй Ρ†Π΅ΠΏΠΈ ΠΏΠΎ Π·Π°ΠΊΠΎΠ½Ρƒ Ома

    ,

    ΠΈ ΠΏΠΎ Π²Ρ‹ΡˆΠ΅ΠΏΡ€ΠΈΠ²Π΅Π΄Π΅Π½Π½Ρ‹ΠΌ Ρ„ΠΎΡ€ΠΌΡƒΠ»Π°ΠΌ Ρ€Π°ΡΡΡ‡ΠΈΡ‚Ρ‹Π²Π°ΡŽΡ‚ ΠΏΠ°Π΄Π΅Π½ΠΈΠ΅ напряТСний $U_1,

    U_3$ Π½Π° ΡΠΎΠΎΡ‚Π²Π΅Ρ‚ΡΡ‚Π²ΡƒΡŽΡ‰ΠΈΡ… участках элСктричСской Ρ†Π΅ΠΏΠΈ (рис. 1.4).

    НСдостаток ΠΏΠΎΡΠ»Π΅Π΄ΠΎΠ²Π°Ρ‚Π΅Π»ΡŒΠ½ΠΎΠ³ΠΎ Π²ΠΊΠ»ΡŽΡ‡Π΅Π½ΠΈΡ элСмСнтов Π·Π°ΠΊΠ»ΡŽΡ‡Π°Π΅Ρ‚ΡΡ Π² Ρ‚ΠΎΠΌ, Ρ‡Ρ‚ΠΎ ΠΏΡ€ΠΈ Π²Ρ‹Ρ…ΠΎΠ΄Π΅ ΠΈΠ· строя хотя Π±Ρ‹ ΠΎΠ΄Π½ΠΎΠ³ΠΎ элСмСнта, прСкращаСтся Ρ€Π°Π±ΠΎΡ‚Π° всСх ΠΎΡΡ‚Π°Π»ΡŒΠ½Ρ‹Ρ… элСмСнтов Ρ†Π΅ΠΏΠΈ.

    ЭлСктричСская Ρ†Π΅ΠΏΡŒ с ΠΏΠ°Ρ€Π°Π»Π»Π΅Π»ΡŒΠ½Ρ‹ΠΌ соСдинСниСм элСмСнтов

    ΠŸΠ°Ρ€Π°Π»Π»Π΅Π»ΡŒΠ½Ρ‹ΠΌ Π½Π°Π·Ρ‹Π²Π°ΡŽΡ‚ Ρ‚Π°ΠΊΠΎΠ΅ соСдинСниС, ΠΏΡ€ΠΈ ΠΊΠΎΡ‚ΠΎΡ€ΠΎΠΌ всС Π²ΠΊΠ»ΡŽΡ‡Π΅Π½Π½Ρ‹Π΅ Π² Ρ†Π΅ΠΏΡŒ ΠΏΠΎΡ‚Ρ€Π΅Π±ΠΈΡ‚Π΅Π»ΠΈ элСктричСской энСргии, находятся ΠΏΠΎΠ΄ ΠΎΠ΄Π½ΠΈΠΌ ΠΈ Ρ‚Π΅ΠΌ ΠΆΠ΅ напряТСниСм (рис. 1.6).

    Π’ этом случаС ΠΎΠ½ΠΈ присоСдинСны ΠΊ Π΄Π²ΡƒΠΌ ΡƒΠ·Π»Π°ΠΌ Ρ†Π΅ΠΏΠΈ Π° ΠΈ b, ΠΈ Π½Π° основании ΠΏΠ΅Ρ€Π²ΠΎΠ³ΠΎ Π·Π°ΠΊΠΎΠ½Π° ΠšΠΈΡ€Ρ…Π³ΠΎΡ„Π° (1.3) ΠΌΠΎΠΆΠ½ΠΎ Π·Π°ΠΏΠΈΡΠ°Ρ‚ΡŒ, Ρ‡Ρ‚ΠΎ ΠΎΠ±Ρ‰ΠΈΠΉ Ρ‚ΠΎΠΊ $I$ всСй Ρ†Π΅ΠΏΠΈ Ρ€Π°Π²Π΅Π½ алгСбраичСской суммС Ρ‚ΠΎΠΊΠΎΠ² ΠΎΡ‚Π΄Π΅Π»ΡŒΠ½Ρ‹Ρ… Π²Π΅Ρ‚Π²Π΅ΠΉ:

    $I = I_1 + I_2 + I_3$, Ρ‚.Π΅. ,

    ΠΎΡ‚ΠΊΡƒΠ΄Π° слСдуСт, Ρ‡Ρ‚ΠΎ

    .

    Π’ Ρ‚ΠΎΠΌ случаС, ΠΊΠΎΠ³Π΄Π° ΠΏΠ°Ρ€Π°Π»Π»Π΅Π»ΡŒΠ½ΠΎ Π²ΠΊΠ»ΡŽΡ‡Π΅Π½Ρ‹ Π΄Π²Π° сопротивлСния $R_1$ ΠΈ $R_2$, ΠΎΠ½ΠΈ Π·Π°ΠΌΠ΅Π½ΡΡŽΡ‚ΡΡ ΠΎΠ΄Π½ΠΈΠΌ эквивалСнтным сопротивлСниСм

    .

    Из ΡΠΎΠΎΡ‚Π½ΠΎΡˆΠ΅Π½ΠΈΡ (1.6), слСдуСт, Ρ‡Ρ‚ΠΎ эквивалСнтная ΠΏΡ€ΠΎΠ²ΠΎΠ΄ΠΈΠΌΠΎΡΡ‚ΡŒ Ρ†Π΅ΠΏΠΈ Ρ€Π°Π²Π½Π° арифмСтичСской суммС проводимостСй ΠΎΡ‚Π΄Π΅Π»ΡŒΠ½Ρ‹Ρ… Π²Π΅Ρ‚Π²Π΅ΠΉ:

    $g_ = g_1 + g_2 + g_3$.

    По ΠΌΠ΅Ρ€Π΅ роста числа ΠΏΠ°Ρ€Π°Π»Π»Π΅Π»ΡŒΠ½ΠΎ Π²ΠΊΠ»ΡŽΡ‡Π΅Π½Π½Ρ‹Ρ… ΠΏΠΎΡ‚Ρ€Π΅Π±ΠΈΡ‚Π΅Π»Π΅ΠΉ ΠΏΡ€ΠΎΠ²ΠΎΠ΄ΠΈΠΌΠΎΡΡ‚ΡŒ Ρ†Π΅ΠΏΠΈ $g_$ возрастаСт, ΠΈ Π½Π°ΠΎΠ±ΠΎΡ€ΠΎΡ‚, ΠΎΠ±Ρ‰Π΅Π΅ сопротивлСниС $R_$ ΡƒΠΌΠ΅Π½ΡŒΡˆΠ°Π΅Ρ‚ΡΡ.

    НапряТСния Π² элСктричСской Ρ†Π΅ΠΏΠΈ с ΠΏΠ°Ρ€Π°Π»Π»Π΅Π»ΡŒΠ½ΠΎ соСдинСнными сопротивлСниями (рис. 1.6)

    $U = IR_ = I_1R_1 = I_2R_2 = I_3R_3$.

    ΠžΡ‚ΡΡŽΠ΄Π° слСдуСт, Ρ‡Ρ‚ΠΎ

    ,

    Ρ‚.Π΅. Ρ‚ΠΎΠΊ Π² Ρ†Π΅ΠΏΠΈ распрСдСляСтся ΠΌΠ΅ΠΆΠ΄Ρƒ ΠΏΠ°Ρ€Π°Π»Π»Π΅Π»ΡŒΠ½Ρ‹ΠΌΠΈ вСтвями ΠΎΠ±Ρ€Π°Ρ‚Π½ΠΎ ΠΏΡ€ΠΎΠΏΠΎΡ€Ρ†ΠΈΠΎΠ½Π°Π»ΡŒΠ½ΠΎ ΠΈΡ… сопротивлСниям.

    По ΠΏΠ°Ρ€Π°Π»Π»Π΅Π»ΡŒΠ½ΠΎ Π²ΠΊΠ»ΡŽΡ‡Π΅Π½Π½ΠΎΠΉ схСмС Ρ€Π°Π±ΠΎΡ‚Π°ΡŽΡ‚ Π² номинальном Ρ€Π΅ΠΆΠΈΠΌΠ΅ ΠΏΠΎΡ‚Ρ€Π΅Π±ΠΈΡ‚Π΅Π»ΠΈ любой мощности, рассчитанныС Π½Π° ΠΎΠ΄Π½ΠΎ ΠΈ Ρ‚ΠΎ ΠΆΠ΅ напряТСниС. ΠŸΡ€ΠΈΡ‡Π΅ΠΌ Π²ΠΊΠ»ΡŽΡ‡Π΅Π½ΠΈΠ΅ ΠΈΠ»ΠΈ ΠΎΡ‚ΠΊΠ»ΡŽΡ‡Π΅Π½ΠΈΠ΅ ΠΎΠ΄Π½ΠΎΠ³ΠΎ ΠΈΠ»ΠΈ Π½Π΅ΡΠΊΠΎΠ»ΡŒΠΊΠΈΡ… ΠΏΠΎΡ‚Ρ€Π΅Π±ΠΈΡ‚Π΅Π»Π΅ΠΉ Π½Π΅ отраТаСтся Π½Π° Ρ€Π°Π±ΠΎΡ‚Π΅ ΠΎΡΡ‚Π°Π»ΡŒΠ½Ρ‹Ρ…. ΠŸΠΎΡΡ‚ΠΎΠΌΡƒ эта схСма являСтся основной схСмой ΠΏΠΎΠ΄ΠΊΠ»ΡŽΡ‡Π΅Π½ΠΈΡ ΠΏΠΎΡ‚Ρ€Π΅Π±ΠΈΡ‚Π΅Π»Π΅ΠΉ ΠΊ источнику элСктричСской энСргии.

    ЭлСктричСская Ρ†Π΅ΠΏΡŒ со ΡΠΌΠ΅ΡˆΠ°Π½Π½Ρ‹ΠΌ соСдинСниСм элСмСнтов

    Π‘ΠΌΠ΅ΡˆΠ°Π½Π½Ρ‹ΠΌ называСтся Ρ‚Π°ΠΊΠΎΠ΅ соСдинСниС, ΠΏΡ€ΠΈ ΠΊΠΎΡ‚ΠΎΡ€ΠΎΠΌ Π² Ρ†Π΅ΠΏΠΈ ΠΈΠΌΠ΅ΡŽΡ‚ΡΡ Π³Ρ€ΡƒΠΏΠΏΡ‹ ΠΏΠ°Ρ€Π°Π»Π»Π΅Π»ΡŒΠ½ΠΎ ΠΈ ΠΏΠΎΡΠ»Π΅Π΄ΠΎΠ²Π°Ρ‚Π΅Π»ΡŒΠ½ΠΎ Π²ΠΊΠ»ΡŽΡ‡Π΅Π½Π½Ρ‹Ρ… сопротивлСний.

    Для Ρ†Π΅ΠΏΠΈ, прСдставлСнной Π½Π° рис. 1.7, расчСт эквивалСнтного сопротивлСния начинаСтся с ΠΊΠΎΠ½Ρ†Π° схСмы. Для упрощСния расчСтов ΠΏΡ€ΠΈΠΌΠ΅ΠΌ, Ρ‡Ρ‚ΠΎ всС сопротивлСния Π² этой схСмС ΡΠ²Π»ΡΡŽΡ‚ΡΡ ΠΎΠ΄ΠΈΠ½Π°ΠΊΠΎΠ²Ρ‹ΠΌΠΈ: $R_1=R_2=R_3=R_4=R_5=R$. БопротивлСния $R_4$ ΠΈ $R_5$ Π²ΠΊΠ»ΡŽΡ‡Π΅Π½Ρ‹ ΠΏΠ°Ρ€Π°Π»Π»Π΅Π»ΡŒΠ½ΠΎ, Ρ‚ΠΎΠ³Π΄Π° сопротивлСниС участка Ρ†Π΅ΠΏΠΈ cd Ρ€Π°Π²Π½ΠΎ:

    .

    Π’ этом случаС ΠΈΡΡ…ΠΎΠ΄Π½ΡƒΡŽ схСму (рис. 1.7) ΠΌΠΎΠΆΠ½ΠΎ ΠΏΡ€Π΅Π΄ΡΡ‚Π°Π²ΠΈΡ‚ΡŒ Π² ΡΠ»Π΅Π΄ΡƒΡŽΡ‰Π΅ΠΌ Π²ΠΈΠ΄Π΅ (рис. 1.8):

    На схСмС (рис. 1.8) сопротивлСниС $R_3$ ΠΈ $R_$ соСдинСны ΠΏΠΎΡΠ»Π΅Π΄ΠΎΠ²Π°Ρ‚Π΅Π»ΡŒΠ½ΠΎ, ΠΈ Ρ‚ΠΎΠ³Π΄Π° сопротивлСниС участка Ρ†Π΅ΠΏΠΈ ad Ρ€Π°Π²Π½ΠΎ:

    .

    Π’ΠΎΠ³Π΄Π° схСму (рис. 1.8) ΠΌΠΎΠΆΠ½ΠΎ ΠΏΡ€Π΅Π΄ΡΡ‚Π°Π²ΠΈΡ‚ΡŒ Π² сокращСнном Π²Π°Ρ€ΠΈΠ°Π½Ρ‚Π΅ (рис. 1.9):

    На схСмС (рис. 1.9) сопротивлСниС $R_2$ ΠΈ $R_$ соСдинСны ΠΏΠ°Ρ€Π°Π»Π»Π΅Π»ΡŒΠ½ΠΎ, Ρ‚ΠΎΠ³Π΄Π° сопротивлСниС участка Ρ†Π΅ΠΏΠΈ Π°b Ρ€Π°Π²Π½ΠΎ

    .

    Π‘Ρ…Π΅ΠΌΡƒ (рис. 1.9) ΠΌΠΎΠΆΠ½ΠΎ ΠΏΡ€Π΅Π΄ΡΡ‚Π°Π²ΠΈΡ‚ΡŒ Π² ΡƒΠΏΡ€ΠΎΡ‰Π΅Π½Π½ΠΎΠΌ Π²Π°Ρ€ΠΈΠ°Π½Ρ‚Π΅ (рис. 1.10), Π³Π΄Π΅ сопротивлСния $R_1$ ΠΈ $R_$ Π²ΠΊΠ»ΡŽΡ‡Π΅Π½Ρ‹ ΠΏΠΎΡΠ»Π΅Π΄ΠΎΠ²Π°Ρ‚Π΅Π»ΡŒΠ½ΠΎ.

    Π’ΠΎΠ³Π΄Π° эквивалСнтноС сопрот

    Π—Π°ΠΊΠΎΠ½ Π›Π΅Π½Ρ†Π° — ΠžΠΏΡ€Π΅Π΄Π΅Π»Π΅Π½ΠΈΠ΅, Ρ„ΠΎΡ€ΠΌΡƒΠ»Π° ΠΈ ΠΏΡ€ΠΈΠΌΠ΅Ρ€

    Γ—

    Π˜Π·Π²ΠΈΠ½ΠΈΡ‚Π΅ !, эта страница сСйчас нСдоступна для добавлСния Π² Π·Π°ΠΊΠ»Π°Π΄ΠΊΠΈ.

    Π§Ρ‚ΠΎ Ρ‚Π°ΠΊΠΎΠ΅ Π·Π°ΠΊΠΎΠ½ Π›Π΅Π½Ρ†Π°?

    Π—Π°ΠΊΠΎΠ½ Π›Π΅Π½Ρ†Π°, Π½Π°Π·Π²Π°Π½Π½Ρ‹ΠΉ Π² Ρ‡Π΅ΡΡ‚ΡŒ Ρ„ΠΈΠ·ΠΈΠΊΠ° Эмиля Π›Π΅Π½Ρ†Π°, Π±Ρ‹Π» сформулирован Π² 1834 Π³ΠΎΠ΄Ρƒ.Он ΡƒΡ‚Π²Π΅Ρ€ΠΆΠ΄Π°Π΅Ρ‚, Ρ‡Ρ‚ΠΎ Π½Π°ΠΏΡ€Π°Π²Π»Π΅Π½ΠΈΠ΅ Ρ‚ΠΎΠΊΠ°, ΠΈΠ½Π΄ΡƒΡ†ΠΈΡ€ΠΎΠ²Π°Π½Π½ΠΎΠ³ΠΎ Π² ΠΏΡ€ΠΎΠ²ΠΎΠ΄Π½ΠΈΠΊΠ΅ ΠΈΠ·ΠΌΠ΅Π½ΡΡŽΡ‰ΠΈΠΌΡΡ ΠΌΠ°Π³Π½ΠΈΡ‚Π½Ρ‹ΠΌ ΠΏΠΎΠ»Π΅ΠΌ, Ρ‚Π°ΠΊΠΎΠ²ΠΎ, Ρ‡Ρ‚ΠΎ ΠΌΠ°Π³Π½ΠΈΡ‚Π½ΠΎΠ΅ ΠΏΠΎΠ»Π΅, создаваСмоС ΠΈΠ½Π΄ΡƒΡ†ΠΈΡ€ΠΎΠ²Π°Π½Π½Ρ‹ΠΌ Ρ‚ΠΎΠΊΠΎΠΌ, противодСйствуСт Π½Π°Ρ‡Π°Π»ΡŒΠ½ΠΎΠΌΡƒ ΠΈΠ·ΠΌΠ΅Π½ΡΡŽΡ‰Π΅ΠΌΡƒΡΡ ΠΌΠ°Π³Π½ΠΈΡ‚Π½ΠΎΠΌΡƒ полю.

    Когда Ρ‚ΠΎΠΊ индуцируСтся ΠΌΠ°Π³Π½ΠΈΡ‚Π½Ρ‹ΠΌ ΠΏΠΎΠ»Π΅ΠΌ, Ρ‚ΠΎ ΠΌΠ°Π³Π½ΠΈΡ‚Π½ΠΎΠ΅ ΠΏΠΎΠ»Π΅, создаваСмоС ΠΈΠ½Π΄ΡƒΡ†ΠΈΡ€ΠΎΠ²Π°Π½Π½Ρ‹ΠΌ Ρ‚ΠΎΠΊΠΎΠΌ, создаСт своС ΠΌΠ°Π³Π½ΠΈΡ‚Π½ΠΎΠ΅ ΠΏΠΎΠ»Π΅. Π’Π°ΠΊΠΈΠΌ ΠΎΠ±Ρ€Π°Π·ΠΎΠΌ, этому ΠΌΠ°Π³Π½ΠΈΡ‚Π½ΠΎΠΌΡƒ полю Π±ΡƒΠ΄Π΅Ρ‚ ΠΏΡ€ΠΎΡ‚ΠΈΠ²ΠΎΡΡ‚ΠΎΡΡ‚ΡŒ создавшСС Π΅Π³ΠΎ ΠΌΠ°Π³Π½ΠΈΡ‚Π½ΠΎΠ΅ ΠΏΠΎΠ»Π΅.

    Π—Π°ΠΊΠΎΠ½ Π›Π΅Π½Ρ†Π° основан Π½Π° Π·Π°ΠΊΠΎΠ½Π΅ ΠΈΠ½Π΄ΡƒΠΊΡ†ΠΈΠΈ ЀарадСя, ΠΊΠΎΡ‚ΠΎΡ€Ρ‹ΠΉ гласит, Ρ‡Ρ‚ΠΎ ΠΈΠ·ΠΌΠ΅Π½ΡΡŽΡ‰Π΅Π΅ΡΡ ΠΌΠ°Π³Π½ΠΈΡ‚Π½ΠΎΠ΅ ΠΏΠΎΠ»Π΅ ΠΈΠ½Π΄ΡƒΡ†ΠΈΡ€ΡƒΠ΅Ρ‚ Ρ‚ΠΎΠΊ Π² ΠΏΡ€ΠΎΠ²ΠΎΠ΄Π½ΠΈΠΊΠ΅, Ρ‚ΠΎΠ³Π΄Π° ΠΊΠ°ΠΊ Π·Π°ΠΊΠΎΠ½ Π›Π΅Π½Ρ†Π° сообщаСт Π½Π°ΠΌ Π½Π°ΠΏΡ€Π°Π²Π»Π΅Π½ΠΈΠ΅ ΠΈΠ½Π΄ΡƒΡ†ΠΈΡ€ΠΎΠ²Π°Π½Π½ΠΎΠ³ΠΎ Ρ‚ΠΎΠΊΠ°, ΠΊΠΎΡ‚ΠΎΡ€ΠΎΠ΅ противодСйствуСт Π½Π°Ρ‡Π°Π»ΡŒΠ½ΠΎΠΌΡƒ ΠΈΠ·ΠΌΠ΅Π½ΡΡŽΡ‰Π΅ΠΌΡƒΡΡ ΠΌΠ°Π³Π½ΠΈΡ‚Π½ΠΎΠΌΡƒ полю, ΠΊΠΎΡ‚ΠΎΡ€ΠΎΠ΅ Π΅Π³ΠΎ ΠΏΠΎΡ€ΠΎΠ΄ΠΈΠ»ΠΎ.Π‘Π»Π΅Π΄ΠΎΠ²Π°Ρ‚Π΅Π»ΡŒΠ½ΠΎ, это ΠΎΠ±ΠΎΠ·Π½Π°Ρ‡Π΅Π½ΠΎ Π² Ρ„ΠΎΡ€ΠΌΡƒΠ»Π΅ Π·Π°ΠΊΠΎΠ½Π° ЀарадСя ΠΎΡ‚Ρ€ΠΈΡ†Π°Ρ‚Π΅Π»ΡŒΠ½Ρ‹ΠΌ Π·Π½Π°ΠΊΠΎΠΌ.

    \ [\ epsilon = — \ frac {d \ Phi_ {B}} {dt} \]

    ΠœΠ°Π³Π½ΠΈΡ‚Π½ΠΎΠ΅ ΠΏΠΎΠ»Π΅ ΠΌΠΎΠΆΠ½ΠΎ ΠΈΠ·ΠΌΠ΅Π½ΠΈΡ‚ΡŒ, ΠΈΠ·ΠΌΠ΅Π½ΠΈΠ² Π΅Π³ΠΎ силу, Π»ΠΈΠ±ΠΎ пСрСмСщая ΠΌΠ°Π³Π½ΠΈΡ‚ ΠΊ ΠΊΠ°Ρ‚ΡƒΡˆΠΊΠ΅ ΠΈΠ»ΠΈ ΠΎΡ‚ Π½Π΅Π΅, Π»ΠΈΠ±ΠΎ пСрСмСщая ΠΊΠ°Ρ‚ΡƒΡˆΠΊΠ° находится Π² ΠΌΠ°Π³Π½ΠΈΡ‚Π½ΠΎΠΌ ΠΏΠΎΠ»Π΅ ΠΈΠ»ΠΈ Π²Π½Π΅ Π΅Π³ΠΎ.

    Π‘Π»Π΅Π΄ΠΎΠ²Π°Ρ‚Π΅Π»ΡŒΠ½ΠΎ, ΠΌΡ‹ ΠΌΠΎΠΆΠ΅ΠΌ ΡΠΊΠ°Π·Π°Ρ‚ΡŒ, Ρ‡Ρ‚ΠΎ Π²Π΅Π»ΠΈΡ‡ΠΈΠ½Π° элСктромагнитного поля, ΠΈΠ½Π΄ΡƒΡ†ΠΈΡ€ΠΎΠ²Π°Π½Π½ΠΎΠ³ΠΎ Π² Ρ†Π΅ΠΏΠΈ, ΠΏΡ€ΠΎΠΏΠΎΡ€Ρ†ΠΈΠΎΠ½Π°Π»ΡŒΠ½Π° скорости измСнСния ΠΌΠ°Π³Π½ΠΈΡ‚Π½ΠΎΠ³ΠΎ ΠΏΠΎΡ‚ΠΎΠΊΠ°.

    \ [\ varepsilon \ propto \ frac {d \ Phi} {dt} \].

    Π€ΠΎΡ€ΠΌΡƒΠ»Π° Π·Π°ΠΊΠΎΠ½Π° Π›Π΅Π½Ρ†Π°:

    Богласно Π·Π°ΠΊΠΎΠ½Ρƒ Π›Π΅Π½Ρ†Π°, ΠΊΠΎΠ³Π΄Π° элСктромагнитноС ΠΏΠΎΠ»Π΅ гСнСрируСтся ΠΈΠ·ΠΌΠ΅Π½Π΅Π½ΠΈΠ΅ΠΌ ΠΌΠ°Π³Π½ΠΈΡ‚Π½ΠΎΠ³ΠΎ ΠΏΠΎΡ‚ΠΎΠΊΠ°, ΠΏΠΎΠ»ΡΡ€Π½ΠΎΡΡ‚ΡŒ ΠΈΠ½Π΄ΡƒΡ†ΠΈΡ€ΠΎΠ²Π°Π½Π½ΠΎΠ³ΠΎ элСктромагнитного поля создаСт ΠΈΠ½Π΄ΡƒΡ†ΠΈΡ€ΠΎΠ²Π°Π½Π½Ρ‹ΠΉ Ρ‚ΠΎΠΊ, ΠΌΠ°Π³Π½ΠΈΡ‚Π½ΠΎΠ΅ ΠΏΠΎΠ»Π΅ ΠΊΠΎΡ‚ΠΎΡ€ΠΎΠ³ΠΎ противодСйствуСт Π½Π°Ρ‡Π°Π»ΡŒΠ½ΠΎΠΌΡƒ ΠΈΠ·ΠΌΠ΅Π½ΡΡŽΡ‰Π΅ΠΌΡƒΡΡ ΠΌΠ°Π³Π½ΠΈΡ‚Π½ΠΎΠΌΡƒ полю, ΡΠΎΠ·Π΄Π°Π²ΡˆΠ΅ΠΌΡƒ Π΅Π³ΠΎ.

    Π€ΠΎΡ€ΠΌΡƒΠ»Π° Π·Π°ΠΊΠΎΠ½Π° Π›Π΅Π½Ρ†Π° ΠΏΠΎΠΊΠ°Π·Π°Π½Π° Π½ΠΈΠΆΠ΅:

    \ [\ epsilon = — N \ frac {\ partial \ Phi_ {B}} {\ partial t} \]

    Π“Π΄Π΅,

    Ξ΅ = индуцированная Π­Π”Π‘

    δΦB = ΠΈΠ·ΠΌΠ΅Π½Π΅Π½ΠΈΠ΅ ΠΌΠ°Π³Π½ΠΈΡ‚Π½ΠΎΠ³ΠΎ ΠΏΠΎΡ‚ΠΎΠΊΠ°

    N = количСство Π²ΠΈΡ‚ΠΊΠΎΠ² Π² ΠΊΠ°Ρ‚ΡƒΡˆΠΊΠ΅

    ΠŸΡ€ΠΈΠΌΠ΅Π½Π΅Π½ΠΈΠ΅ Π·Π°ΠΊΠΎΠ½Π° Π›Π΅Π½Ρ†Π°:

    ΠŸΡ€ΠΈΠΌΠ΅Π½Π΅Π½ΠΈΡ Π·Π°ΠΊΠΎΠ½Π° Π›Π΅Π½Ρ†Π° Π²ΠΊΠ»ΡŽΡ‡Π°ΡŽΡ‚:

    1. Когда источник элСктромагнитного поля ΠΏΠΎΠ΄ΠΊΠ»ΡŽΡ‡Π°Π΅Ρ‚ΡΡ ΠΊ ΠΊΠ°Ρ‚ΡƒΡˆΠΊΠ΅ индуктивности, Ρ‚ΠΎΠΊ Π½Π°Ρ‡ΠΈΠ½Π°Π΅Ρ‚ Ρ‚Π΅Ρ‡ΡŒ Ρ‡Π΅Ρ€Π΅Π· Π½Π΅Π³ΠΎ.ΠžΠ±Ρ€Π°Ρ‚Π½ΠΎΠ΅ элСктромагнитноС ΠΏΠΎΠ»Π΅ Π±ΡƒΠ΄Π΅Ρ‚ ΠΏΡ€ΠΎΡ‚ΠΈΠ²ΠΎΠ΄Π΅ΠΉΡΡ‚Π²ΠΎΠ²Π°Ρ‚ΡŒ этому ΡƒΠ²Π΅Π»ΠΈΡ‡Π΅Π½ΠΈΡŽ Ρ‚ΠΎΠΊΠ° Ρ‡Π΅Ρ€Π΅Π· ΠΈΠ½Π΄ΡƒΠΊΡ‚ΠΎΡ€. Π§Ρ‚ΠΎΠ±Ρ‹ ΡƒΡΡ‚Π°Π½ΠΎΠ²ΠΈΡ‚ΡŒ Ρ‚ΠΎΠΊ, внСшний источник элСктромагнитного поля Π΄ΠΎΠ»ΠΆΠ΅Π½ ΠΏΡ€ΠΎΠ΄Π΅Π»Π°Ρ‚ΡŒ Π½Π΅ΠΊΠΎΡ‚ΠΎΡ€ΡƒΡŽ Ρ€Π°Π±ΠΎΡ‚Ρƒ для прСодолСния этого противостояния.

    2. Π—Π°ΠΊΠΎΠ½ Π›Π΅Π½Ρ†Π° ΠΈΡΠΏΠΎΠ»ΡŒΠ·ΡƒΠ΅Ρ‚ΡΡ Π² элСктромагнитных Ρ‚ΠΎΡ€ΠΌΠΎΠ·Π°Ρ… ΠΈ ΠΈΠ½Π΄ΡƒΠΊΡ†ΠΈΠΎΠ½Π½Ρ‹Ρ… Π²Π°Ρ€ΠΎΡ‡Π½Ρ‹Ρ… панСлях.

    3. ΠŸΡ€ΠΈΠΌΠ΅Π½ΡΠ΅Ρ‚ΡΡ Ρ‚Π°ΠΊΠΆΠ΅ для элСктрогСнСраторов, Π³Π΅Π½Π΅Ρ€Π°Ρ‚ΠΎΡ€ΠΎΠ² ΠΏΠ΅Ρ€Π΅ΠΌΠ΅Π½Π½ΠΎΠ³ΠΎ Ρ‚ΠΎΠΊΠ°.

    4. Π’ΠΈΡ…Ρ€Π΅Ρ‚ΠΎΠΊΠΎΠ²Ρ‹Π΅ вСсы

    404: Π‘Ρ‚Ρ€Π°Π½ΠΈΡ†Π° Π½Π΅ Π½Π°ΠΉΠ΄Π΅Π½Π°

    Π§Ρ‚ΠΎ Ρ‚Π°ΠΊΠΎΠ΅.com Π˜Ρ‰ΠΈΡ‚Π΅ тысячи тСхничСских ΠΎΠΏΡ€Π΅Π΄Π΅Π»Π΅Π½ΠΈΠΉ ΠŸΡ€ΠΎΡΠΌΠΎΡ‚Ρ€Π΅Ρ‚ΡŒ опрСдСлСния :
    • А
    • B
    • Π‘
    • D
    • E
    • F
    • G
    • H
    • I
    • Π”ΠΆ
    • К
    • Π»
    • M
    • N
    • O
    • -P
    • Q
    • R
    • S
    • Ρ‚
    • U
    • Π’
    • Π’Ρ‚
    • X
    • Y
    • Z
    • #
    ΠΠ²Ρ‚ΠΎΡ€ΠΈΠ·ΠΎΠ²Π°Ρ‚ΡŒΡΡ рСгистр
    • Π‘Π΅Ρ‚ΡŒ Techtarget
    • ВСхничСский ΡƒΡΠΊΠΎΡ€ΠΈΡ‚Π΅Π»ΡŒ
    RSS
    • Π§Ρ‚ΠΎ Ρ‚Π°ΠΊΠΎΠ΅.com
    • ΠŸΡ€ΠΎΡΠΌΠΎΡ‚Ρ€Π΅Ρ‚ΡŒ опрСдСлСния По Ρ‚Π΅ΠΌΠ΅

      Π’Ρ‹Π±Π΅Ρ€ΠΈΡ‚Π΅ ΠΊΠ°Ρ‚Π΅Π³ΠΎΡ€ΠΈΡŽ

      • AppDev
      • ΠŸΡ€ΠΎΠ³Ρ€Π°ΠΌΠΌΠ½ΠΎΠ΅ обСспСчСниС для бизнСса
      • ΠšΠΎΠΌΠΏΡŒΡŽΡ‚Π΅Ρ€Π½Ρ‹Π΅ Π½Π°ΡƒΠΊΠΈ
      • ΠŸΠΎΡ‚Ρ€Π΅Π±ΠΈΡ‚Π΅Π»ΡŒΡΠΊΠΈΠ΅ Ρ‚Π΅Ρ…Π½ΠΎΠ»ΠΎΠ³ΠΈΠΈ
      • Π”Π°Ρ‚Π°-Ρ†Π΅Π½Ρ‚Ρ€
      • ИВ-ΠΌΠ΅Π½Π΅Π΄ΠΆΠΌΠ΅Π½Ρ‚
      • Π‘Π΅Ρ‚ΡŒ
      • Π‘Π΅Π·ΠΎΠΏΠ°ΡΠ½ΠΎΡΡ‚ΡŒ
      • Π₯Ρ€Π°Π½Π΅Π½ΠΈΠ΅ ΠΈ Π΄Π°Π½Π½Ρ‹Π΅ Mgmt
      AppDev ΠŸΡ€ΠΎΡΠΌΠΎΡ‚Ρ€Π΅Ρ‚ΡŒ всС
      • Agile, Scrum, XP
      • Π―Π±Π»ΠΎΠΊΠΎ
      • DevOps
      • Π˜Π½Ρ‚Π΅Ρ€Π½Π΅Ρ‚-прилоТСния
      • Java
      • Linux
      • Microsoft
      • ΠžΡ‚ΠΊΡ€Ρ‹Ρ‚Ρ‹ΠΉ исходный ΠΊΠΎΠ΄
      • ΠžΠΏΠ΅Ρ€Π°Ρ†ΠΈΠΎΠ½Π½Ρ‹Π΅ систСмы
      • ΠŸΡ€ΠΎΠ³Ρ€Π°ΠΌΠΌΠΈΡ€ΠΎΠ²Π°Π½ΠΈΠ΅
      • ΠŸΡ€ΠΎΠ³Ρ€Π°ΠΌΠΌΠ½Ρ‹Π΅ прилоТСния
      • Π Π°Π·Ρ€Π°Π±ΠΎΡ‚ΠΊΠ° ΠΏΡ€ΠΎΠ³Ρ€Π°ΠΌΠΌΠ½ΠΎΠ³ΠΎ обСспСчСния
      • Π’Π΅Π±-сСрвисы, SOA

    ΠžΠ±Π·ΠΎΡ€: Ρ„ΠΎΡ€ΠΌΡƒΠ»Ρ‹ Π·Π°ΠΊΠΎΠ½Π° Ома — ΡΠΊΠ°Ρ‡Π°Ρ‚ΡŒ ppt

    ΠŸΡ€Π΅Π·Π΅Π½Ρ‚Π°Ρ†ΠΈΡ Π½Π° Ρ‚Π΅ΠΌΡƒ: Β«ΠžΠ±Π·ΠΎΡ€: Ρ„ΠΎΡ€ΠΌΡƒΠ»Ρ‹ Π·Π°ΠΊΠΎΠ½Π° Ома» — стСнограмма ΠΏΡ€Π΅Π·Π΅Π½Ρ‚Π°Ρ†ΠΈΠΈ:

    1 ΠžΠ±Π·ΠΎΡ€: Ρ„ΠΎΡ€ΠΌΡƒΠ»Ρ‹ Π·Π°ΠΊΠΎΠ½Π° Ома
    БущСствуСт Ρ‚Ρ€ΠΈ Ρ„ΠΎΡ€ΠΌΡ‹ Π·Π°ΠΊΠΎΠ½Π° Ома: V = IR I = V / R R = V / I, Π³Π΄Π΅: V = напряТСниС I = Ρ‚ΠΎΠΊ R = сопротивлСниС Рис.3-4: ΠšΡ€ΡƒΠ³ΠΎΠ²Π°Ρ Π΄ΠΈΠ°Π³Ρ€Π°ΠΌΠΌΠ° для ΠΏΠΎΠΌΠΎΡ‰ΠΈ Π² Π·Π°ΠΏΠΎΠΌΠΈΠ½Π°Π½ΠΈΠΈ Ρ„ΠΎΡ€ΠΌΡƒΠ» Π·Π°ΠΊΠΎΠ½Π° Ома V = IR, I = V / R ΠΈ R = V / I. Π‘ΡƒΠΊΠ²Π° V всСгда Π²Π²Π΅Ρ€Ρ…Ρƒ. Copyright Β© McGraw-Hill Companies, Inc. Для воспроизвСдСния ΠΈΠ»ΠΈ дСмонстрации трСбуСтся Ρ€Π°Π·Ρ€Π΅ΡˆΠ΅Π½ΠΈΠ΅.

    2 ΠŸΠΎΡΠ»Π΅Π΄ΠΎΠ²Π°Ρ‚Π΅Π»ΡŒΠ½ΠΎΠ΅ ΠΈ ΠΏΠ°Ρ€Π°Π»Π»Π΅Π»ΡŒΠ½ΠΎΠ΅ соСдинСниС
    ΠŸΠΎΡΠ»Π΅Π΄ΠΎΠ²Π°Ρ‚Π΅Π»ΡŒΠ½ΠΎΠ΅ соСдинСниС ΠŸΠ°Ρ€Π°Π»Π»Π΅Π»ΡŒΠ½ΠΎΠ΅ соСдинСниС

    3 ΠŸΠΎΡΠ»Π΅Π΄ΠΎΠ²Π°Ρ‚Π΅Π»ΡŒΠ½Ρ‹ΠΉ ΠΈ ΠΏΠ°Ρ€Π°Π»Π»Π΅Π»ΡŒΠ½Ρ‹ΠΉ ряд: Π΄Π²Π° ΠΈΠ»ΠΈ Π±ΠΎΠ»Π΅Π΅ элСмСнта Π²ΠΊΠ»ΡŽΡ‡Π΅Π½Ρ‹ ΠΏΠΎΡΠ»Π΅Π΄ΠΎΠ²Π°Ρ‚Π΅Π»ΡŒΠ½ΠΎ, Ссли ΠΎΠ½ΠΈ Π²ΠΊΠ»ΡŽΡ‡Π΅Π½Ρ‹ каскадом ΠΈΠ»ΠΈ соСдинСны ΠΏΠΎΡΠ»Π΅Π΄ΠΎΠ²Π°Ρ‚Π΅Π»ΡŒΠ½ΠΎ ΠΈ, ΡΠ»Π΅Π΄ΠΎΠ²Π°Ρ‚Π΅Π»ΡŒΠ½ΠΎ, нСсут ΠΎΠ΄ΠΈΠ½Π°ΠΊΠΎΠ²Ρ‹ΠΉ Ρ‚ΠΎΠΊ.ΠŸΠ°Ρ€Π°Π»Π»Π΅Π»ΡŒΠ½ΠΎ: Π΄Π²Π° ΠΈΠ»ΠΈ Π±ΠΎΠ»Π΅Π΅ элСмСнта Ρ€Π°Π±ΠΎΡ‚Π°ΡŽΡ‚ ΠΏΠ°Ρ€Π°Π»Π»Π΅Π»ΡŒΠ½ΠΎ, Ссли ΠΎΠ½ΠΈ ΠΏΠΎΠ΄ΠΊΠ»ΡŽΡ‡Π΅Π½Ρ‹ ΠΊ ΠΎΠ΄Π½ΠΈΠΌ ΠΈ Ρ‚Π΅ΠΌ ΠΆΠ΅ Π΄Π²ΡƒΠΌ ΡƒΠ·Π»Π°ΠΌ ΠΈ, ΡΠ»Π΅Π΄ΠΎΠ²Π°Ρ‚Π΅Π»ΡŒΠ½ΠΎ, ΠΈΠΌΠ΅ΡŽΡ‚ ΠΎΠ΄ΠΈΠ½Π°ΠΊΠΎΠ²ΠΎΠ΅ напряТСниС Π½Π° Π½ΠΈΡ….

    4 Π¦Π΅Π»ΠΈ обучСния Π˜ΡΡ‚ΠΎΡ‡Π½ΠΈΠΊ ΠΏΠ΅Ρ€Π΅ΠΌΠ΅Π½Π½ΠΎΠ³ΠΎ напряТСния ΠšΠΎΠ½Π΄Π΅Π½ΡΠ°Ρ‚ΠΎΡ€Ρ‹

    5 ΠŸΠΎΡΡ‚ΠΎΡΠ½Π½Ρ‹ΠΉ Ρ‚ΠΎΠΊ ΠΈ ΠΏΠ΅Ρ€Π΅ΠΌΠ΅Π½Π½Ρ‹ΠΉ Ρ‚ΠΎΠΊ
    ΠŸΠΎΡΡ‚ΠΎΡΠ½Π½Ρ‹ΠΉ Ρ‚ΠΎΠΊ (dc) остаСтся постоянным (ΠΊΠ°ΠΊ ΠΏΠΎ Π½Π°ΠΏΡ€Π°Π²Π»Π΅Π½ΠΈΡŽ, Ρ‚Π°ΠΊ ΠΈ ΠΏΠΎ Π²Π΅Π»ΠΈΡ‡ΠΈΠ½Π΅) Π²ΠΎ Π²Ρ€Π΅ΠΌΠ΅Π½ΠΈ.+ ve ΠŸΠ΅Ρ€Π΅ΠΌΠ΅Π½Π½Ρ‹ΠΉ Ρ‚ΠΎΠΊ (ac) ΡΠΈΠ½ΡƒΡΠΎΠΈΠ΄Π°Π»ΡŒΠ½ΠΎ измСняСтся со Π²Ρ€Π΅ΠΌΠ΅Π½Π΅ΠΌ. — Π²Π΅


    9 Как сохраняСтся заряд Π² диэлСктрикС
    ΠšΠΎΠ½Π΄Π΅Π½ΡΠ°Ρ‚ΠΎΡ€ состоит ΠΈΠ· Π΄Π²ΡƒΡ… ΠΏΡ€ΠΎΠ²ΠΎΠ΄Π½ΠΈΠΊΠΎΠ², Ρ€Π°Π·Π΄Π΅Π»Π΅Π½Π½Ρ‹Ρ… диэлСктриком (изолятором). ΠšΠΎΠ½Π΄Π΅Π½ΡΠ°Ρ‚ΠΎΡ€Ρ‹ Π½Π°ΠΊΠ°ΠΏΠ»ΠΈΠ²Π°ΡŽΡ‚ ΡΠ½Π΅Ρ€Π³ΠΈΡŽ Π² элСктричСском ΠΏΠΎΠ»Π΅. Π₯Ρ€Π°Π½Π΅Π½ΠΈΠ΅ ΠΎΠ·Π½Π°Ρ‡Π°Π΅Ρ‚, Ρ‡Ρ‚ΠΎ заряд остаСтся послС ΠΎΡ‚ΠΊΠ»ΡŽΡ‡Π΅Π½ΠΈΡ источника напряТСния. ΠŸΠΎΠΊΠ°Π·Π°Ρ‚Π΅Π»Π΅ΠΌ Π½Π°ΠΊΠΎΠΏΠ»Π΅Π½Π½ΠΎΠ³ΠΎ заряда являСтся Π΅ΠΌΠΊΠΎΡΡ‚ΡŒ C.Рис. 16-1: Π•ΠΌΠΊΠΎΡΡ‚ΡŒ сохраняСт заряд Π² диэлСктрикС ΠΌΠ΅ΠΆΠ΄Ρƒ двумя ΠΏΡ€ΠΎΠ²ΠΎΠ΄Π½ΠΈΠΊΠ°ΠΌΠΈ. (Π°) Π‘Ρ‚Ρ€ΡƒΠΊΡ‚ΡƒΡ€Π°.

    10 Π•Π΄ΠΈΠ½ΠΈΡ†Π° Смкости ΠΏΠΎ Π€Π°Ρ€Π°Π΄Ρƒ
    Π•ΠΌΠΊΠΎΡΡ‚ΡŒ C — это ΠΌΠ΅Ρ€Π° Смкости кондСнсатора Π² Π½Π°ΠΊΠΎΠΏΠ»Π΅Π½ΠΈΠΈ элСктричСского заряда ΠΏΠΎΠ΄ напряТСниСм. Π€Π°Ρ€Π°Π΄ (F) — это основная Π΅Π΄ΠΈΠ½ΠΈΡ†Π° измСрСния Смкости C. Одна Ρ„Π°Ρ€Π°Π΄Π° Смкости C Ρ€Π°Π²Π½Π° ΠΎΠ΄Π½ΠΎΠΌΡƒ ΠΊΡƒΠ»ΠΎΠ½Ρƒ заряда, Π½Π°ΠΊΠΎΠΏΠ»Π΅Π½Π½ΠΎΠ³ΠΎ Π² диэлСктрикС ΠΏΡ€ΠΈ ΠΏΡ€ΠΈΠ»ΠΎΠΆΠ΅Π½ΠΈΠΈ ΠΎΠ΄Π½ΠΎΠ³ΠΎ Π²ΠΎΠ»ΡŒΡ‚. C = Q / V Π‘ΠΎΠ»ΡŒΡˆΠΈΠ½ΡΡ‚Π²ΠΎ кондСнсаторов ΠΈΠΌΠ΅ΡŽΡ‚ значСния Смкости ΠΌΠ΅Π½Π΅Π΅ 1 Π€: C = 1 ΠΌΠΊΠ€ (ΠΌΠΈΠΊΡ€ΠΎΡ„Π°Ρ€Π°Π΄) = 1 Γ— 10-6 FC = 1 Π½Π€ (Π½Π°Π½ΠΎΡ„Π°Ρ€Π°Π΄) = 1 Γ— 10-9 FC = 1 ΠΏΠ€ (ΠΏΠΈΠΊΠΎΡ„Π°Ρ€Π°Π΄) = 1 Γ— F

    .

    Π”ΠΎΠ±Π°Π²ΠΈΡ‚ΡŒ ΠΊΠΎΠΌΠΌΠ΅Π½Ρ‚Π°Ρ€ΠΈΠΉ

    Π’Π°Ρˆ адрСс email Π½Π΅ Π±ΡƒΠ΄Π΅Ρ‚ ΠΎΠΏΡƒΠ±Π»ΠΈΠΊΠΎΠ²Π°Π½. ΠžΠ±ΡΠ·Π°Ρ‚Π΅Π»ΡŒΠ½Ρ‹Π΅ поля ΠΏΠΎΠΌΠ΅Ρ‡Π΅Π½Ρ‹ *